P02 - Capital Budgeting

240
MANAGEMENT ADVISORY SERVICES CAPITAL BUDGETING CASH FLOW COMPUTATIONS Cash Flow from Sale of Old Machine Sold at a Loss 43. Hoff is considering the sale of a machine with a book value of $80,000 and 3 years remaining in its useful life. Straight-line depreciation of $25,000 annually is available. The machine has a current market value of $50,000. What is the cash flow from selling the machine if the tax rate is 40%? a. $25,000 c . $62,000 b. $50,000 d. $80,000 D, L & H 9e 43. Hoff is considering the sale of a machine with a book value of $160,000 and 3 years remaining in its useful life. Straight-line depreciation of $50,000 annually is available. The machine has a current market value of $100,000. What is the cash flow from selling the machine if the tax rate is 40%? a. $50,000 c . $124,000 b. $100,000 d. $160,000 L & H 10e 72. Pebble Co. recently sold a used machine for $40,000. The machine had a book value of $60,000 at the time of the sale. What is the after-tax cash flow from the sale, assuming the company's marginal tax rate is 20 percent? a. $40,000 c . $44,000 b. $60,000 d. $32,000 Barfield 1 . The Alpha Beta Corporation disposes of a capital asset with an original cost of $85,000 and accumulated depreciation of $54,500 for $25,000. Alpha Beta's tax rate is 40%. Calculate the after-tax cash inflow from the disposal of the capital asset. (M) a. $2,200 c . $27,200 b. ($2,200) d. $31,500 Horngren Sold at a Gain 42. Acme is considering the sale of a machine with a book value of $80,000 and 3 years remaining in its useful life. Straight-line depreciation of $25,000 annually is available. The machine has a current market value of $100,000. What is the cash flow from selling the machine if the tax rate 40%. a. $25,000 c . $92,000 b. $80,000 d. $100,000 D, L & H 9e 42. Acme is considering the sale of a machine with a book value of $160,000 and 3 years remaining in its useful life. Straight-line depreciation of $50,000 annually is available. The machine has a current market value of $200,000. What is the cash flow from selling the machine if the tax rate is 40%? a. $50,000 c . $184,000 b. $160,000 d. $200,000 L & H 10e 2 . A corporation with a taxable income of $200,000 and a 40 percent tax rate is considering the sale of an asset. The original cost of the asset is $10,000, with $6,000 of its depreciated. How much total after-tax cash will be produced from the sale of the asset for $12,000? CMA EXAMINATION QUESTIONS Page 1 of 240

description

tb

Transcript of P02 - Capital Budgeting

Page 1: P02 - Capital Budgeting

MANAGEMENT ADVISORY SERVICES CAPITAL BUDGETING

CASH FLOW COMPUTATIONSCash Flow from Sale of Old MachineSold at a Loss43. Hoff is considering the sale of a machine with a book value of $80,000 and 3 years remaining

in its useful life. Straight-line depreciation of $25,000 annually is available. The machine has a current market value of $50,000. What is the cash flow from selling the machine if the tax rate is 40%?a. $25,000 c. $62,000b. $50,000 d. $80,000 D, L & H 9e

43. Hoff is considering the sale of a machine with a book value of $160,000 and 3 years remaining in its useful life. Straight-line depreciation of $50,000 annually is available. The machine has a current market value of $100,000. What is the cash flow from selling the machine if the tax rate is 40%?a. $50,000 c. $124,000b. $100,000 d. $160,000 L & H 10e

72. Pebble Co. recently sold a used machine for $40,000. The machine had a book value of $60,000 at the time of the sale. What is the after-tax cash flow from the sale, assuming the company's marginal tax rate is 20 percent?a. $40,000 c. $44,000b. $60,000 d. $32,000 Barfield

1. The Alpha Beta Corporation disposes of a capital asset with an original cost of $85,000 and accumulated depreciation of $54,500 for $25,000. Alpha Beta's tax rate is 40%. Calculate the after-tax cash inflow from the disposal of the capital asset. (M)a. $2,200 c. $27,200b. ($2,200) d. $31,500 Horngren

Sold at a Gain42. Acme is considering the sale of a machine with a book value of $80,000 and 3 years remaining

in its useful life. Straight-line depreciation of $25,000 annually is available. The machine has a current market value of $100,000. What is the cash flow from selling the machine if the tax rate 40%. a. $25,000 c. $92,000b. $80,000 d. $100,000 D, L & H 9e

42. Acme is considering the sale of a machine with a book value of $160,000 and 3 years remaining in its useful life. Straight-line depreciation of $50,000 annually is available. The machine has a current market value of $200,000. What is the cash flow from selling the machine if the tax rate is 40%?

a. $50,000 c. $184,000b. $160,000 d. $200,000 L & H 10e

2. A corporation with a taxable income of $200,000 and a 40 percent tax rate is considering the sale of an asset. The original cost of the asset is $10,000, with $6,000 of its depreciated. How much total after-tax cash will be produced from the sale of the asset for $12,000?a. $10,400 d. $(3,200)b. $12,000 e. $8,800c. $11,200 H & M

Initial Net Cash InvestmentAfter Tax Cash Inflow of Sale of Old Machine at a Gain36. Eyring Industries has a truck purchased seven years ago at a cost of $6,000. At the time of

purchase, the ultimate salvage value was estimated at $500, but salvage value was ignored in depreciation deductions. The truck is now fully depreciated. Assuming a tax rate of 40%, if the truck is sold for $500, the after-tax cash inflow for capital budgeting purposes will be: (E)a. $500. c. $200.b. $300. d. $100. G & N 9e

Old Machine Sold at Book Value50. Hatchet Company is considering replacing a machine with a book value of $400,000, a

remaining useful life of 5 years, and annual straight-line depreciation of $80,000. The existing machine has a current market value of $400,000. The replacement machine would cost $550,000, have a 5-year life, and save $75,000 per year in cash operating costs. If the replacement machine would be depreciated using the straight-line method and the tax rate is 40%, what would be the net investment required to replace the existing machine? (D)a. $90,000. c. $330,000b. $150,000 d. $550,000 D, L & H 9e

56. Big City Motors is trying to decide whether it should keep its existing car washing machine or purchase a new one that has technological advantages (which translate into cost savings) over the existing machine. Information on each machine follows:

Old machine New machineOriginal cost $9,000 $20,000Accumulated depreciation 5,000 0Annual cash operating costs 9,000 4,000Current salvage value of old machine 2,000Salvage value in 10 years 500 1,000Remaining life 10 yrs. 10 yrs.

The incremental cost to purchase the new machine isa. $11,000. c. $13,000.

CMA EXAMINATION QUESTIONS Page 1 of 155

Page 2: P02 - Capital Budgeting

MANAGEMENT ADVISORY SERVICES CAPITAL BUDGETING

b. $20,000. d. $18,000. Barfields

Old Machine Sold at a Gain3. Regal Industries is replacing a grinder purchased 5 years ago for $15,000 with a new one

costing $25,000 cash. The original grinder is being depreciated on a straight-line basis over 15 years to a zero salvage value. Regal will sell this old equipment to a third party for $6,000 cash. The new equipment will be depreciated on a straight-line basis over 10 years to a zero salvage value. Assuming a 40% marginal tax rate, Regal’s net cash investment at the time of purchase if the old grinder is sold and the new one is purchased is a. $19,000 c. $17,400b. $15,000 d. $25,000 CMA 1292 4-9

4. A machine that cost $50,000 and is fully depreciated is sold for $10,000. The $10,000 is then used as a down payment on the purchase of a new machine costing $75,000. Assuming a 40% tax rate, the out-of-pocket cost of the new machine is:A. $75,000 C. $65,000B. $71,000 D. $69,000 C & U

Old Equipment Sold at a Loss*. In making a decision to invest in a project the cash flow should be adjusted for their tax effect.

Assume an income tax rate of 35% an old machine with a book value of P70,000 will be replaced by a new machine costing P150,000. The market value of the old machine is P50,000. The after tax investment outlay is (E)a. P82,500 c. P107,000b. P93,000 d. P135,000 RPCPA 1085

*. In deciding the investment in a project, cash flows should be adjusted for their tax effect. Assume an income tax rate of 35%. An old equipment with a book value of P15,000 will be replaced by a new equipment costing P50,000. The market value of the old equipment is P11,000. The after-tax investment outlay is (E)a. P34,400 c. P39,000b. P37,600 d. P40,400 RPCPA 0581

Old Equipment Sold at a Loss, Additional Working Capital*. Diliman Republic Publishers, Inc. is considering replacing an old press that cost P800,000 six

years ago with a new one that would cost P2,250,000. Shipping and installation would cost an additional P200,000. The old press has a book value of P150,000 and could be sold currently for P50,000. The increased production of the new press would increase inventories by P40,000, accounts receivable by P160,000 and accounts payable by P140,000. Diliman Republic’s net initial investment for analyzing the acquisition of the new press assuming a 35% income tax rate would be (D)

a. P2,450,000 c. P2,600,000b. P2,425,000 d. P2,250,000 RPCPA 0595

44. Superstrut is considering replacing an old press that cost $80,000 six years ago with a new one that would cost $245,000. The old press has a net book value of $15,000 and could be sold for $5,000. The increased production of the new press would require an investment in additional working capital of $6,000. The company's tax rate is 40%. Superstrut's net investment now in the project would be: (M)a. $256,000. c. $250,000.b. $242,000. d. $245,000. CMA adapted

Old Machine Sold at a Loss, Cash Cost Savings on New Machine5. A company is considering replacing existing 2-year-old equipment. This project will require a

discounted cash flow analysis to determine if the benefits exceed the costs. Year-end data regarding the existing and new equipment are shown below.

Existing Equipment New EquipmentOriginal cost $600,000 $540,000Useful life (in years) 5 3Remaining life (in years) 3 3Annual depreciation $120,000 $180,000Accumulated depreciation $240,000 N/A*Book value $360,000 N/A*Current cash disposal value $100,000 N/A*

* Value is not applicable here.The new equipment will result in cash operating cost savings of $150,000 annually, before taxes. The new equipment would be purchased late in the current year to be operational at the beginning of the first year of the project. The existing equipment would be sold early in the first year of the project, meaning no further depreciation would be taken on it. The company has an effective income tax rate of 40%. Of the following, which are the correct figures to be used in the cash flow analysis for the first year of this proposed project? CIA 0596 III-78

After-Tax Cash Operating Savings

Proceeds from Saleof Existing Equipment

Tax Benefit from Saleof Existing Equipment

A. $90,000 $100,000 $0B. $60,000 $ 60,000 $104,000C. $90,000 $100,000 $104,000D. $60,000 $ 40,000 $156,000

Old Machine Traded-in6. A machine that cost $50,000 and is fully depreciated is allowed as a $10,000 trade-in on a

machine costing $75,000. Assuming a 40% tax rate, the out-of-pocket cost of the new machine is:

CMA EXAMINATION QUESTIONS Page 2 of 155

Page 3: P02 - Capital Budgeting

MANAGEMENT ADVISORY SERVICES CAPITAL BUDGETING

A. $75,000 C. $65,000B. $71,000 D. $69,000 C & U

Old Machine Traded-in, Avoidable Cost*. Key Corp. plans to replace a production machine that was acquired several years ago.

Acquisition cost is P450,000 with salvage value of P50,000. The machine being considered is worth P800,000 and the supplier is willing to accept the old machine at a trade-in value of P60,000. Should the company decide not to acquire the new machine, it needs to repair the old one at a cost of P200,000. Tax-wise, the trade-in transaction will not have any implication but the cost to repair is tax-deductible. The effective corporate tax rate is 35% of net income subject to tax. For purposes of capital budgeting, the net investment in the new machine is (M)a. P540,000 c. P660,000b. P610,000 d. P800,000 RPCPA 0597

Old Machine Traded-in, Other Assets Written Off, Avoidable Cost, Additional Working Capital*. Great Value Company is planning to purchase a new machine costing P50,000 with freight

and installation costs amounting to P1,500. The old unit is to be traded-in will be given a trade-in allowance of P7,500. Other assets that are to be retired as a result of the acquisition of the new machine can be salvaged and sold for P3,000. The loss on retirement of these other assets is P1,000 which will reduce income taxes of P400. If the new equipment is not purchased, repair of the old unit will have to be made at an estimated cost of P4,000. This cost can be avoided by purchasing the new equipment. Additional gross working capital of P12,000 will be needed to support operation planned with the new equipment.The net investment assigned to the new machine for decision analysis is (D)a. P50,200 c. P53,600b. P52,600 d. P57,600 RPCPA 1080

Initial Cash Outlay7. Lawson Inc. is expanding its manufacturing plant, which requires an investment of $4 million in

new equipment and plant modifications. Lawson's sales are expected to increase by $3 million per year as a result of the expansion. Cash investment in current assets averages 30% of sales; accounts payable and other current liabilities are 10% of sales. What is the estimated total investment for this expansion? (E)A. $3.4 million. C. $4.6 million.B. $4.3 million. D. $4.9 million. CMA 1295 4-8

8. Kline Corporation is expanding its plant, which requires an investment of $8 million in new equipment. Kline's sales are expected to increase by $6 million per year as a result of the expansion. Cash investment in current assets averages 30% of sales, and accounts payable

and other current liabilities are 10% of sales. What is the estimated total cash investment for this expansion? (E)A. $6.8 million. C. $9.2 million.B. $8.6 million. D. $9.8 million. Gleim

Tax on Sale of Old Machine9. A machine with a book value of $30,000 could be sold for $40,000. The corporation that owns

the machine has taxable income of $35,000 and a 40 percent tax rate. What would be the tax on the sale of the machine?a. $0 d. $4,000b. $10,000 e. $13,600c. $6,000 H & M

Opportunity Costs11. A firm owns a building with a book value of $100,000 and a market value of $250,000. If the

building is utilized for a project, then the opportunity cost ignoring taxes is: A. $100,000 C. $250,000B. $150,000 D. None of the above B & M

1 . ($85,000 - 54,500) = $30,500 - $25,000 = $5,500 loss x 0.4 = $2,200 tax savings from loss plus $25,000 proceeds = $27,200.

2 . Taxes = ($12,000 - $4,000) x 0.40 = $3,200Cash flow = $12,000 - $3,200 = $8,800

3 . Answer (C) is correct. The old machine has a book value of $10,000 [$15,000 cost - 5($15,000 cost ÷ 15 years) depreciation]. The loss on the sale is $4,000 ($10,000 - $6,000 cash received), and the tax savings from the loss is $1,600 (40% x $4,000). Thus, total inflows are $7,600. The only outflow is the $25,000 purchase price of the new machine. The net cash investment is therefore $17,400 ($25,000 - $7,600). Answer (A) is incorrect because $19,000 overlooks the tax savings from the loss on the old machine. Answer (B) is incorrect because $15,000 is obtained by deducting the old book value from the purchase price. Answer (D) is incorrect because the net investment is less than $25,000 given sales proceeds from the old machine and the tax savings.

4

Cost of new machine$75,000Less: After-tax inflow from old machine ($10,000 x .60) 6,000$69,000

5 . Answer (C) is correct. The amount of the after-tax cash operating savings is $90,000 [$150,000 pretax savings x (1.0 - .40)]. The proceeds from sale of existing equipment are given as $100,000. The tax benefit from sale of existing equipment is $104,000 [($360,000 carrying amount - $100,000 proceeds) x 40% tax rate].

CMA EXAMINATION QUESTIONS Page 3 of 155

Page 4: P02 - Capital Budgeting

MANAGEMENT ADVISORY SERVICES CAPITAL BUDGETING

12. A firm has a general-purpose machine which has a book value of $500,000 and is sold for $600,000 in the market. If the tax rate is 30%, what is the opportunity cost of using the machine in a project? A. $500,000 C. $570,000B. $600,000 D. None of the above B & M

Working Capital23. For project A in year 2, inventories increase by $16,000 and accounts payable by $4,000.

Calculate the increase or decrease in net working capital for year 2. (E)A. Increases by $12,000 C. Increases by $16,000B. Decreases by $12,000 D. Decreases by $16,000 B & M

24. For project X, year 5 inventories decrease by $5,000, accounts receivable by $3,000 and accounts payables by $2,000. Calculate the increase or decrease in working capital for year 5. (E)A. Increases by $6,000 C. Increases by $8,000B. Decreases by $6,000 D. Decreases by $7,000 B & M

34. A corporation is considering expanding operations to meet growing demand. With the capital

Answer (A) is incorrect because the sale of existing equipment will result in a loss that will produce a $104,000 tax benefit. Answer (B) is incorrect because $60,000 is the tax on the $150,000 of net profit arising from the $150,000 cost savings. Also, the $100,000 of proceeds from the equipment sale should not be reduced by the 40% tax rate to $60,000. Answer (D) is incorrect because $60,000 is the tax on the $150,000 of net income arising from the $150,000 cost savings. Also, $40,000 equals the tax on $100,000, and $156,000 equals 60% of $260,000.

6

Cost of new machine$75,000Less: Trade-in allowance 10,000$65,0007 . Answer (C) is correct. The investment required includes increases in working capital (e.g.,

additional receivables and inventories resulting from the acquisition of a new manufacturing plant). The additional working capital is an initial cost of the investment, but one that will be recovered (i.e., it has a salvage value equal to its initial cost). Lawson can use current liabilities to fund assets to the extent of 10% of sales. Thus, the total initial cash outlay will be $4.6 million {$4 million + [(30% - 10%) x $3 million sales]}. Answer (A) is incorrect because $3.4 million deducts the investment in working capital from the cost of equipment. Answer (B) is incorrect because $4.3 million equals $4 million plus 10% of $3 million. Answer (D) is incorrect because $4.9 million fails to consider the financing of 33-1/3% of the incremental current assets with accounts payable.

expansion, the current accounts are expected to change. Management expects cash to increase by $20,000, accounts receivable by $40,000, and inventories by $60,000. At the same time accounts payable will increase by $50,000, accruals by $10,000, and long-term debt by $100,000. The change in net working capital is (E)A. an increase of $120,000. C. a decrease of $120,000.B. a decrease of $40,000. D. an increase of $60,000. Gitman

35. A corporation is considering expanding operations to meet growing demand. With the capital expansion the current accounts are expected to change. Management expects cash to increase by $10,000, accounts receivable by $20,000, and inventories by $30,000. At the same time accounts payable will increase by $40,000, accruals by $30,000, and long-term debt by $80,000. The change in net working capital is (E)A. an increase of $10,000. C. a decrease of $90,000.B. a decrease of $10,000. D. an increase of $80,000. Gitman

Cash InflowBefore-tax Cash Inflow69. C Corp. faces a marginal tax rate of 35 percent. One project that is currently under evaluation

has a cash flow in the fourth year of its life that has a present value of $10,000 (after-tax). C Corp. assumes that all cash flows occur at the end of the year and the company uses 11 percent as its discount rate. What is the pre-tax amount of the cash flow in year 4? (Round to the nearest dollar.) (M)a. $15,181 c. $9,868b. $23,356 d. $43,375 Barfield

43. At the Bartholomew Company last year all sales were for cash and all expenses were paid in cash. The tax rate was 30%. If the after-tax net cash inflow from these operations last year was $10,500, and if the total before tax cash expenses were $35,000, then the total before-tax cash sales must have been: (M)a. $65,000. c. $45,000.b. $60,000. d. $50,000. G & N 9e

Timing of Cash Flow10. Assume that the interest rate is greater than zero. Which of the following cash-inflow streams

should you prefer? Gleim Year 1 Year 2 Year 3 Year 4A. $400 $300 $200 $100B. $100 $200 $300 $400C. $250 $250 $250 $250D. Any of these, since they each sum to $1,000.

CMA EXAMINATION QUESTIONS Page 4 of 155

Page 5: P02 - Capital Budgeting

MANAGEMENT ADVISORY SERVICES CAPITAL BUDGETING

After-tax Net Cash Inflow for a Certain Year5. You are given the following data for year 1. Revenue = $43; Total costs = $30; Depreciation =

$3; Tax rate = 30%. Calculate the cash flow for the project for year 1. (E)A. $7 C. $13B. $10 D. None of the above B & M

6. You are given the following data for year 1: Revenues = 100, Fixed costs = 30; Total variable costs = 50; Depreciation = $10; Tax rate = 30%. Calculate the after tax cash flow for the project for year 1. (E)A. $17 C. $10B. $7 D. None of the above B & M

After-Tax Net Cash Inflow40. Last year the sales at Jersey Company were $200,000 and were all cash sales. The expenses

at Jersey were $125,000 and were all cash expenses. The tax rate was 30%. The after-tax net cash inflow at Jersey last year from these operations was: (E)a. $37,500. c. $22,500.b. $60,000. d. $52,500. G & N 9e

41. Last year a firm had taxable cash receipts of $800,000 and the tax rate was 30%. The after-tax net cash inflow from these receipts was (E)a. $800,000. c. $560,000.b. $640,000. d. $240,000. G & N 9e

46. Last year the sales at Seidelman Company were $700,000 and were all cash sales. The company's expenses were $450,000 and were all cash expenses. The tax rate was 35%. The after-tax net cash inflow at Seidelman last year was: (E)a. $700,000. c. $162,500.b. $250,000. d. $87,500. G & N 9e

11. A project is expected to result in the following adjustments over the next year: Cash sales increase by 400,000. Expenses (except depreciation) increase by 180,000. Depreciation increases by 80,000.Assume the corporate tax rate is 34%. The total relevant net cash flows during that year are A. 92,400 C. 172,400B. 140,000 D. 220,000 CIA 0591 IV-52

Annual Cash Inflow12. A company considers a project that will generate cash sales of $50,000 per year. Fixed costs

will be $10,000 per year, variable costs will be 40% of sales, and depreciation of the

equipment in the project will be $5,000 per year. Taxes are 40%. The expected annual cash flow to the company resulting from the project is A. $15,000 C. $19,000B. $9,000 D. $14,000 CIA 1193 IV-50

*. Guemon Company is taking into account the replacement of an old machine now in use with a new machine costing P100,000. The replacement is expected to produce an annual cash savings of P22,500 before income taxes.The estimated useful life of the new machine is ten years with no residual value. The book value of the old machine is P37,500 and is expected to last for another five years. It is being depreciated at P8,000 per year. The income tax rate is 25%.The annual cash savings after tax is (M)a. P15,375 c. P17,375b. P16,875 d. P20,520 RPCPA 0583

Total Cash Inflow13. The Phenom Corporation has an annual cash inflow from operations from its investment in a

capital asset of $50,000 for five years. The corporation's income tax rate is 40%. Calculate the five years total after-tax cash inflow from operations. (M)a. $250,000 c. $150,000b. $175,000 d. $50,000 Horngren

Cash OutflowBefore-Tax Cash Outflow39. Consider a machine which costs $115,000 now and which has a useful life of seven years.

This machine will require a major overhaul at the end of the fourth year which will cost "X" dollars. If the tax rate is 40%, and if the after-tax cash outflow for this overhaul is $3,600, then the amount of "X" in dollars is: (E)a. $6,000. c. $2,160.b. $9,000. d. $1,440. G & N 9e

“End-of-Life” Cash FlowBased on Internal Rate of Return*. A company is considering putting up P50,000 in a three-year project. The company’s

expected rate of return is 12%. The present value of P1.00 at 12% for one year is 0.893, for two years is 0.797, and for three years is 0.712. The cash flow, net of income taxes will be P18,000 (present value of P16,074) for the first year and P22,000 (present value of P17,534) for the second year. Assuming that the rate of return is exactly 12%, the cash flow, net of income taxes, for the third year would be (M)a. P7,120 c. P16,392b. P10,000 d. P23,022 RPCPA 1081

CMA EXAMINATION QUESTIONS Page 5 of 155

Page 6: P02 - Capital Budgeting

MANAGEMENT ADVISORY SERVICES CAPITAL BUDGETING

Machine Sold at a Gain, Working Capital Released14. Garfield Inc. is considering a 10-year capital investment project with forecasted revenues of

$40,000 per year and forecasted cash operating expenses of $29,000 per year. The initial cost of the equipment for the project is $23,000, and Garfield expects to sell the equipment for $9,000 at the end of the tenth year. The equipment will be depreciated over 7 years. The project requires a working capital investment of $7,000 at its inception and another $5,000 at the end of year 5. Assuming a 40% marginal tax rate, the expected net cash flow from the project in the tenth year is (D)a. $32,000 c. $20,000b. $24,000 d. $11,000 CMA 1292 4-10

Machine Sold at a Gain, Cost to Remove*. Lor Industries is analyzing a capital investment proposal for new machinery to produce a new

product over the next ten years. At the end of the ten years, the machinery must be disposed of with a zero net book value but with a scrap salvage value of P20,000. It will require some P30,000 to remove the machinery. The applicable tax rate is 35%. The appropriate “end-of-life” cash flow based on the foregoing information is (D)a. Inflow of P30,000. c. Outflow of P10,000.b. Outflow of P6,500. d. Outflow of P17,000. RPCPA 0596

Machine Sold at a Taxable Loss, Cost to Remove15. Kore Industries is analyzing a capital investment proposal for new equipment to produce a

product over the next 8 years. The analyst is attempting to determine the appropriate “end-of-life” cash flows for the analysis. At the end of 8 years, the equipment must be removed from the plant, and will have a net book value of zero, a tax basis of $75,000, a cost to remove of $40,000, and scrap salvage value of $10,000. Kore’s effective tax rate is 40%. What is the appropriate “end-of-life” cash flow related to these items that should be used in the analysis? (D)a. $45,000 c. $12,000b. $27,000 d. $(18,000) CMA 1295 4-15

16. Metrejean Industries is analyzing a capital investment proposal for new equipment to produce a product over the next 8 years. At the end of 8 years, the equipment must be removed from the plant and will have a net book value of $0, a tax basis of $150,000, a cost to remove of $80,000, and scrap salvage value of $20,000. Metrejean’s effective tax rate is 40%. What is the appropriate “end-of-life” cash flow related to these items that should be used in the analysis? (D)a. $90,000 c. $24,000b. $54,000 d. $(36,000) Gleim

Tax SavingsOn Depreciation17. If the tax rate is 40% and a company has $400,000 of income, a depreciation deduction of

$50,000 would result in a tax savings of a. $17,000 c. $30,000b. $20,000 d. $33,000 H & M

18. If the tax rate is 40% and a company has $400,000 of income, a depreciation deduction of $80,000 would result in a tax savings ofa. $52,800 c. $32,000b. $48,000 d. $27,200 H & M

25. If the depreciation amount is $100,000 and the marginal tax rate is 30%, then the tax shield due to depreciation is: (E)A. $333,333 C. $30,000B. $100,000 D. None of the above B & M

26. If the depreciation amount is 600,000 and the marginal tax rate is 35%, then the tax shield due to depreciation is: (E)A. $180,000 C. $210,000B. $600,000 D. None of the above B & M

45. Kane Company is in the process of purchasing a new machine for its production line. It is near the end of the year, and the machine is being offered at a special discount if purchased before the end of the year. Kane has determined that the depreciation deduction for tax purposes on the new machine for the year of purchase would be $13,000. The tax rate is 30%. If Kane purchases the machine and reports a positive net income for the year, then the tax savings from the deprecation tax shield related to this machine for the year of purchase would be: (E)a. $3,900. c. $13,000.b. $9,100. d. $0. G & N 9e

38. Suppose a machine that costs $80,000 has a useful life of 10 years. Also suppose that depreciation on the machine is $8,000 for tax purposes in year 4. The tax rate is 40%. The tax savings from the depreciation tax shield in year 4 would be: (E)a. $4,800 inflow. c. $4,800 outflow.b. $3,200 inflow. d. $3,200 outflow. G & N 9e

Incremental Before-tax Profit19. Maxwell Company has an opportunity to acquire a new machine to replace one of its present

machines. The new machine would cost $90,000, have a 5-year life, and no estimated salvage value. Variable operating costs would be $100,000 per year. The present machine

CMA EXAMINATION QUESTIONS Page 6 of 155

Page 7: P02 - Capital Budgeting

MANAGEMENT ADVISORY SERVICES CAPITAL BUDGETING

has a book value of $50,000 and a remaining life of 5 years. Its disposal value now is $5,000, but it would be zero after 5 years. Variable operating costs would be $125,000 per year. Ignore income taxes. Considering the 5 years in total, what would be the difference in profit before income taxes by acquiring the new machine as opposed to retaining the present one?A. $10,000 decrease C. $35,000 increaseB. $15,000 decrease D. $40,000 increase C & U

Net Cash FlowPresent Value of Net Cash Flow20. At the end of the next four years, a new machine is expected to generate net cash flows of

$8,000, $12,000, $10,000, and $15,000, respectively. What are the cash flows worth today if a 3% interest rate properly reflects the time value of money in this situation? (E)A. $41,556. C. $32,400.B. $47,700. D. $38,100. S, S & T

3. As a 19th century economist, you are faced with the following problem. The world's shipping fleet consists of steamships and sailing ships. Each can be used to carry cargo or passengers. The ships have similar sailing capacities but differ in their annual operating costs as follows:

Steam SailCargo $80,000 $95,000Passenger $90,000 100,000

Assume: (i) Fares are competitively determined, (ii) demand is not expected to change, (iii) each vessel has a life of 15 years, (iv) current salvage value of either ship (sailing or steam) is $114,091, and (v) Cost of capital is 10%, (vi) no taxes. What is the present value of a steam ship? A. $190,152 C. $609,486B. $251,326 D. None of the above B & M

5. As a 19th century economist, you are faced with the following problem. The world's shipping fleet consists of steamships and sailing ships. Each can be used to carry cargo or passengers. The ships have similar sailing capacities but differ in their annual operating costs as follows:

Steam SailCargo $80,000 $95,000Passenger $90,000 100,000

Assume: (i) Fares are competitively determined, (ii) demand is not expected to change, (iii) each vessel has a life of 15 years, (iv) current salvage value of either ship (sailing or steam) is $114,091, and (v) Cost of capital is 10%, (vi) no taxes. If the cost of carrying cargo by sailing ship were $75,000 per year, what would be the present value of a steamship?

A. $114,091 C. $215,000B. $152,814 D. None of the above B & M

Present Value of 1 ComputationsPresent Value of Taxable Cash Receipts24. A company anticipates a taxable cash receipt of $50,000 in year 4 of a project. The company's

tax rate is 30% and its discount rate is 12%. The present value of this future cash flow is closest to: (M)a. $22,243. c. $9,533.b. $35,000. d. $15,000. G & N 9e

25. A company anticipates a taxable cash receipt of $20,000 in year 3 of a project. The company's tax rate is 30% and its discount rate is 8%. The present value of this future cash flow is closest to: (M)a. $6,000. c. $14,000.b. $4,763. d. $11,114. G & N 9e

26. A company anticipates a taxable cash receipt of $50,000 in year 3 of a project. The company's tax rate is 30% and its discount rate is 14%. The present value of this future cash flow is closest to: (M)a. $10,125. c. $23,624.b. $35,000. d. $15,000. G & N 9e

Present Value of After-Tax Cash Expense27. A company anticipates a taxable cash expense of $10,000 in year 2 of a project. The

company's tax rate is 30% and its discount rate is 8%. The present value of this future cash flow is closest to: (M)a. ($3,000). c. ($7,000).b. ($2,572). d. ($6,001). G & N 9e

28. A company anticipates a taxable cash expense of $40,000 in year 2 of a project. The company's tax rate is 30% and its discount rate is 10%. The present value of this future cash flow is closest to: (M)a. ($23,140). c. ($12,000).b. ($9,917). d. ($28,000). G & N 9e

29. A company anticipates a taxable cash expense of $60,000 in year 2 of a project. The company's tax rate is 30% and its discount rate is 14%. The present value of this future cash flow is closest to: (M)a. ($13,850). c. ($32,318).b. ($42,000). d. ($18,000). G & N 9e

CMA EXAMINATION QUESTIONS Page 7 of 155

Page 8: P02 - Capital Budgeting

MANAGEMENT ADVISORY SERVICES CAPITAL BUDGETING

37. Suppose a machine costs $20,000 now, has an expected life of eight years, and will require a $7,000 overhaul at the end of the third year. If the tax rate is 40%, then the after-tax cost of this overhaul would be: (E)a. $12,000. c. $8,000.b. $4,200. d. $2,800. G & N 9e

42. A company had tax-deductible cash expenses of $650,000 last year and the tax rate was 30%. The after-tax net cash outflow for these expenses was: (E)a. $195,000. c. $650,000.b. $455,000. d. $390,000. G & N 9e

Present Value of Depreciation Tax Shield30. A company anticipates a depreciation deduction of $20,000 in year 2 of a project. The

company's tax rate is 30% and its discount rate is 12%. The present value of the depreciation tax shield resulting from this deduction is closest to: (M)a. $11,161. c. $6,000.b. $14,000. d. $4,783. G & N 9e

31. A company anticipates a depreciation deduction of $30,000 in year 3 of a project. The company's tax rate is 30% and its discount rate is 12%. The present value of the depreciation tax shield resulting from this deduction is closest to: (M)a. $21,000. c. $6,406.b. $14,947. d. $9,000. G & N 9e

32. A company anticipates a depreciation deduction of $70,000 in year 2 of a project. The company's tax rate is 30% and its discount rate is 14%. The present value of the depreciation tax shield resulting from this deduction is closest to: (M)a. $16,159. c. $21,000.b. $49,000. d. $37,704. G & N 9e

Present Value of Salvage Value21. The following information pertains to an investment:

Investment $120,000Annual revenues $70,000Annual variable costs $15,000Annual fixed out-of-pocket costs $11,000Salvage value $27,000Discount rate 16%Expected life of project 3 years

Ignoring income taxes, the present value of the salvage value is (rounded) (E)

a. $15,673 c. $17,307b. $17,550 d. $23,220

Present Value of Working Capital Released at End-of-Life77. A project under consideration by the White Corp. would require a working capital investment of

$200,000. The working capital would be liquidated at the end of the project's 10-year life. If White Corp. has an after-tax cost of capital of 10 percent and a marginal tax rate of 30 percent, what is the present value of the working capital cash flow expected to be received in year 10?a. $36,868 c. $53,970b. $77,100 d. $23,130 Barfield

33. A company needs an increase in working capital of $20,000 in project that will last 4 years. The company's tax rate is 30% and its discount rate is 10%. The present value of the release of the working capital at the end of the project is closest to: (M)a. $6,000. c. $9,562.b. $13,660. d. $14,000. G & N 9e

34. A company needs an increase in working capital of $50,000 in project that will last 4 years. The company's tax rate is 30% and its discount rate is 8%. The present value of the release of the working capital at the end of the project is closest to: (M)a. $36,751. c. $25,726.b. $15,000. d. $35,000. G & N 9e

35. A company needs an increase in working capital of $70,000 in project that will last 3 years. The company's tax rate is 30% and its discount rate is 8%. The present value of the release of the working capital at the end of the project is closest to: (M)a. $49,000. c. $38,898.b. $21,000. d. $55,568. G & N 9e

Present Value of Annuity of 1 ComputationPresent Value of Annuity of Depreciation Tax Shield70. The Salvage Co. is considering the purchase of a new ocean-going vessel that could

potentially reduce labor costs of its operation by a considerable margin. The new ship would cost $500,000 and would be fully depreciated by the straight-line method over 10 years. At the end of 10 years, the ship will have no value and will be sunk in some already polluted harbor. The Salvage Co.'s cost of capital is 12 percent, and its marginal tax rate is 40 percent. What is the present value of the depreciation tax benefit of the new ship? (Round to the nearest dollar.)a. $113,004 c. $169,506b. $282,510 d. $200,000 Barfield

CMA EXAMINATION QUESTIONS Page 8 of 155

Page 9: P02 - Capital Budgeting

MANAGEMENT ADVISORY SERVICES CAPITAL BUDGETING

78. B Company is considering two alternative ways to depreciate a proposed investment. The investment has an initial cost of $100,000 and an expected five-year life. The two alternative depreciation schedules follow:

Method 1 Method 2Year 1 depreciation $20,000 $40,000Year 2 depreciation $20,000 $30,000Year 3 depreciation $20,000 $20,000Year 4 depreciation $20,000 $10,000Year 5 depreciation $20,000 $0

Assuming that the company faces a marginal tax rate of 40 percent and has a cost of capital of 10 percent, what is the difference between the two methods in the present value of the depreciation tax benefit?a. $7,196 c. $2,878b. $0 d. $6,342 Barfield

Present Value of 1 & Annuity of 1 ComputationPresent Value of Salvage Value & Annual Net Cash InflowQuestions 1 and 2 are based on the following information. H & MThe following information pertains to an investment:

Investment $70,000Annual revenues $48,000Annual variable costs $16,000Annual fixed out-of-pocket costs $10,000Salvage value $ 6,000Discount rate 12%Expected life of project 8 years

22. Ignore income taxes. The present value of the salvage value is (rounded)a. $2,424 c. $3,114b. $2,869 d. $3,224

23. Ignore income taxes. The present value of the annual net cash inflows from operations is (rounded)a. $68,411 c. $102,442b. $76,269 d. $109,296

ComprehensiveQuestions 7 through 9 are based on the following information. CMA 1295 4-3 to 5The Moore Corporation is considering the acquisition of a new machine. The machine can be purchased for $90,000, it will cost $6,000 to transport to Moore’s plant and $9,000 to install. It is estimated that the machine will last 10 years, and it is expected to have an estimated salvage value

of $5,000. Over its 10-year life, the machine is expected to produce 2,000 units per year with a selling price of $500 and combined materials and labor costs of $450 per unit. Federal tax regulations permit machines of this type to be depreciated using the straight-line method over 5 years with no estimated salvage value. Moore has a marginal tax rate of 40%

24. What is the net cash outflow at the beginning of the first year that Moore Corporation should use in a capital budgeting analysis?a. $(85,000) c. $(96,000)b. $(90,000) e. $(105,000)

25. What is the net cash flow for the third year that Moore Corporation should use in a capital budgeting analysis? a. $68,400 c. $64,200b. $68,000 d. $79,000

26. What is the net cash flow for the tenth year of the project that Moore Corporation should use in a capital budgeting analysis? a. $100,000 c. $68,400b. $81,000 d. $63,000

Questions 10 through 13 are based on the following information. GleimThe Dickins Corporation is considering the acquisition of a new machine at a cost of $180,000. Transporting the machine to Dickins’ plant will cost an additional $18,000. It has a 10-year life and is expected to have a salvage value of $10,000. Furthermore, the machine is expected to produce 4,000 units per year with a selling price of $500 and combined direct materials and direct labor costs of $450 per unit. Federal tax regulations permit machines of this type to be depreciated using the straight-line method over 5 years with no estimated salvage value. Dickins has a marginal tax rate of 40%.

27. What is the net cash outflow at the beginning of the first year that Dickens should use in a capital budgeting analysis?a. $(170,000) c. $(192,000)b. $(180,000) d. $(210,000)

28. What is the net cash flow for the third year that Dickins Corporation should use in a capital budgeting analysis?a. $136,800 c. $128,400b. $136,000 d. $107,400

9 . ($40,000 - $30,000) x 0.40 = $4,000

CMA EXAMINATION QUESTIONS Page 9 of 155

Page 10: P02 - Capital Budgeting

MANAGEMENT ADVISORY SERVICES CAPITAL BUDGETING

29. What is the net cash flow for the tenth year of the project that Dickins should use in a capital budgeting analysis?a. $200,000 c. $136,800b. $158,000 d. $126,000

30. What is the approximate payback period on the new machine?a. 1.05 years. c. 1.33 yearsb. 1.54 years d. 2.22 years

ACCOUNTING RATE OF RETURNNumerator31. Lin Co. is buying machinery it expects will increase average annual operating income by

$40,000. The initial increase in the required investment is $60,000, and the average increase in required investment is $30,000. To compute the accrual accounting rate of return, what amount should be used as the numerator in the ratio? (E)a. $20,000 c. $40,000b. $30,000 d. $60,000

ARR on Net Initial InvestmentCash Flows Given, Ignore Income Tax46. An investment opportunity costing $150,000 is expected to yield net cash flows of $45,000

annually for five years. The cost of capital is 10%. The book rate of return would be (M)a. 10%. c. 30%.b. 20%. d. 33.3%. L & H 10e

50. An investment opportunity costing $80,000 is expected to yield net cash flows of $25,000 annually for four years. The cost of capital is 10%. The book rate of return would be (M)a. 10.0%. c. 21.3%.b. 12.5%. d. 32.0%. L & H 10e

*. The Habagat Inc. is planning to spend P600,000 for a machine that it will depreciate on a straight-line basis over a ten-year period with no terminal disposal price. The machine will generate cash flow from operations of P120,000 a year. Ignoring income taxes, what is the accounting rate of return on the net initial investment? (M)a. 5% c. 10%b. 12% d. 15% RPCPA 0595

32. A project requires an investment of $80,000 in equipment. Annual cash inflows of $16,000 are expected to occur for the next 8 years. No salvage value is expected. The company uses the straight-line method of depreciation with no mid-year convention. Ignore income taxes.The accounting rate of return on original investment for the project is (M)

a. 6.25% c. 16.00%b. 7.50% d. 20.00% H & M

*. Doro Co. is considering the purchase of a $100,000 machine that is expected to result in a decrease of $25,000 per year in cash expenses after taxes. This machine, which has no residual value, has an estimated useful life of 10 years and will be depreciated on a straight-line basis. For this machine, the accounting rate of return based on the initial investment will be (M)a. 10% c. 25%b. 15% d. 35% AICPA 1189 II-40

68. (Ignore income taxes in this problem.) The Jason Company is considering the purchase of a machine that will increase revenues by $32,000 each year. Cash outflows for operating this machine will be $6,000 each year. The cost of the machine is $65,000. It is expected to have a useful life of five years with no salvage value. For this machine, the simple rate of return is: (E)a. 20%. c. 49.2%.b. 40%. d. 9.2%. G & N 9e

Old Machine has Scrap Value, Net Cost Savings, Ignore Income Tax67. (Ignore income taxes in this problem.) Denny Corporation is considering replacing a

technologically obsolete machine with a new state-of-the-art numerically controlled machine. The new machine would cost $450,000 and would have a ten-year useful life. Unfortunately, the new machine would have no salvage value. The new machine would cost $20,000 per year to operate and maintain, but would save $100,000 per year in labor and other costs. The old machine can be sold now for scrap for $50,000. The simple rate of return on the new machine is closest to: (M)a. 8.75%. c. 7.78%.b. 20.00%. d. 22.22%. G & N 9e

After-tax Cash Flow Given*. Benny Company is planning to purchase a new machine for P600,000. The new machine will

be depreciated on the straight-line basis over six-year period with no salvage, and a full year’s depreciation will be taken in the year of acquisition. The new machine is expected to produce cash flow from operations, net of income taxes, of P150,000 a year in each of the next six years. The accounting (book value) rate of return on the initial investment is expected to be (D)a. 16.7% c. 8.3%b. 12.0% d. 25.0% RPCPA 0598

33. Hooker Oak Furniture Company is considering the purchase of wood cutting equipment. Data on the equipment are as follows:

CMA EXAMINATION QUESTIONS Page 10 of 155

Page 11: P02 - Capital Budgeting

MANAGEMENT ADVISORY SERVICES CAPITAL BUDGETING

Original investment $30,000Net annual cash inflow $12,000Expected economic life in years 5Salvage value at the end of five years $3,000

The company uses the straight-line method of depreciation with no mid-year convention.What is the accounting rate of return on original investment rounded off to the nearest percent, assuming no taxes are paid? (D)a. 40.0% d. 24.0%b. 72.7% e. 22.0%c. 20.0% H & M

ARR on Average InvestmentCash Flows Given, Ignore Income Tax66. Microsoft Co. is considering the purchase of a $100,000 machine that is expected to result in a

decrease of $15,000 per year in cash expenses. This machine, which has no residual value, has an estimated useful life of 10 years and will be depreciated on a straight-line basis. For this machine, the accounting rate of return would be (M)a. 10 percent. c. 30 percent.b. 15 percent. d. 35 percent. Barfield

72. Mat Company is negotiating to purchase equipment that would cost P200,000 with the expectation that P40,000 per year could be saved in after-tax cash costs if the equipment were acquired. The equipment’s estimated useful life is 10 years, with no salvage value, and would be depreciated by the straight-line method. Mat’s minimum desired rate of return is 12%. Present value of an annuity of 1 at 12% for 10 periods is 5.65. Present value of 1 due in 10 periods at 12% is 0.322.The average accrual accounting rate of return during the first year of asset’s use isA. 20.0% C. 10.0%B. 10.5% D. 40.0% Pol Bobadilla

Required InvestmentARR based on Initial Investment Given34. The Mutya ng Pasig Company, a calendar company, purchased a new machine for P280,000

on January 1. Depreciation for tax purposes will be P35,000 annually for eight years. The accounting (book value) rate of return (ARR) is expected to be 20% on the initial increase in required investment. On the assumption of a uniform cash inflow, this investment is expected to provide annual cash flow from operations, before 30 percent income taxes, of (M)A. P80,000 C. P115,000B. P91,000 D. P175,000 Pol Bobadilla

ARR based on Average Investment Given35. The Zambales Co. is planning to purchase a new machine which it will depreciate for book

purposes, on a straight-line basis over a ten-year period with no salvage value and a full year-‘s depreciation taken in the year of acquisition. The new machine is expected to product cash flow from operations, net of income taxes, of P175,000 a year in each of the next ten years. The accounting (book value) rate of return on the average investment is expected to be 15%. How much will the new machine cost? (M)A. P1,000,000 C. P1,666,667B. P700,000 D. P1,800,000 Pol Bobadilla

Required Life83. The IRV Company has made an investment in video and recording equipment that costs

P106,700. The equipment is expected to generate cash inflows of P20,000 per year. How many years will the equipment have to be used to provide the company with a 10% average accounting rate of return on its investment?A. 7.28 years C. 5.55 yearsB. 9.05 years D. 4.75 years. Pol Bobadilla

Sensitivity Analysis*. Lyben Inc. is planning to produce a new product. To do this, it is necessary to acquire a new

equipment that will cost the company P100,000. The estimated life of the new equipment is five years with no salvage value. The estimated income and costs based on expected sales of P10,000 units per year are:

Sales @ P10.00 per unit P100,000Costs @ P8.00 per unit 80,000

CMA EXAMINATION QUESTIONS Page 11 of 155

Page 12: P02 - Capital Budgeting

MANAGEMENT ADVISORY SERVICES CAPITAL BUDGETING

Net income P 20,000The accounting rate of return based on initial investment is 20%What will be the accounting rate of return based on initial investment of P100,000 if management decrease its selling price of the new product by 10%? (M)a. 5% c. 15%b. 10% d. 20% RPCPA 1077

PAYBACK PERIODInitial Investment*. APJ, Inc. is planning to purchase a new machine that will take six years to recover the cost.

The new machine is expected to produce cash flow from operations, net of income taxes, of P4,500 a year for the first three years of the payback period and P3,500 a year of the last three years of the payback period. Depreciation of P3,000 a year shall be charged to income of the six years of the payback period. How much shall the machine cost? (M)a. P12,000 c. P24,000b. P18,000 d. none of these RPCPA 1087

92. Louis recently invested in a project that has an expected annual cash inflow of $7,000 for 10 years, and an expected payback period of 3.6 years. How much did Louis invest in the project?a. $19,444 c. $25,200b. $36,000 d. $40,000 Barfield

Minimum Annual Before-tax Operating Cash Savings36. Whatney Company is considering the acquisition of a new, more efficient press. The cost of

the press is $360,000, and the press has an estimated 6-year life with zero salvage value. Whatney uses straight-line depreciation for both financial reporting and income tax reporting purposes and has a 40% corporate income tax rate. In evaluating equipment acquisition of this type, Whatney uses a goal of a 4-year payback period. To meet Whatney’s desired payback period, the press must produce a minimum annual before-tax operating cash savings of (M)a. $90,000 c. $114,000b. $110,000 d. $150,000 CMA 1296 4-13

37. Jasper Company has a payback goal of 3 years on new equipment acquisitions. A new sorter is being evaluated that costs $450,000 and has a 5-year life. Straight-line depreciation will be used; no salvage is anticipated. Jasper is subject to a 40% income tax rate. To meet the company’s payback goal. The sorter must generate reductions in annual cash operating costs of (M)a. $60,000 c. $150,000b. $100,000 d. $190,000 CMA 0693 4-30

Given Net Income, Ignore Income Tax62. (Ignore income taxes in this problem.) Jarvey Company is studying a project that would have a

ten-year life and would require a $450,000 investment in equipment that has no salvage value. The project would provide net income each year as follows for the life of the project:

Sales $500,000Less cash variable expenses 200,000Contribution margin 300,000Less fixed expenses:

Fixed cash expenses $150,000Depreciation expenses 45,000 195,000

Net income $105,000The company's required rate of return is 12%. What is the payback period for this project? (E)a. 3 years c. 4.28 yearsb. 2 years d. 9 years G & N 9e

82. The Silverbowl Company is considering the purchase of electronic pinball machines to place in amusement houses. The machines would cost a total of P300,000, have an eight-year useful life, and have a total salvage value of P20,000. Based on experience with other equipment, the company estimates that annual revenues and expenses associated with the machines would be as follows:

Revenues from use P200,000Less operating expenses Commissions to amusement houses P100,000 Insurance 7,000 Depreciation 35,000 Maintenance 18,000 160,000Operating Income 40,000

Ignoring the effect of income taxes, the payback period for the pinball machines would be (M)A. 3.73 years. C. 4.0 years.B. 3.23 years. D. 7.5 years. Pol Bobadilla

Given Cash Flow, Ignore Income TaxEven Cash Flows44. An investment opportunity costing $85,000 is expected to yield net cash flows of $22,000

annually for five years. The payback period of the investment is (E)a. 0.26 years. c. $63,000.b. 3.86 years. d. All of the above. D, L & H 9e

44. An investment opportunity costing $55,000 is expected to yield net cash flows of $22,000 annually for five years. The payback period of the investment is

CMA EXAMINATION QUESTIONS Page 12 of 155

Page 13: P02 - Capital Budgeting

MANAGEMENT ADVISORY SERVICES CAPITAL BUDGETING

a. 0.4 years. c. $33,000.b. 2.5 years. d. some other number. L & H 10e

48. An investment opportunity costing $200,000 is expected to yield net cash flow of $44,000 annually for seven years. The payback period of the investment is (E)a. 0.22 years. c. 4.55 years.b. 3.08 years. d. Some other number. D, L & H 9e

48. An investment opportunity costing $100,000 is expected to yield net cash flows of $22,000 annually for seven years. The payback period of the investment is (E)a. 0.22 years. c. 4.55 years.b. 3.08 years. d. some other number. L & H 10e

38. The net initial investment for a piece of construction equipment is $1,000,000. Annual cash inflows are expected to increase by $200,000 per year. The equipment has an 8-year useful life. What is the payback period? (E)a. 8.00 years c. 6.00 yearsb. 7.00 years d. 5.00 years Horngren

9. A firm is evaluating a proposal which has an initial investment of $50,000 and has cash flows of $15,000 per year for five years. The payback period of the project is (E)A. 1.5 years. C. 3.3 years.B. 2 years. D. 4 years. Gitman

*. Assuming that capital requirement for new machinery is P135,000; net sale of present equipment – P35,000; average annual operating cash flow – P25,000. The payback period would be: (E)a. 4 years c. 5.4 yearsb. 4.5 years d. 6.8 years RPCPA 1078

64. (Ignore income taxes in this problem.) A company with $800,000 in operating assets is considering the purchase of a machine that costs $75,000 and which is expected to reduce operating costs by $20,000 each year. The payback period for this machine in years is closest to: (E)a. 0.27 years. c. 3.75 years.b. 10.7 years. d. 40 years. G & N 9e

39. An investment project is expected to yield $10,000 in annual revenues, will incur $2,000 in fixed costs per year, and requires an initial investment of $5,000. Given a cost of goods sold of 60% of sales and ignoring taxes, what is the payback period in years? (M)a. 2.50 c. 2.00

b. 5.00 d. 1.25 CIA 0586 IV-25

65. (Ignore income taxes in this problem.) The Higgins Company has just purchased a piece of equipment at a cost of $120,000. This equipment will reduce operating costs by $40,000 each year for the next eight years. This equipment replaces old equipment that was sold for $8,000 cash. The new equipment has a payback period of: (E)a. 8.0 years. c. 10.0 years.b. 2.8 years. d. 3.0 years. G & N 9e

40. Haig Aircraft is considering a project which has an up-front cost paid today at t = 0. The project will generate positive cash flows of $60,000 a year at the end of each of the next five years. The project’s NPV is $75,000 and the company’s WACC is 10 percent. What is the project’s simple, regular payback? (M)a. 3.22 years d. 2.35 yearsb. 1.56 years e. 4.16 yearsc. 2.54 years Brigham

41. Deming, Inc. is considering the purchase of production equipment that costs $600,000. The equipment is expected to generate annual cash inflows of $200,000 and have a useful life of 5 years with no salvage value. The firm’s cost of capital is 14 percent. The company uses the straight-line method of depreciation with no mid-year convention. Ignore income taxes.Payback for the project isa. 5.00 years c. 3.00 yearsb. 3.50 years d. 2.38 years H & M

63. (Ignore income taxes in this problem.) Buy-Rite Pharmacy has purchased a small auto for delivering prescriptions. The auto was purchased for $9,000 and will have a 6-year useful life and a $3,000 salvage value. Delivering prescriptions (which the pharmacy has never done before) should increase gross revenues by at least $5,000 per year. The cost of these prescriptions to the pharmacy will be about $2,000 per year. The pharmacy depreciates all assets using the straight-line method. The payback period for the auto is: (E)a. 3.0 years. c. 2.0 years.b. 1.8 years. d. 1.2 years. G & N 9e

45. Bryant is considering an investment in a new cheese-cutting machine to replace its existing cheese cutter. Information on the existing machine and the replacement machine follow:

Cost of new machine P40,000Net annual savings in operating costs 9,000Salvage value now of the old machine 6,000Salvage value of the old machine in 8 years 0Salvage value of the new machine in 8 years 5,000

CMA EXAMINATION QUESTIONS Page 13 of 155

Page 14: P02 - Capital Budgeting

MANAGEMENT ADVISORY SERVICES CAPITAL BUDGETING

Estimated life of the new machine 8 yearsWhat is the expected payback period for the new machine? (M)A. 4.44 years. C. 6.50 years.B. 2.67 years. D. 3.78 years. Pol Bobadilla

44. Salve Company is considering an investment in a new cheese-cutting machine to replace its existing cheese cutter. Information on the existing machine and the replacement machine follow:

Cost of the new machine P100,000Net annual savings in operating costs 20,000Salvage value now of the old machine 10,000Salvage value of the old machine in 8 years 0Salvage value of the new machine in 8 years 20,000Estimated life of the new machine 8 years

What is the expected payback period for the new machine?a. 4.00 years c. 4.50 yearsb. 4.33 years d. 5.00 years Pol Bobadilla

Uneven Cash Flows42. If an initial investment outlay is $60,000 and the cash flows projected are $15,000, $20,000,

$25,000, and $10,000 in each of the first four years, respectively, the payback period in years would be: (E)A. 3.3 D. 4.0B. 3.0 E. 5.0C. 2.5 C & U

8. A firm is evaluating a proposal which has an initial investment of $35,000 and has cash flows of $10,000 in year 1, $20,000 in year 2, and $10,000 in year 3. The payback period of the project is (E)A. 1 year. C. between 1 and 2 years.B. 2 years. D. between 2 and 3 years. Gitman

43. The Seattle Corporation has been presented with an investment opportunity which will yield cash flows of $30,000 per year in Years 1 through 4, $35,000 per year in Years 5 through 9, and $40,000 in Year 10. This investment will cost the firm $150,000 today, and the firm’s cost of capital is 10 percent. Assume cash flows occur evenly during the year, 1/365th each day. What is the payback period for this investment? (E)a. 5.23 years d. 6.12 yearsb. 4.86 years e. 4.35 yearsc. 4.00 years Brigham

44. Michigan Mattress Company is considering the purchase of land and the construction of a new plant. The land, which would be bought immediately (at t = 0), has a cost of $100,000 and the building, which would be erected at the end of the first year (t = 1), would cost $500,000. It is estimated that the firm’s after-tax cash flow will be increased by $100,000 starting at the end of the second year, and that this incremental flow would increase at a 10 percent rate annually over the next 10 years. What is the approximate payback period? (M)a. 2 years d. 8 yearsb. 4 years e. 10 yearsc. 6 years Brigham

45. A machine costing $1,000 produces total cash inflows of $1,400 over 4 years. Determine the payback period given the following cash flows: (E)

Year After-Tax Cash Flows Cumulative Cash Flows1 $400 $ 4002 300 7003 500 1,2004 200 1,400

a. 2 years. c. 2.86 years.b. 2.60 years. d. 3 years. CIA 1187 IV-19

*. Given these data: Net after tax inflows are: P24,000 for year 1, P30,000 for year 2, P36,000 for year 3, and

P30,00 for year 4. Initial investment outlay is P60,000. Cost of capital is 18%Determine the payback period for this investment (E)a. 2.50 years. c. 3.00 years.b. 2.17 years. d. 3.17 years. RPCPA 0594

66. (Ignore income taxes in this problem.) The Keego Company is planning a $200,000 equipment investment that has an estimated five-year life with no estimated salvage value. The company has projected the following annual cash flows for the investment.

Year Cash Inflows1 $120,0002 60,0003 40,0004 40,0005 40,000 Total $300,000

Assuming that the cash inflows occur evenly over the year, the payback period for the investment is: (E)

CMA EXAMINATION QUESTIONS Page 14 of 155

Page 15: P02 - Capital Budgeting

MANAGEMENT ADVISORY SERVICES CAPITAL BUDGETING

a. 0.75 years. c. 4.91 years.b. 1.67 years. d. 2.50 years. CMA adapted

*. Sweets, Etc., Inc. plans to undertake a capital expenditure requiring P2 million cash outlay. Below are the projected after-tax cash inflow for the five year period covering the useful life. The company’s tax rate is 35%.

Year P’0001 6002 7003 4804 4005 400

The founder and president of the candy company believes that the best gauge for capital expenditure is cash payback period and that the recovery period should not be more than 75% of the useful life of the project or the asset. Should the company undertake the project? (M)a. No, since the payback period is 4 years or 80% of the useful life of the project.b. Yes, since the payback period is 3.55 years or 71% of the useful life of the project.c. No, since the payback period extends beyond the life of the project. RPCPA 1096d. Yes, since the payback period is 4 years and still shorter than the useful life of the project.

46. Monck Management Services is considering an investment of $30,000. Data related to the investment are as follows:

Year Cash Inflows1 $10,0002 12,0003 15,0004 20,0005 10,000

Cost of capital is 18 percent.What is the payback period in years approximated to two decimal points assuming no taxes are paid?a. 3.00 d. 2.22b. 2.00 e. 5.00c. 2.53 H & M

47. Handy Products Company was considering the purchase of equipment. Details on the equipment are as follows:

Year Original Investment Cash Inflow from Operations0 $100,0001 $20,000

2 20,0003 30,0004 20,0005 30,000

What is the payback period in years, assuming no taxes are paid?a. 4.00 d. 3.85b. 4.33 e. 3.33c. 5.00 H & M

*. A company has two projects – Project Zoom and Project Oz. The table below shows the projected cash flows for the first five years of the project.

Project Zoom Project OzYear 0 -10,000 -15,000Year 1 +4,000 +7,600Year 2 +2,000 +3,500Year 3 +1,500 +3,900Year 4 +2,500 +2,000Year 5 +3,000 +1,000

Given the data above, compute the number of years within which the company may be able to recoup back its investments from both projects:a. 4 years; 3 years c. 5 years, 3 yearsb. 4 years; 5 years d. 5 years, 5 years RPCPA 1091

48. For $45,000, Harmon Company purchased a new machine with an estimated useful life of five years with no salvage value. The machine is expected to produce cash flow from operations, net of income taxes, as follows:

1st year $ 9,0002d year 12,0003d year 15,0004th year 9,0005th year 8,000

Harmon will use the sum-of-the-years-digits' method to depreciate the new machine as follows: 1st year $15,0002d year 12,000

CMA EXAMINATION QUESTIONS Page 15 of 155

Page 16: P02 - Capital Budgeting

MANAGEMENT ADVISORY SERVICES CAPITAL BUDGETING

3d year 9,0004th year 6,0005th year 3,000

What is the payback period? (EA. 3 years C. 5 yearsB. 4 years D. 2 years AICPA adapted

Given Before Tax Cash FlowEven Cash Flows*. Mary Company recently acquired a machine at a cost of P64,000. It will be depreciated on a

straight-line basis over eight years with no estimated salvage value. Mary estimates that this will produce an annual net cash inflow (before income taxes) of P18,000. Assuming an income tax rate of 35%, what is the approximate payback period for this investment? (E)a. 4.4 years. c. 7.1 years.b. 12.8 years. d. 3.6 years. RPCPA 0598

49. Fitzgerald Company is planning to acquire a $250,000 machine that will provide increased efficiencies, thereby reducing annual operating costs by $80,000. The machine will be depreciated by the straight-line method over a 5-year life with no salvage value at the end of 5 years. Assuming a 40% income tax rate, the machine's payback period is (E)A. 3.13 years. C. 3.68 years.B. 3.21 years. D. 4.81 years. CMA 0691 4-16

50. Jordan Company is considering the purchase of a new machine for $200,000. The machine generates annual revenues of $125,000 and annual expenses of $75,000 which includes $15,000 of depreciation. What is the payback period in years on the machine approximated to one decimal point?a. 1.6 d. 1.7b. 3.1 e. 2.1c. 4.0 H & M

Payback Reciprocal*. The payback reciprocal is an estimate of the internal rate of return. The Bravo, Inc. is

considering the acquisition of a merchandise picking system to improve customer service. Annual cash returns on investment cost of P1.2 million is P220,000. Useful life is estimated at 8 years. The company’s cost of capital is 14% and income tax rate is 35%.Calculate Bravo, Inc.’s payback reciprocal for this investment: (E)a. 20.5% c. 11.9%b. 18.3% d. 22.2% RPCPA 0594

Payback & Payback ReciprocalQuestions 48 and 49 are based on the following information. RPCPA 0591If a machine costs P5,000 and will generate annual cash inflows of P1,000 for the next 8 years,

*. What is the payback period?a. 8 years c. 6 yearsb. 5 years d. 3 years

*. What is the payback reciprocal? (E)a. 125% c. 20%b. 15% d. 33%

Questions 71 through 73 are based on the following information. GleimHenderson Inc. has purchased a new fleet of trucks to deliver its merchandise. The trucks have a useful life of 8 years and cost a total of $500,000. Henderson expects its next increase in after-tax cash flow to be $150,000 in Year 1, $175,000 in Year 2, $125,000 in Year 3, and $100,000 in each of the remaining years.

51. Ignoring the time value of money, how long will it take Henderson to recover the amount of investment.a. 3.5 years. c. 4.2 years.b. 4.0 years. d. 5 years.

52. What is the payback reciprocal for the fleet of trucks?a. 29% c. 24%b. 25% d. 20%

53. Assume the net cash flow to be $130,000 a year. What is the payback time for the fleet of trucks?a. 3 years. c. 3.85 years.b. 3.15 years. d. 4 years.

BAILOUT PAYBACK37. The manufacture of folic acid is a competitive business. A new plant costs $100,000 and lasts

for three years. The cash flow from the plant is as follows: Year 1: +43,300, Year 2: $43,300 and Year 3 = 58,300. If the salvage value of the plant at the end of year 1 is $80,000, would you scrap the plant at the end of year 1? Assume there is no tax. A. Yes C. Need more informationB. No D. Don't know B & M

CMA EXAMINATION QUESTIONS Page 16 of 155

Page 17: P02 - Capital Budgeting

MANAGEMENT ADVISORY SERVICES CAPITAL BUDGETING

40. The manufacture of folic acid is a competitive business. A new plant costs $100,000 and lasts for three years. The cash flow from the plant is as follows: Year 1: +43,300, Year 2: $43,300 and Year 3 = 58,300. If the salvage value of the plant at the end of year 2 is $60,000, would you scrap the plant at the end of year 2? Assume there is no tax. A. Yes C. Don't knowB. No D. Need more information B & M

39. The manufacture of folic acid is a competitive business. A new plant costs $100,000 and lasts for three years. The cash flow from the plant is as follows: Year 1: +43,300, Year 2: $43,300 and Year 3 = 58,300. If the salvage value of the plant at the end of year 2 is $40,000, would you scrap the plant at the end of year 2? Assume there is no tax. A. Yes C. Don't knowB. No D. Need more information B & M

54. Womark Company purchased a new machine on January 1 of this year for $90,000, with an estimated useful life of 5 years and a salvage value of $10,000. The machine will be depreciated using the straight-line method. The machine is expected to produce cash flow from operations, net of income taxes, of $36,000 a year in each of the next 5 years. The new machine’s salvage value is $20,000 in years 1 and 2, and $15,0000 in years 3 and 4. What will be the bailout period (rounded) for the new machine? (E)a. 1.4 years. c. 1.9 years.b. 2.2 years. d. 3.4 years. AICPA 0582 I-36

DISCOUNTED PAYBACKEven Cash Flow55. Coughlin Motors is considering a project with the following expected cash flows:

Year Project Cash Flow0 -$700 million1 200 million2 370 million3 225 million4 700 million

The project’s WACC is 10 percent. What is the project’s discounted payback? (M)a. 3.15 years d. 2.58 yearsb. 4.09 years e. 3.09 years Brighamc. 1.62 years

Uneven Cash Flow17. Given the following cash flows for project Z: C0 = -2,000, C1 = 600, C2 = 2160 and C3 = 6000,

calculate the discounted payback period for the project at a discount rate of 20%. (E)A. One year C. 3 years

B. 2 years D. None of the above B & M

56. A project has the following cash flows:Year Project Cash Flow0 -$3,0001 1,0002 1,0003 1,0004 1,000

Its cost of capital is 10 percent. What is the project’s discounted payback period? (E)a. 3.00 years d. 3.75 yearsb. 3.30 years e. 4.75 yearsc. 3.52 years Brigham

57. Lloyd Enterprises has a project which has the following cash flows:Year Project Cash Flow0 -$200,0001 50,0002 100,0003 150,0004 40,0005 25,000

The cost of capital is 10 percent. What is the project’s discounted payback? (M)a. 1.8763 years d. 2.4793 yearsb. 2.0000 years e. 2.6380 yearsc. 2.3333 years Brigham

58. Polk Products is considering an investment project with the following cash flows:Year Project Cash Flow0 -$100,0001 40,0002 90,0003 30,0004 60,000

The company has a 10 percent cost of capital. What is the project’s discounted payback? (M)a. 1.67 years d. 2.49 yearsb. 1.86 years e. 2.67 yearsc. 2.11 years Brigham

59. Davis Corporation is faced with two independent investment opportun-ities. The corporation has an investment policy which requires acceptable projects to recover all costs within 3 years.

CMA EXAMINATION QUESTIONS Page 17 of 155

Page 18: P02 - Capital Budgeting

MANAGEMENT ADVISORY SERVICES CAPITAL BUDGETING

The corporation uses the discounted payback method to assess potential projects and utilizes a discount rate of 10 percent. The cash flows for the two projects are:

Year Project A Cash Flow Project B Cash Flow0 -$100,000 -$80,0001 40,000 50,0002 40,000 20,0003 40,000 30,0004 30,000 0

Which investment project(s) does the company invest in? (M)a. Project A only. c. Project A and Project B.b. Neither Project A nor Project B. d. Project B only. Brigham

NET PRESENT VALUENet Initial InvestmentEven Cash Inflows Given, Ignore Income Taxes60. Shirt Company wants to purchase a new cutting machine for its sewing plant. The investment

is expected to generate annual cash inflows of $300,000. The required rate of return is 12% and the current machine is expected to last for four years. What is the maximum dollar amount Shirt Company would be willing to spend for the machine, assuming its life is also four years? Income taxes are not considered. (M)a. $507,000 c. $791,740b. $720,600 d. $911,100 Horngren

54. (Ignore income taxes in this problem.) Stratford Company purchased a machine with an estimated useful life of seven years. The machine will generate cash inflows of $90,000 each year over the next seven years. If the machine has no salvage value at the end of seven years, and assuming the company's discount rate is 10%, what is the purchase price of the machine if the net present value of the investment is $170,000? (M)a. $221,950. c. $268,120.b. $170,000. d. $438,120. G & N 9e

Even After-tax Cash Flows Given*. Garwood Company has purchased a machine that will be depreciated on the straight-line

basis over an estimated useful life of 7 years with no salvage value. The machine is expected to generate cash flow from operations, net of income taxes, of $80,000 in each of the 7 years. Garwood’s expected rate of return is 12%. Information on present value factors is as follows:

Present value of $1 at 12% for seven periods 0.452Present value of an ordinary annuity of $1 at 12% for seven periods 4.564

Assuming a positive net present value of $12,720, what was the cost of the machine? (M)a. $240,400 c. $352,400b. $253,120 d. $377,840 AICPA 1181 I-39

*. It is the start of the year and St. Tropez Co. plans to replace its old sing-along equipment. These information are available:

Old NewEquipment cost P70,000 P120,000Current salvage value 10,000 -Salvage value, end of useful life 2,000 16,000Annual operating costs 56,000 38,000Accumulated depreciation 55,300 -Estimated useful life 10 years 10 years

The company’s income tax rate is 35% and its cost of capital is 12%. What is the present value of all the relevant cash flows at time zero? (D)a. (P54,000) c. (P120,000)b. (P110,000) d. (P124,700) RPCPA 0594

Uneven Cash Inflows Given, Ignore Income Taxes50. (Ignore income taxes in this problem.) Horn Corporation is considering investing in a four-year

project. Cash inflows from the project are expected to be as follows: Year 1, $2,000; Year 2, $2,200; Year 3, $2,400; Year 4, $2,600. If using a discount rate of 8%, the project has a positive net present value of $500, what was the amount of the original investment? (M)a. $1,411. c. $7,054.b. $2,411. d. $8,054. AICPA adapted

Net Present Value Given*. McIndon Corporation bought a major equipment which is depreciable over 7 years on a

straight-line basis without any salvage value. It is estimated that it would generate cash flow from operations, net of income taxes, of P800,000 in each of the seven years. The company’s expected rate of return is 12%. Based on estimates, the project has a net present value of P127,200. What is the cost of the equipment? (E)To facilitate computations, below are present value factors:Present value of P1 at 12% for seven years is 0.452.Present value of an ordinary annuity of P1 at 12% for seven years is 4.564.a. P3,651,200 c. P2,404,000b. P3,524,000 d. P3,778,400 RPCPA 1095

*. On January 1, a company invested in an asset with a useful life of 3 years. The company’s expected rate of return is 10%. The cash flow and present and future value factors for the 3 years are as follows:

YearCash Inflow

From the AssetPresent Valueof $1 at 10%

Future Valueof $1 at 10%

1 $ 8,000 .91 1.10CMA EXAMINATION QUESTIONS Page 18 of 155

Page 19: P02 - Capital Budgeting

MANAGEMENT ADVISORY SERVICES CAPITAL BUDGETING

2 9,000 .83 1.213 10,000 .75 1.33

All cash inflows are assumed to occur at year-end. If the asset generates a positive net present value of $2,000, what was the amount of the original investment?a. $20,250 c. $30,991b. $22,250 d. $33,991 CIA 1185 IV-24

Required Salvage Value105.Cramden Armored Car Co. is considering the acquisition of a new armored truck. The truck is

expected to cost $300,000. The company's discount rate is 12 percent. The firm has determined that the truck generates a positive net present value of $17,022. However, the firm is uncertain as to whether it has determined a reasonable estimate of the salvage value of the truck. In computing the net present value, the company assumed that the truck would be salvaged at the end of the fifth year for $60,000. What expected salvage value for the truck would cause the investment to generate a net present value of $0? Ignore taxes. (D)a. $30,000 c. $55,278b. $0 d. $42,978 Barfield

Required Annual Cash Flows, Ignore Income TaxesNPV is Zero61. Investment A requires a net investment of $800,000. The required rate of return is 12% for the

four-year annuity. What are the annual cash inflows if the net present value equals 0? (rounded) (M)a. $189,483 c. $274,848b. $263,418 d. $ 295,733 Horngren

71. Salvage Co. is considering the purchase of a new ocean-going vessel that could potentially reduce labor costs of its operation by a considerable margin. The new ship would cost $500,000 and would be fully depreciated by the straight-line method over 10 years. At the end of 10 years, the ship will have no value and will be sunk in some already polluted harbor. The Salvage Co.'s cost of capital is 12 percent, and its marginal tax rate is 40 percent. If the ship produces equal annual labor cost savings over its 10-year life, how much do the annual savings in labor costs need to be to generate a net present value of $0 on the project? (Round to the nearest dollar.) (M)a. $68,492 c. $88,492b. $114,154 d. $147,487 Barfield

NPV is Positive49. (Ignore income taxes in this problem.) Benz Company is considering the purchase of a

machine that costs $100,000 and has a useful life of 18 years. The company's required

discount rate is 12%. If the machine's net present value is $5,850, then the annual cash inflows associated with the machine must be (round to the nearest whole dollar): (M)a. $42,413.b. $14,600.c. $13,760.d. it is impossible to determine from the data given. G & N 9e

43. (Ignore income taxes in this problem.) Given the following data:Present investment required $12,000Net present value $ 430Annual cost savings $ ?Discount rate 12%Life of the project 10 years

Based on the data given, the annual cost savings would be: (M)a. $1,630.00. c. $2,123.89.b. $2,200.00. d. $2,553.89. G & N 9e

NPV is Negative42. Highpoint, Inc., is considering investing in automated equipment with a ten-year useful life.

Managers at Highpoint have estimated the cash flows associated with the tangible costs and benefits of automation, but have been unable to estimate the cash flows associated with the intangible benefits. Using the company's 10% discount rate, the net present value of the cash flows associated with just the tangible costs and benefits is a negative $184,350. How large would the annual net cash inflows from the intangible benefits have to be to make this a financially acceptable investment? (M)a. $18,435. c. $35,000.b. $30,000. d. $37,236. G & N 9e

Required Life106.Booker Steel Inc. is considering an investment that would require an initial cash outlay of

$400,000 and would have no salvage value. The project would generate annual cash inflows of $75,000. The firm's discount rate is 8 percent. How many years must the annual cash flows be generated for the project to generate a net present value of $0? (M)a. between 5 and 6 years c. between 7 and 8 yearsb. between 6 and 7 years d. between 8 and 9 years Barfield

Present Value ComputationsCombined Projects45. Consider two projects A and B with present values PV(A. = 300 (discounted at 10%) and

PV(B. = 500 (discounted at 15%). The present value of the combined project is: A. 800 C. 300

CMA EXAMINATION QUESTIONS Page 19 of 155

Page 20: P02 - Capital Budgeting

MANAGEMENT ADVISORY SERVICES CAPITAL BUDGETING

B. 500 D. None of the above B & M

Differential Depreciation Tax Shield40. For P450,000, Mabini Corporation purchased a new machine with an estimated useful life of

five years with no salvage value. The machine is expected to produce cash flow from operations net of 40% income taxes (as shown below). Mabini will use the sum-of-the-years’-digits method to depreciate the new machine (as shown below).

After-Tax Cash Flow SYD DepreciationFirst year P160,000 P150,000Second year 140,000 120,000Third year 180,000 90,000Fourth year 120,000 60,000Fifth year 100,000 30,000

The present value of 1 for 5 periods at 12% is 3.60478. The present value of 1 at 12% at end of each period are: End of Period 1 – 0.8928; Period 2 – 0.79719, Period 3 – 0.71178; Period 4 – 0.63552; Period 5 – 0.56743.Had Mabini used straight-line method of depreciation, what is the difference in net present value provided by the machine at a discount rate of 12%?A. Increase of P9,750 C. Decrease of P24,376B. Decrease of P9,750 D. Increase of P24,376 Pol Bobadilla

17. Tristan Consulting, Inc. operates consulting offices in Manila, Olongapo, and Cebu. The firm is presently considering an investment in a new mainframe computer and communication software. The computer would cost P6 million and have an expected life of 8 years. For tax purposes, the computer can be depreciated using either straight-line method or Sum-of-the-Years’-Digits (SYD) method over five years. No salvage value is recognized in computing depreciation expense and no salvage value is expected at the end of the life of the equipment. The company’s cost of capital is 10% and its tax rate is 40%.The present value of an annuity of 1 for 5 periods is 3.791 and for 8 periods is 5.335. The present values of 1 at the end of each period are:

1 0.9091 5 0.62092 0.8264 6 0.56453 0.7513 7 0.51324 0.6830 8 0.4665

The present value of the net advantage using SYD method of depreciation with a five-year life- instead of straight-line method of depreciating the equipment isA. P86,224 C. P115,168B. P215,560 D. P287,893 Pol Bobadilla

43. Silliman Corporation purchased a new machine for P450,000. The new machine has an estimated useful life of five years with no salvage value. The machine is expected to produce cash flows from operations, net of 40 percent income taxes, as follows:

First year P160,000Second year 140,000Third year 180,000Fourth year 120,000Fifth year 100,000

Silliman will use the sum-of-the-years-digits’ method to depreciate the new machine as follows:First year P150,000Second year 120,000Third year 90,000Fourth year 60,000Fifth year 30,000

The present value of 1 for 5 periods at 12 percent is 3.60478. The present values of 1 at 12 percent at end of each period are:End of: Period 1 – 0.8928, Period 2 - 0.79719, Period 3 - 0.71178, Period 4 - 0.63552, Period 5 - 0.56743The net advantage (in present value) of using the Sum-of-the-Years’-Digits method over the straight-line method at a discount rate of 12 percent isa. P14,620 c. P 7,340b. P12,188 d. P 9,750 Pol Bobadilla

Uneven Cash Flows11. What is the present value of the following cash flow at a discount rate of 15% APR?

t=1 t=2-100,000 300,000

A. $185,000 C. $139,887B. $200,000 D. None of the above B & M

9. USSA company has an opportunity to invest in a gold mine. The initial investment is $150 million. The mine is estimated to produce 80,000 ounces of gold per year for the next ten years. The extraction cost of gold per ounce is $100 and the expected revenue is at that level. Suppose the current price of gold is $300 per ounce. The price of gold is expected to grow at 4% per year for the foreseeable future. If the approximate discount rate is 10%, then the present value of gold is: A. Less than $300 per ounce C. Greater than $300 per ounceB. $300 per ounce D. Not enough information B & M

CMA EXAMINATION QUESTIONS Page 20 of 155

Page 21: P02 - Capital Budgeting

MANAGEMENT ADVISORY SERVICES CAPITAL BUDGETING

10. USSA company has an opportunity to invest in a gold mine. The initial investment is $150 million. The mine is estimated to produce 80,000 ounces of gold per year for the next ten years. The extraction cost of gold per ounce is $100 and the expected revenue is at that level. Suppose the current price of gold is $290 per ounce. The price of gold is expected to grow at 5 % per year for foreseeable future. If the approximated discount rate is 8%, the present value of gold is: A. Less than $290 per ounce C. $290 per ounceB. Greater than $290 per ounce D. None of the above B & M

11. USSA company has an opportunity to invest in a gold mine. The initial investment is $150 million. The mine is estimated to produce 80,000 ounces of gold per year for the next ten years. The extraction cost of gold per ounce is $100 and the expected revenue is at that level. Suppose the current price of gold is $300 per ounce and the price of gold is expected to increase at a rate of 5% per year for the foreseeable future. What is the current value of 0.2 million ounces of gold to be produced each year for the next five years (the discount rate is 8% per year) A. $300 million C. 261.49 millionB. 275.70 million D. None of the above B & M

12. USSA company has an opportunity to invest in a gold mine. The initial investment is $150 million. The mine is estimated to produce 80,000 ounces of gold per year for the next ten years. The extraction cost of gold per ounce is $100 and the expected revenue is at that level. Suppose the current price of gold is $300 per ounce, the price of gold is expected to increase by 10% for the next two years and at 4% afterward. They discount rate is 10%, what is the current value of 0.2 million ounces of gold to be produced each year for the next 5 years? A. $300 million C. $261.49 millionB. 281.07 million D. None of the above B & M

Net Annual Cost Savings*. BRG Assembly, Inc. is considering the purchase of an automatic wirebonder which cost

P750,000. It has a ten year life without any salvage value. BRG would save P200,000 in labor costs annually as a result of the use of the new machine. Power cost would however increase P25,000 annually. The cost of capital is 16%. The present value factor for 10 years at 16% is 4.8332. The present value of the net annual cost savings is (E)a. P845,810 c. P745,810b. P575,000 d. P966,640 RPCPA 1097

62. Pole Co. is investing in a machine with a 3-year life. The machine is expected to reduce annual cash operating costs by $30,000 in each of the first 2 years and by $20,000 in year 3. Present values of an annuity of $1 at 14% are:

Period 1 2 3

Factor 0.88 1.65 2.32Using a 14% cost of capital, what is the present value of these future savings? (E)a. $59,600 c. $62,900b. $60,800 d. $69,500

63. Murray is planning a project that will cost $22,000. The annual cash inflow, net of income taxes, will be $5,000 a year for 7 years. The present value of $1 at 12% is as follows:

Period Present Value of $1 at 12%1 .8932 .7973 .7124 .6365 .5676 .5077 .452

Using a rate of return of 12%, what is the present value of the cash flow generated by this project? (E)a. $22,600 c. $34,180b. $22,820 d. $35,000 AICPA 1177 I-23

Cash Flow for Certain YearsQuestions 89 thru 91 are based on the following information. G & N 9e (Ignore income taxes in this problem.) The Becker Company is interested in buying a piece of equipment that it needs. The following data have been assembled concerning this equipment:

Cost of required equipment $250,000Working capital required $100,000Annual operating cash inflows $ 80,000Cash repair at end of 4 years $ 40,000Salvage value at end of 6 years $ 90,000

This equipment is expected to have a useful life of 6 years. At the end of the sixth year the working capital would be released for use elsewhere. The company's discount rate is 10%.

89. The present value of all future operating cash inflows is closest to: (M)a. $480,000. c. $348,400.b. $452,300. d. $278,700.

90. The present value of the net cash flows (all cash inflows less all cash outflows) occurring during year 4 is: (E)a. $40,000. c. $54,640.b. $27,320. d. $42,790.

CMA EXAMINATION QUESTIONS Page 21 of 155

Page 22: P02 - Capital Budgeting

MANAGEMENT ADVISORY SERVICES CAPITAL BUDGETING

91. The present value of the net cash flows (all cash inflows less all cash outflows) occurring during year 6 is closest to:a. $270,000. c. $107,200.b. $195,900. d. $152,300.

Certain Year & Net Present ValueQuestions 92 & 93 are based on the following information. G & N 9e (Ignore income taxes in this problem.) UR Company is considering rebuilding and selling used alternators for automobiles. The company estimates that the net operating cash flows (sales less cash operating expenses) arising from the rebuilding and sale of the used alternators would be as follows (numbers in parentheses indicate an outflow):

Years 1 - 10 ... $ 90,000Year 11 ........ (20,000)Year 12 ........ 100,000

In addition to the above net operating cash flows, UR Company would purchase production equipment costing $200,000 now to use in the rebuilding of the alternators. The equipment would have a 12-year life and a $15,000 salvage value. The company's discount rate is 10%.

92. The present value of the net operating cash flows (sales less cash operating expenses) arising from the rebuilding and sale of the alternators (rounded to the nearest dollar) is: (M)a. $582,735. c. $577,950.b. $596,735. d. $591,950.

93. The net present value of all cash flows associated with this investment (rounded to the nearest dollar) is: (M)a. $377,950. c. $392,950.b. $382,735. d. $362,950.

Risk-adjusted Discount RateQuestions 19 thru 21 are based on the following information. GitmanA firm is considering investment in a capital project which is described below. The firm's cost of capital is 18 percent and the risk-free rate is 6 percent. The project has a risk index of 1.5. The firm uses the following equation to determine the risk adjusted discount rate, RADR, for each project: RADR = Rf + Risk Index (Cost of capital - Rf)

Initial Investment $1,000,000

Year Cash Inflow 1 $500,000 2 500,000 3 500,000

19. The net present value without adjusting the discount rate for risk is A. $336,000. C. $179,400.B. $250,000. D. $87,000.

20. The discount rate that should be used in the net present value calculation to compensate for risk is A. 6 percent. C. 18 percent.B. 15 percent. D. 24 percent.

21. The net present value of the project when adjusting for risk is A. -$9,500. C. $87,000.B. $0. D. $105,000.

Net Present Value Computation - DerivationGiven Payback Period64. Shannon Industries is considering a project which has the following cash flows:

Year Project Cash Flow 0 ? 1 $2,000 2 3,000 3 3,000 4 1,500

The project has a payback of 2.5 years. The firm’s cost of capital is 12 percent. What is the project’s net present value NPV? (M)a. $ 577.68 d. $2,761.32b. $ 765.91 e. $3,765.91c. $1,049.80 Brigham

Given Profitability Index93. The McNally Co. is considering an investment in a project that generates a profitability index of

1.3. The present value of the cash inflows on the project is $44,000. What is the net present value of this project? (M)a. $10,154 c. $57,200b. $13,200 d. $33,846 Barfield

Net Present Value – Ignore Income TaxesGiven Present Value of Cash Inflow16. An initial investment of $400,000 will produce an end of year cash flow of $450,000. What is

the NPV of the project at a discount rate of 10%? (E)A. $9090.90 C. $0 (zero)B. $409,090.90 D. None of the above B & M

CMA EXAMINATION QUESTIONS Page 22 of 155

Page 23: P02 - Capital Budgeting

MANAGEMENT ADVISORY SERVICES CAPITAL BUDGETING

Combined Project4. If the net present value of project A is +$80, and of project B is +$60, then the net present

value of the combined project is: (E)A. +$80 C. +$140B. +$60 D. None of the above B & M

5. If the NPV of project A is +$100, and that of project B is -$50 and that of project C is +$20, what is the NPV of the combined project? (E)A. $100 C. $120B. -$50 D. $70 B & M

7. If the NPV of project A is +$50 and that of project B is -$60, then the NPV of the combined project is: (E)A. +$50 C. -$10B. +$60 D. None of the above. B & M

Single Cash Inflow15. If the present value of a cash flow generated by an initial investment of $100,000 is $120,000,

what is the NPV of the project? (E)A. $120,000 C. $100,000B. $20,000 D. None of the above B & M

Even Cash Flows, No Present Value Table43. An investment opportunity costing $75,000 is expected to yield net cash flows of $22,000

annually for five years. The NPV of the investment at a cutoff rate of 12% would bea. $(4,310) c. $75,000b. $4,310 d. $79,310 D, L & H 9e

43. An investment opportunity costing $75,000 is expected to yield net cash flows of $23,000 annually for five years. The NPV of the investment at a cutoff rate of 14% would be (E)a. $(3,959). c. $75,000.b. $3,959. d. $78,959. L & H 10e

47. An investment opportunity costing $110,000 is expected to yield net cash flows of $28,000 annually for six years. The NPV of the investment at a cutoff rate of 12% would be (E)a. $(5,108) c. $110,000b. $5,108 d. $115,108 D, L & H 9e

47. An investment opportunity costing $150,000 is expected to yield net cash flows of $36,000 annually for six years. The NPV of the investment at a cutoff rate of 12% would be (E)

a. $(2,004). c. $150,000.b. $2,004. d. $147,996. L & H 10e

51. The Whitton Company uses a discount rate of 16%. The company has an opportunity to buy a machine now for $18,000 that will yield cash inflows of $10,000 per year for each of the next three years. The machine would have no salvage value. The net present value of this machine to the nearest whole dollar is: (E)a. $22,460. c. $(9,980).b. $4,460. d. $12,000. G & N 9e

28. Valentine Company is considering investing in a new project. The project will need an initial investment of $1,200,000 and will generate $600,000 (after-tax) cash flows for three years. Calculate the NPV for the project if the cost of capital is 15%. (E)A. $16,994 C. $60,000B. $29,211 D. $25,846 B & M

32. Elephant company is investing in a giant crane. It is expected to cost 2.2 million in initial investment and it is expected to generate an end of year cash flow of 1.0 million each year for three years. Calculate the NPV at 12% (approximately). (E)A. 2.4 million C. 0.80 millionB. 0.20 million D. 0.40 million B & M

Even Cash Flow, With Present Value Table*. A project requires the purchase of an asset for P6,500. Annual cash benefits are P2,000 for 5

years. The firm’s cost of capital is 12% and the present value of an annuity of P1 for 5 periods at 12% is 3.605. The net present value of the project is: (E)a. P520 c. P710b. P650 d. P805 RPCPA 1078

*. Alang-alang sa Lahat Foundation, Inc. a nonstock, nonprofit and tax-exempt foundation invested P1 million in a five-year project at the beginning of the year. The foundation estimates that the annual savings form the project will amount to P325,000. The P1 million asset is depreciable over five (5) years on a straight-line basis. The foundation’s hurdle rate is 12%. To facilitate computations, below are present value factors:

N=5 12% 14% 16%Present value of P1 0.57 0.52 0.48Present value of an annuity of P1 3.60 3.40 3.30

The net present value of the project is (E)a. P170,000 c. P182,000b. P625,000 d. P450,000 RPCPA 1095

CMA EXAMINATION QUESTIONS Page 23 of 155

Page 24: P02 - Capital Budgeting

MANAGEMENT ADVISORY SERVICES CAPITAL BUDGETING

Questions 8, 9 and 10 are based on the following information. RPCPA 1077ABC Manufacturing Company is taking into account two alternative strategies to market a new product. To make known the product will necessitate an outlay of P75,000. With a low price the product will generate cash proceeds of P50,000 per year and will have a life of two years. With a high price the product will generate cash proceeds of P90,000 but will have a life of only one year. The cost of money for the company is 10%. The following is the table of Present Value of 10%

N Present Value of P1 Present Value of P1 Received per Period1 .9091 0.90912 .8264 1.73553 .7513 2.48694 .6830 3.16995 .6209 3.7908

*. The net present value of the low-price product strategy is:a. P11,775 c. P75,000b. P25,000 d. P175,000

*. The net present value of the high-price strategy is:a. P6,819 c. P65,000b. P15,342 d. P165,000

*. The marketing strategy that should be accepted is:a. The low-price strategy c. Either of the two marketing strategiesb. The high-price strategy d. Neither of the two marketing strategies

Even Cash Flow, Salvage Value, No Present Value Table.45. (Ignore income taxes in this problem.) A piece of equipment has a cost of $20,000. The

equipment will provide cost savings of $3,500 each year for ten years, after which time it will have a salvage value of $2,500. If the company's discount rate is 12%, the equipment's net present value is: (M)a. $580. c. $17,500.b. ($225). d. $2,275. G & N 9e

6. A new grocery store cost $30 million in initial investment. It is estimated that the store will generate 2 million dollars after tax cash flow for five years. At the end of 5 years it can be sold for $40 million. What is the NPV of the project at a discount rate of 10%? A. $2.42 million C. $.69 millionB. $18 million D. None of the above B & M

7. A new grocery store cost $30 million in initial investment. It is estimated that the store will generate 2 million dollars after tax cash flow for five years. At the end of 5 years it can be sold for $35 million. What is the NPV of the project at a discount rate of 10%?A. $2.42 million C. $-0.69 millionB. $18 million D. None of the above B & M

52. The following data pertain to an investment:Cost of the investment $18,955Life of the project 5 yearsAnnual cost savings $ 5,000Estimated salvage value $ 1,000Discount rate 10%

The net present value of the proposed investment is: (M)a. $3,355. c. $-0-.b. ($3,430). d. $621. G & N 9e

53. The following data pertain to an investment proposal:Cost of the investment $20,000Annual cost savings $ 5,000Estimated salvage value $ 1,000Life of the project 8 yearsDiscount rate 16%

The net present value of the proposed investment is: (M)a. $1,720. c. $2,154.b. $6,064. d. $2,025. G & N 9e

55. Sam Weller is thinking of investing $70,000 to start a bookstore. Sam plans to withdraw $15,000 from the business at the end of each year for the next five years. At the end of the fifth year, Sam plans to sell the business for $110,000 cash. At a 12% discount rate, what is the net present value of the investment? (M)a. $54,075. c. $46,445.b. $62,370. d. $70,000. G & N 9e

Even Cash Flow, Working Capital, No Present Value Table46. Parks Company is considering an investment proposal in which a working capital investment

of $10,000 would be required. The investment would provide cash inflows of $2,000 per year for six years. The working capital would be released for use elsewhere when the project is completed. If the company's discount rate is 10%, the investment's net present value is: (M)a. $1,290. c. $2,000.b. ($1,290). d. $4,350. G & N 9e

CMA EXAMINATION QUESTIONS Page 24 of 155

Page 25: P02 - Capital Budgeting

MANAGEMENT ADVISORY SERVICES CAPITAL BUDGETING

48. Boston Company is contemplating the purchase of a new machine on which the following information has been gathered:

Cost of the machine $38,900Annual cash inflows expected $10,000Salvage value $ 5,000Life of the machine 6 years

The company's discount rate is 16%, and the machine will be depreciated using the straight-line method. Given these data, the machine has a net present value of: (M)a. -$26,100. c. $0.b. -$23,900. d. +$26,100. G & N 9e

Even Cash Flow, Salvage Value, Working Capital, No Present Value Table11. A project requires an initial investment in equipment of $90,000 and then requires an

investment in working capital of $10,000 at the beginning (t = 0). The project is expected to produce sales revenues of $120,000 for three years. Manufacturing costs are estimated to be 60% of the revenues. The assets are depreciated using straight-line depreciation. At the end of the project, the firm can sell the equipment for $10,000. The corporate tax rate is 30% and the cost of capital is 15%. Calculate the NPV of the project. (M)A. 8443 C. –2735B. 3840 D. None of the above B & M

44. The following data pertain to an investment in equipment:Investment in the project $10,000Net annual cash inflows 2,400Working capital required 5,000Salvage value of the equipment 1,000Life of the project 8 years

At the completion of the project, the working capital will be released for use elsewhere. Compute the net present value of the project, using a discount rate of 10%: (M)a. $606. c. ($1,729).b. $8,271. d. $1,729. G & N 9e

47. The following data pertain to an investment proposal:Investment in the project (equipment) $14,000Net annual cash inflows promised 2,800Working capital required 5,000Salvage value of the equipment 1,000Life of the project 10 years

The working capital would be released for use elsewhere when the project is completed. What is the net present value of the project, using a discount rate of 8%? (M)a. $2,566. c. $251.

b. ($251). d. $5,251. G & N 9e

Questions 87 & 88 are based on the following information. G & N 9eThe Sawyer Company has $80,000 to invest and is considering two different projects, X and Y. The following data are available on the projects:

Project X Project YCost of equipment needed now ... $80,000 --Working capital requirement .... -- $80,000Annual cash operating inflows .. $23,000 $18,000Salvage value in 5 years ....... $ 6,000 --

Both projects will have a useful life of 5 years; at the end of 5 years, the working capital will be released for use elsewhere. Sawyer's discount rate is 12%.

87. The net present value of project X is: (M)a. $2,915. c. $5,283.b. $(11,708). d. $6,317.

88. The net present value of project Y is closest to: (M)a. $15,110. c. $11,708.b. $30,250. d. $(11,708).

Questions 96 thru 98 are based on the following information. G & N 9eLambert Manufacturing has $60,000 to invest in either Project A or Project B. The following data are available on these projects:

Project A Project B

CMA EXAMINATION QUESTIONS Page 25 of 155

Page 26: P02 - Capital Budgeting

MANAGEMENT ADVISORY SERVICES CAPITAL BUDGETING

Cost of equipment needed now $120,000 $70,000Working capital investment needed now - $50,000Annual net operating cash inflows $ 50,000 $45,000Salvage value of equipment in 6 years $ 15,000 -

Both projects have a useful life of 6 years. At the end of 6 years, the working capital investment will be released for use elsewhere. Lambert's discount rate is 14%.

96. The net present value of Project A is closest to: (M)a. $82,241. c. $74,450.b. $67,610. d. $81,290.

97. The net present value of Project B is closest to: (M)a. $77,805. c. $55,005.b. $127,805. d. $105,005.

98. Which of the following statements is (are) correct? I. Project A is acceptable according to the net present value method. II. Project A has an internal rate of return greater than 14%. (M)a. Only I. c. Both I and II.b. Only II. d. Neither I nor II.

Even Cash Flow, Salvage Value for Old & New Machine, Working Capital, No PV TableQuestions 94 & 95 are based on the following information. G & N 9eWestland College has a telephone system that is in poor condition. The system either can be overhauled or replaced with a new system. The following data have been gathered concerning these two alternatives:

Present System Proposed New SystemPurchase cost new $250,000 $300,000Accumulated depreciation $240,000 -Overhaul costs needed now $230,000 -Annual cash operating costs $180,000 $170,000Salvage value now $160,000 -Salvage value at the end of 8 years $152,000 $165,000Working capital required - $200,000

Westland College uses a 10% discount rate and the total cost approach to capital budgeting analysis. Both alternatives are expected to have a useful life of eight years. (Use three decimal places for PV factor)

94. The net present value of the alternative of overhauling the present system is: (D)a. $(1,279,316). c. $801,284.b. $(1,119,316). d. $(1,194,036).

95. The net present value of the alternative of purchasing the new system is: (D)a. $(1,076,495). c. $(1,169,895).b. $(1,236,495). d. $(969,895).

Uneven Cash Flows, No Present Value Table.12. What is the net present value of the following cash flow at a discount rate of 15%?

T=0 t=1 t=2-120,000 -100,000 300,000

A. $19,887 C. $26,300B. $80,000 D. None of the above B & M

10. What is the net present value of the following cash flows at a discount rate on 12%t = 0 t = 1 t=2 t=3

-250,000 100,000 150,000 200,000A. $101,221 C. $142,208B. $200,000 D. None of the above B & M

13. Given the following cash flow for project A: C0 = -2000, C1 = +500, C2 = +1500 and C3 = +5000, calculate the NPV of the project using a 15% discount rate. (E)A. $5000 C. $3201B. $2857 D. $2352 B & M

65. The Seattle Corporation has been presented with an investment opportunity which will yield end-of-year cash flows of $30,000 per year in Years 1 through 4, $35,000 per year in Years 5 through 9, and $40,000 in Year 10. This investment will cost the firm $150,000 today, and the firm’s cost of capital is 10 percent. What is the NPV for this investment? (M)a. $135,984 d. $ 51,138b. $ 18,023 e. $ 92,146c. $219,045 Brigham

66. You are considering the purchase of an investment that would pay you $5,000 per year for Years 1-5, $3,000 per year for Years 6-8, and $2,000 per year for Years 9 and 10. If you require a 14 percent rate of return, and the cash flows occur at the end of each year, then how much should you be willing to pay for this investment? (M)a. $15,819.27 d. $38,000.00b. $21,937.26 e. $52,815.71c. $32,415.85 Brigham

CMA EXAMINATION QUESTIONS Page 26 of 155

Page 27: P02 - Capital Budgeting

MANAGEMENT ADVISORY SERVICES CAPITAL BUDGETING

8. USSA company has an opportunity to invest in a gold mine. The initial investment is $150 million. The mine is estimated to produce 80,000 ounces of gold per year for the next ten years. The extraction cost of gold per ounce is $100 and the expected revenue is at that level. The current price of gold is $300 per ounce and it is expected to increase by 4% per year for the next 10 years. What is the NPV of the project at a discount rate of 10%? (Ignore taxes.) (D)A. $-34 million. C. $-27.5 millionB. $40.8 million D. None of the above B & M

67. Grant Company is considering an investment of $30,000. Data related to the investment are as follows:

Year Cash Inflows1 $10,0002 12,0003 15,0004 20,0005 10,000

Cost of capital is 18 percent.What is the net present value of the investment, assuming no taxes are paid?a. $7,000 c. $40,911b. $10,911 d. $37,000 H & M

68. A capital investment project requires an investment of $50,000 and has an expected life of 4 years. Annual cash flows at the end of each year are expected to be as follows:

Year Amount1 $20,0002 24,0003 38,0004 28,000

Ignoring income taxes, the net present value of the project using a 6% discount rate is a. $44,316 c. $34,148b. $12,396 d. $(14,148) H & M

69. The Zeron Corporation wants to purchase a new machine for its factory operations at a cost of $950,000. The investment is expected to generate $350,000 in annual cash flows for a period of four years. The required rate of return is 14%. The old machine can be sold for $50,000. The machine is expected to have zero value at the end of the four-year period. What is the net present value of the investment? Would the company want to purchase the new machine? Income taxes are not considered. (M)a. $119,550; yes c. $1,019,550; yesb. $69,550; no d. $326,750; no Horngren

70. Hawkeye Cleaners has been considering the purchase of an industrial dry-cleaning machine. The existing machine is operable for three more years and will have a zero disposal price. If the machine is disposed of now, it may be sold for $60,000. The new machine will cost $200,000 and an additional cash investment in working capital of $60,000 will be required. The new machine will reduce the average amount of time required to wash clothing and will decrease labor costs. The investment is expected to net $50,000 in additional cash inflows during the year of acquisition and $150,000 each additional year of use. The new machine has a three-year life, and zero disposal value. These cash flows will generally occur throughout the year and are recognized at the end of each year. Income taxes are not considered in this problem. The working capital investment will not be recovered at the end of the asset's life.What is the net present value of the investment, assuming the required rate of return is 10%? Would the company want to purchase the new machine? (M)a. $82,000; yes c. $(50,000); yesb. $50,000; no d. $(82,000); no Horngren

71. Hawkeye Cleaners has been considering the purchase of an industrial dry-cleaning machine. The existing machine is operable for three more years and will have a zero disposal price. If the machine is disposed of now, it may be sold for $60,000. The new machine will cost $200,000 and an additional cash investment in working capital of $60,000 will be required. The new machine will reduce the average amount of time required to wash clothing and will decrease labor costs. The investment is expected to net $50,000 in additional cash inflows during the year of acquisition and $150,000 each additional year of use. The new machine has a three-year life. These cash flows will generally occur throughout the year and are recognized at the end of each year. Income taxes are not considered in this problem. The working capital investment will not be recovered at the end of the asset's life.What is the net present value of the investment, assuming the required rate of return is 24%? Would the company want to purchase the new machine? (M)a. $(32,800); yes c. $16,400; yesb. $(16,400); no d. $32,800; no Horngren

72. Wet and Wild Water Company drills small commercial water wells. The company is in the process of analyzing the purchase of a new drill. Information on the proposal is provided below.

Initial investment: Asset $160,000 Working capital $ 32,000Operations (per year for four years): Cash receipts $160,000 Cash expenditures $ 88,000Disinvestment:

CMA EXAMINATION QUESTIONS Page 27 of 155

Page 28: P02 - Capital Budgeting

MANAGEMENT ADVISORY SERVICES CAPITAL BUDGETING

Salvage value of drill (existing) $ 16,000Discount rate 20%

What is the net present value of the investment? Assume there is no recovery of working capital. (M)a. $(62,140) c. $42,362b. $10,336 d. $186,336 Horngren

Uneven Cash Flow, With Present Value Table*. You have been consulted to advise CPA Corp. on the projected acquisition of another

production line costing P1 million. The line has an expected useful life of 5 years without any salvage value. The company’s hurdle rate is 20% and the following information were made available to you.

Year Estimated Annual Cash Inflow Present Value of P1 at 20%1 P 600,000 0.912 300,000 0.763 200,000 0.634 200,000 0.535 200,000 0.44

P 1,500,000 3.27Assuming that the cash flow is generated evenly during the year, your advice is (E)a. To invest due to net present value of P94,000.b. To invest due to net present value of P541,280.c. To invest due to net present value of P635,000.d. To invest due to net advantage of P500,000. RPCPA 1096

*. ABC Corporation is planning to buy production machinery costing P100,000. This machinery’s expected useful life is 5 years, with no residual value. ABC requires a rate of return of 20%, and has calculated the following data pertaining to the purchase and operation of this machinery:

Year Estimated Annual Cash Inflow Present Value of P1 at 20%1 P 60,000 0.912 30,000 0.763 20,000 0.634 20,000 0.535 20,000 0.44

P 150,000 3.27What is the net present value of this investment? (E)a. P80,000. c. P109,400b. P9,400. d. P54,128 RPCPA 1001

13. Kanlaon University has a telephone system that is in poor condition. The system either can be overhauled or replaced with a new system. The following data have been gathered concerning these two alternatives:

Present System Proposed New SystemPurchase cost - new P250,000 P300,000Accumulated depreciation 240,000 -0-Overhaul costs needed now 230,000 -0-Annual cash operating costs 180,000 170,000Salvage value now 160,000 -0-Salvage value at the end of 8 years 152,000 165,000Working capital required -0- 200,000

Kanlaon University uses a 10% discount rate and the total cost approach to capital budgeting analysis. Both alternatives are expected to have a useful life of eight years. The present value at 10% of annuity of 1 for 8 periods is 5.33493 and of 1 at the end of 8 periods is 0.46651. In deciding based on the less cost to the university, the net present value of the alternative of purchasing the new system is (Note: Use only three decimal places for PV factors).A. P1,076,662 C. P1,236,662B. P1,392,662 D. P1,084,662 Pol Bobadilla

NPV Given After Tax Cash FlowEven Cash Flows, No Present Value Table73. Jerkins Company is considering an investment project which will generate a level after-tax

cash flow of $500,000 a year in the next 5 years. Returns on comparable risk investment opportunities are 14%. The investment requires a cash outlay of $1.5 million. What is the net present value of this project? (E)A. ($43,150) C. $216,500B. $108,250 D. $444,350 Gleim

74. R. D. Inc. purchased a machine for $240,000. The machine has a useful life of six years, no salvage value, and straight-line depreciation is to be used. The machine is expected to generate cash flows from operations, net of income tax, of $70,000 in each of the six years. R. D. Inc's cost of capital is 12%. The net present value is:A. $180,000 D. $121,680B. $35,490 E. $123,330C. $47,770 C & U

Even Cash Flows, With WACC Computation, No Present Value Table10. The MM Corp. is planning construction of a new warehouse for its single manufacturing plant.

The initial cost of the investment is $1 million. Efficiencies from the new facility are expected to reduce after-tax costs by $100,000 for each of the next 15 years. The corporation has a

CMA EXAMINATION QUESTIONS Page 28 of 155

Page 29: P02 - Capital Budgeting

MANAGEMENT ADVISORY SERVICES CAPITAL BUDGETING

total value of $60 million and has outstanding debt of $40 million. What is the NPV of the project if the firm has an after-tax cost of debt of 3% and a cost equity of 9%? (M)A. $37,970 C. $69,901B. $60,401 D. None of the above B & M

34. A firm is proposing to undertake a scale expansion. It would cost $40 million and produce an expected cash flow of $8 million a year in perpetuity before tax. The tax rate is 35%. The firm is financed 40% by debt. The expected return on the firm's equity is 20%, and the interest rate on its debt is 12%. What is the NPV of the project using the weighted average cost of capital? A. -$21.48 million C. -$9.05 millionB. $12.91 million D. -$5.61 million B & M

NPV Given Before Tax Cash FlowEven Cash Flow, No Present Value Table75. Jackson Corporation uses net present value techniques in evaluating its capital investment

projects. The company is considering a new equipment acquisition that will cost $100,000, fully installed, and have a zero salvage value at the end of its five-year productive life. Jackson will depreciate the equipment on a straight-line basis for both financial and tax purposes. Jackson estimates $70,000 in annual recurring operating cash income and $20,000 in annual recurring operating cash expenses. Jackson’s cost of capital is 12% and its effective income tax rate is 40%. What is the net present value of this investment on an after-tax basis?(E)a. $28,840 c. $36,990b. $8,150 d. $80,250 CMA Samp Q4-4

76. Drillers Inc. is evaluating a project to produce a high-tech deep-sea oil exploration device. The investment required is $80 million for a plant with a capacity of 15,000 units a year for 5 years. The device will be sold for a price of $12,000 per unit. Sales are expected to be 12,000 units per year. The variable cost is $7,000 and fixed costs, excluding depreciation, are $25 million per year. Assume Drillers employs straight-line depreciation on all depreciable assets, and assume that they are taxed at a rate of 36%. If the required rate of return is 12%, what is the approximate NPV of the project? (M)A. $17,225,000 C. $26,780,000B. $21,511,000 D. $56,117,000 Gleim

77. Sparrow Corporation is considering an investment in equipment for $30,000. Sparrow uses the straight-line method of depreciation with no mid-year convention. In addition, their tax rate is 40 percent, and the life of the equipment is five years with no salvage value. The expected income before depreciation and taxes is projected to be $15,000 per year. The cost of capital is 18 percent.What is the net present value of the investment?

a. $5,648 c. $35,648b. $(1,857) d. $28,143 H & M

Even Cash Flow, With Present Value Table*. On January 1, Studley Company purchased a new machine for $100,000 to be depreciated

over 5 years. It will have no salvage value at the end of 5 years. For book and tax purposes, depreciation will be $20,000 per year. The machine is expected to produce annual cash flow from operations, before income taxes, of $40,000. Assume that Studley uses a discount rate of 12% and that its income tax rate will be 40% for all years. The present value of $1 at 12% for 5 periods is 0.57, and the present value of an ordinary annuity of $1 at 12% for five periods is 3.61. The NPV of the machine should be (E)a. $15,520 positive. c. $60,000 positive.b. $15,520 negative. d. $25,000 negative. AICPA 0582 I-37

*. The General Manager of Tela Mills Inc. is considering the purchase of some new machines. The machines would cost P4,000,000 with an economic life of 8 years without any salvage value. Once set up, they would generate P12,500,000 additional revenues but yearly expenses for additional labor and materials would also increase by P11,500,000. Assume the company uses straight-line depreciation for taxes and that the appropriate tax rate is 35%. The required after-tax rate of return is 14%.The following data are for an interest rate of 15% and 8 periods.

Present value of P1 0.3506Future value of P1 2.8526Present value of annuity of P1 4.6389Future value of annuity of P1 13.2328

The company should (M)a. Purchase the machines due to positive NPV of P638,900.b. Not purchase the machines due to negative NPV of P984,715.c. Not purchase the machines due to negative NPV of P172,907.50. RPCPA 0595d. Be indifferent as to the option does not bring about any advantage nor disadvantage.

Even Cash Flow, Uneven Depreciation, Salvage Value, With Present Value Table*. JJ Corp. is considering the purchase of a new machine that will cost P320,000. It has an

estimated useful life of 3 years. Assume that 30% of the depreciable base will be depreciated in the first year, 40% in the second year, and 30% in the third year. It has a resale value of P20,000 at the end of its economic life. Savings are expected from the use of machine estimated at P170,000 annually. The company has an effective tax rate of 40%. It uses 16% as hurdle rate in evaluating capital projects. Should the company proceed with the P320,000 capital investment? (D)

Year Present Value of P1 Present Value of an Ordinary Annuity of P11 0.862 0.862

CMA EXAMINATION QUESTIONS Page 29 of 155

Page 30: P02 - Capital Budgeting

MANAGEMENT ADVISORY SERVICES CAPITAL BUDGETING

2 0.743 1.6053 0.641 2.246

a. Yes, due to NPV of P6,556. c. Yes, due to NPV of P61,820. RPCPA 0596b. Yes, due to NPV of P11,684. d. No, due to negative NPV of P1,136

Even Cash Flow, SL & SYD Depreciation Method, With Present Value TableNumbers 14 and 15 are based on the following information: RPCPA 0589MENUDO CORP. is evaluating an investment of P480,000 in equipment with a useful life of 3 years and no salvage value. An estimate indicates that the cash flow before income tax from the investment will amount to P240,000 a year and is expected to yield a discounted rate of return of 12%. Normally, straight-line method of depreciation is used, but the President was informed that investment would be more favorable if the sum-of-the-years digit method of depreciation is used. You were using the two methods of depreciation for purposes of comparison. Income tax rate is 35%.The present value of P1.00 at 12% for

1 year 0.8932 years 0.7973 years 0.712

The present value of an annuity for 3 years at 12% is 2.402.

*. The net present value of the investment using the straight-line method of depreciation isa. P5,068 c. P38,032b. P36,480 d. P29,224

*. The net present value of the investment using the sum-of-years digit method isa. P34,292 c. P36,480b. P38,032 d. P43,116

Uneven Cash Flow13. Goldsmith labs recover gold from printed circuit boards. It has developed a new equipment for

the purpose. The following data is given. 1) Equipment costs $250,0002) It will cost 100,000 per year to run3) It has an economic life of 5 years and is depreciated using straight-line method4) It will recover 1000 ounces of gold per year5) The current price of gold is $300 per ounce and it expected to increase at a rate 4% per

year for the foreseeable future6) The tax rate is 30%7) The cost of capital is 8%What is NPV of the equipment? A. $580,400 C. $470,400

B. $520,510 D. None of the above B & M

With Pessimistic, Expected and Optimistic Outcome78. The following forecasts have been prepared for a new investment by Oxford Industries of $20

million with an 8-year life: Pessimistic Expected OptimisticMarket size 60,000 90,000 140,000Market share, % 25 30 35Unit price $750 $800 $875Unit variable cost $500 $400 $350Fixed cost, millions $7 $4 $3.5

Assume that Oxford employs straight-line depreciation, and that they are taxed at 35%. Assuming an opportunity cost of capital of 14%, what is the NPV of this project, based on expected outcomes? A. $2,626,415 C. $6,722,109B. $4,563,505 D. $8,055,722 Gleim

7. A project has an initial investment of 100. You have come up with the following estimates of the projects with cash flows.

NPV Pessimistic Most Likely OptimisticRevenues 15 20 25Costs -10 -8 -5

If the cash flows are perpetuities and the cost of capital is 10%. What does a sensitivity analysis of NPV (no taxes) show? B & M

NPV Pessimistic Most Likely OptimisticA. -50 20 +100B. -100 -50 +80C. -50 +50 +70D. None of the above

8. A project has an initial investment of $150. You have come up with the following estimates of revenues and costs.

Pessimistic Most Likely Optimistic

CMA EXAMINATION QUESTIONS Page 30 of 155

Page 31: P02 - Capital Budgeting

MANAGEMENT ADVISORY SERVICES CAPITAL BUDGETING

Total revenues +30 +50 65Total costs -25 -20 -15

If the cash flows are perpetuities and the cost of capital is 10%. What does a sensitivity analysis of NPV (no taxes) show? B & MNPV Pessimistic Most Likely OptimisticA. -100 +150 +350B. -50 +300 +500C. +50 -100 +400D. None of the above

9. You have come up with the following estimates of project cash flows:Pessimistic Most Likely Optimistic

Investments -100 -80 -60Total Revenues +30 +40 +50Total Costs -20 -15 -10

The cash flows are perpetuities and the cost of capital is 10%. What does a sensitivity analysis of NPV (without taxes) show? B & M

NPV Pessimistic Most Likely OptimisticA. 0 +170 +340B. 0 +500 +800C. -90 -55 - 20D. None of the above

Sensitivity AnalysisChange in Revenues & Costs12. A project has the following cash flows: C0 = -100,000; C1 = 50,000; C2 = 150,000; C3 =

100,000. If the discount rate changes from 12% to 15%, what would the NPV of the project if the revenues were higher by 10% and the costs were 65% of the revenues? (D)A. $5566 C. $5611B. $964 D. None of the above B & M

Change in Fixed Costs79. What happens to the NPV of a 1-year project if fixed costs are increased from $500 to $600,

the firm is profitable, has a 35% tax rate and employs a 12% cost of capital? (E)A. NPV decreases by $100.00. C. NPV decreases by $65.00.B. NPV decreases by $89.29. D. NPV decreases by $58.04. Gleim

Change in Discount Rate10. A project has the following cash flows: C0 = -100,000; C1 = 50,000; C2 = 150,000; C3 =

100,000. If the discount rate changes from 12% to 15%, what is the change in the NPV of the project (approximately)? (M)A. 12,750 decrease C. 122,650 increaseB. 12,750 increase D. 135,400 decrease B & M

13. A project has the following cash flows: C0 = -100,000; C1 = 50,000; C2 = 150,000; C3 = 100,000. If the discount rate changes from 12% to 15%, what would be the NPV of the project if the discount rate were higher by 10%? A. $5648 C. –$2735B. $3840 D. None of the above B & M

LeasesPresent Value80. For the next 2 years, a lease is estimated to have an operating net cash inflow of $7,500 per

annum, before adjusting for $5,000 per annum tax basis lease amortization, and a 40% tax rate. The present value of an ordinary annuity of $1 per year at 10% for 2 years is 1.74. What is the lease’s after-tax present value using a 10% discount factor? (E)a. $2,610 c. $9,570b. $4,350 d. $11,310 Gleim, RPCPA 1001

Discount Rate81. Stone Co. is considering the acquisition of equipment. To buy the equipment, the cost is

$15,192. To lease the equipment, Stone must sign a non-cancelable lease and make five payments of $4,000 each. The first payment will be paid on the first day of the lease. At the time of the last payment, Stone will receive title to the equipment. The present value of an ordinary annuity of $1 is as follows:

Present ValueNo. of Periods 10% 12% 16%

1 0.909 0.893 0.8622 1.736 1.690 1.6053 2.487 2.402 2.2464 3.170 3.037 2.7985 3.791 3.605 3.274

The interest rate implicit in this lease is approximatelya. 10% c. Between 10% and 12%b. 12% d. 16% AICPA 1178 I-39

PROFITABILITY INDEXGiven Net Present Value

CMA EXAMINATION QUESTIONS Page 31 of 155

Page 32: P02 - Capital Budgeting

MANAGEMENT ADVISORY SERVICES CAPITAL BUDGETING

*. Friendly Corp.’s Project Sky has a net investment of P1.2 million. The net present value of all future net cash inflows is P2.4 million. The company’s tax rate is 40%. The profitability index is (E)a. 0.50 c. 0.83b. 1.20 d. 2.00 RPCPA 0597

Given Internal Rate of Return with Present Value Table41. Sulu Company is considering to acquire a machine in order to reduce its direct labor costs.

This machine shall last for 4 years with no salvage value. Currently, the cash expenses amounted to P120,000 per year. The initial analysis indicated that the time-adjusted rate of return is 15%. At 12% (cost of capital to finance the purchase of the machine), the company expects net present value of P5,470.80.The present value of 1 for four period at 12% is 3.03735 and at 15% is 2.85499. Ignoring income tax considerations, the profitability index is (D)A. 1.064 C. 1.047B. 1.183 D. 1.250 Pol Bobadilla

Even Cash Flows, Ignore Income Taxes, No Present Value Table82. Mesa Company is considering an investment to open a new banana processing division. The

project in question would entail an initial investment of $45,000, and as a result of the project cash inflows of $20,000 can be expected in each of the next 3 years. The hurdle rate is 10%. What is the profitability index for the project? (E)A. 1.0784 C. 1.1379B. 1.1053 D. 1.1771 Gleim

45. An investment opportunity costing $150,000 is expected to yield net cash flows of $50,000 annually for five years. The profitability index of the investment at a cutoff rate of 14% would be (E)a. 3.0 c. 0.33b. 1.14 d. 14% D, L & H 9e

45. An investment opportunity costing $300,000 is expected to yield net cash flows of $100,000 annually for five years. The profitability index of the investment at a cutoff rate of 14% would bea. 3.0. c. 0.33.b. 1.14. d. 14%. L & H 10e

46. A project has a NPV of $30,000 when the cutoff rate is 10%. The annual cash flows are $41,010 on an investment of $100,000. The profitability index for this project isa. 1.367 c. 2.438b. 3.3333 d. 1.300 D, L & H 9e

68. A project has an initial cost of $100,000 and generates a present value of net cash inflows of $120,000. What is the project's profitability index? (E)a. .20 c. .80b. 1.20 d. 5.00 Barfield

Given Present Value of After Tax Cash FlowsQuestions 33 and 34 are based on the following information. RPCPA 0593Pertinent data for two (2) alternatives which are being considered by the Blue Nun Co. are:

1 2Present values of annual cash flows P546,480 P306,495Present values of the net investment 500,000 312,500Excess (deficiency) present value P 46,480 (P 6,005)

*. The profitability index is a measure of the desirability of investments which are not of the same size. Given the above data, the profitability indices for alternatives 1 and 2 are

a. b. c. d.Alternative 1 1.09 1.21 1.09 0.98Alternative 2 0.69 1.15 0.98 1.11

*. In evaluating the alternatives for the above Blue Nun Co., the following statements are true except:a. Alternative 1 is better because there is an excess present value of the cash flow.b. The profitability index of Alternative 1 means that it will earn less than the lowest

acceptable rate of return.c. Alternative 2 is not acceptable because its profitability index is less than 1.d. The deficiency present value for Alternative 2 means it does not meet the minimum rate of

return requirement.

INTERNAL RATE OF RETURNIRR Formula*. Two projects have an initial outlay of $497 and each has an income stream lasting 3 years.

Project A returns $200 per year for 3 years. Project B returns $200 for the first 2 years and $248 for the third year.

n 8% 10% 12% 14%1 .9259 .9091 .8929 .87722 .8573 .8264 .7972 .76953 .7938 .7513 .7118 .6750

The appropriate internal rate of return valuation for Project B isa. $200(.8772) + $200(.7695) + $248(.6750) = $496.74b. $200(.8929) + $200(.7972) + $248(.7118) = $514.41

CMA EXAMINATION QUESTIONS Page 32 of 155

Page 33: P02 - Capital Budgeting

MANAGEMENT ADVISORY SERVICES CAPITAL BUDGETING

c. $200(.9091) + $200(.8264) + $248(.7513) = $533.42d. $200(.9259) + $200(.8573) + $248(.7938) = $553.50 CIA 1185 IV-31

Required InvestmentGiven Present Value of Cash Inflow, Ignore Income Taxes38. (Ignore income taxes in this problem.) A planned factory expansion project has an estimated

initial cost of $800,000. Using a discount rate of 20%, the present value of future cost savings from the expansion is $843,000. To yield exactly a 20% internal rate of return, the actual investment cost cannot exceed the $800,000 estimate by more than: (m)a. $160,000. c. $43,000.b. $20,000. d. $1,075. AICPA adapted

Even Cash Flow, Ignore Income Taxes No Present Value Table91. Ann recently invested in a project that promised an internal rate of return of 15 percent. If the

project has an expected annual cash inflow of $12,000 for six years, with no salvage value, how much did Ann pay for the project?a. $35,000 c. $72,000b. $45,414 d. $31,708 Barfield

83. A firm is considering a project with annual cash flows of $100,000. The project would have a 7-year life, and the company uses a discount rate of 10%. Ignoring income taxes, what is the maximum amount the company could invest in the project and the project still be acceptable?a. $359,100 c. $486,800b. $700,000 d. $100,000 H & M

84. A firm is considering a project with annual cash flows of $40,000. The project would have a 10-year life, and the company uses a discount rate of 8%. Ignoring income taxes, what is the maximum amount the company could invest in the project and the project still be acceptable (rounded)?a. $400,000 c. $203,210b. $268,400 d. $363,604 H & M

85. A firm is considering a project with annual cash flows of $120,000. The project would have an 8-year life, and the company uses a discount rate of 12 percent. Ignoring income taxes, what is the maximum amount the company could invest in the project and the project still be acceptable (rounded)?a. $488,740 c. $580,291b. $562,614 d. $596,160 H & M

33. (Ignore income taxes in this problem.) Kipling Company has invested in a project that has an eight-year life. It is expected that the annual cash inflow from the project will be $20,000.

Assuming that the project has a internal rate of return of 12%, how much was the initial investment in the project? (M)a. $160,000 c. $80,800b. $99,360 d. $64,640 AICPA adapted

34. (Ignore income taxes in this problem.) White Company's required rate of return on capital budgeting projects is 12%. The company is considering an investment opportunity which would yield a cash flow of $10,000 in five years. What is the most that the company should be willing to invest in this project? (M)a. $36,050. c. $17,637.b. $2,774. d. $5,670. G & N 9e

Even Cash Flow, Salvage Value, Ignore Income Taxes, No Present Value Table65. Tiger Inc. bought a piece of machinery with the following data:

Useful life 6 yearsYearly net cash inflow $45,000Salvage value - 0 -Internal rate of return 18%Cost of capital 14%

The initial cost of the machinery wasa. $157,392.b. $174,992.c. $165,812.d. impossible to determine from the information given. Barfield

41. (Ignore income taxes in this problem.) The Baker Company purchased a piece of equipment with the following expected results:

Useful life 7 yearsYearly net cash inflow $50,000Salvage value -0-Internal rate of return 20%Discount rate 16%

The initial cost of the equipment was: (M)a. $300,100.b. $180,250c. $190,600.d. Cannot be determined from the information given. G & N 9e

Uneven Even Cash Flow, Ignore Income Taxes, No Present Value Table.56. (Ignore income taxes in this problem.) Arthur operates a part-time auto repair service. He

estimates that a new diagnostic computer system will result in increased cash inflows of

CMA EXAMINATION QUESTIONS Page 33 of 155

Page 34: P02 - Capital Budgeting

MANAGEMENT ADVISORY SERVICES CAPITAL BUDGETING

$2,100 in Year 1, $3,200 in Year 2, and $4,000 in Year 3. If Arthur's discount rate is 10%, then the most he would be willing to pay for the new computer system would be: (M)a. $6,652. c. $7,747.b. $6,984. d. $7,556. G & N 9e

35. (Ignore income taxes in this problem.) In order to receive $12,000 at the end of three years and $10,000 at the end of five years, how much must be invested now if you can earn 14% rate of return? (M)a. $12,978. c. $13,290.b. $8,100. d. $32,054. G & N 9e

Even After-tax Annual Cash Flow, No Present Value Table86. Conte Inc. invested in a machine with a useful life of six years and no salvage value. The

machine is expected to produce annual cash flows from operations, net of income tax, of $2,000. If the estimated internal rate of return is 10%, the amount of the original investment was: (M)A. $9,000 D. $5,640B. $11,280 E. $8,710C. $12,000 C & U

Even After-tax Annual Cash Flow, Salvage Value No Present Value Table87. The Hopkins Company has estimated that a proposed project's 10-year annual net cash

benefit, received each year end, will be $2,500 with an additional terminal benefit of $5,000 at the end of the 10th year. Assuming that these cash inflows satisfy exactly Hopkins' required rate of return of 8%, calculate the initial cash outlay. (E)A. $16,775 C. $25,000B. $19,090 D. $30,000 Gleim

Even After-tax Cash Flow, with Present Value Table. Cause Company is planning to invest in a machine with a useful life of five years and no

salvage value. The machine is expected to produce cash flow from operations, net of income taxes, of P20,000 in each of the five years. Cause’s expected rate of return is 10%. Information on present value and future amount factors is as follows:

Period1 2 3 4 5

PV of P1 at 10% .900 .826 .751 .683 .621

PVA of P1 at 10% .909 1.736 2.487 3.170 1.611FV of P1 at 10% 1.000 1.210 1.330 1.464 1.611FVA of P1 at 10% 1.000 2.100 3.310 4.641 6.105

How much will the machine cost?A. P32,220 C. P75,820B. P62,100 D. P122,100 Pol Bobadilla

Uneven After-tax Cash Flow, Salvage Value, with Present Value Table.88. Kern Co. is planning to invest in a 2-year project that is expected to yield cash flows from

operations, net of income taxes, of $50,000 in the first year and $80,000 in the second year. Kern requires an internal rate of return of 15%. The present value of $1 for one period at 15% is 0.870 and for two periods at 15% is 0.756. The future value of $1 for one period at 15% is 1.150 and for two periods at 15% is 1.323. The maximum that Kern should invest immediately is (E)a. $81,670 c. $130,000b. $103,980 d. $163,340 AICPA 1189 II-36

89. Orab Co. has the chance to invest in a 2-year project expected to produce cash flows from operations, net of income taxes, of $100,000 in the first year and $200,000 in the second year. Orab requires an internal rate of return of 20%. The present value of $1 for one period at 20% is 0.833; for two periods at 20% is 0.694. For this project, Orab should be willing to invest immediately a maximum of: (E)A. $222,100 C. $283,300B. $208,200 D. $249,900 AICPA adapted

Required LifeEven Cash Flow, Ignore Income Taxes, No Present Value Table37. (Ignore income taxes in this problem.) The following information is available on a new piece of

equipment:Cost of the equipment $21,720Annual cash inflows $5,000Internal rate of return 16%Required rate of return 10%

The life of the equipment is approximately: (M)a. 6 years.b. 4.3 years.c. 8 years.d. it is impossible to determine from the data given. G & N 9e

CMA EXAMINATION QUESTIONS Page 34 of 155

Page 35: P02 - Capital Budgeting

MANAGEMENT ADVISORY SERVICES CAPITAL BUDGETING

Required Increase in Cash Flows*. The following data pertain to Sunlight Corp., whose management is planning to purchase an

automated tanning equipment.1. Economic life of equipment – 8 years.2. Disposal value after 8 years – nil.3. Estimated net annual cash inflows for each of the 8 years – P81,000.4. Time-adjusted internal rate of return – 14%5. Cost of capital of Sunlight Corp – 16%6. The table of present values of P1 received annually for 8 years has these factors: at

14% = 4.639, at 16% = 4.3447. Depreciation is approximately P46,970 annually.

Find the required increase in annual cash inflows in order to have the time-adjusted rate of return approximately equal the cost of capital. (M)a. P5,501 c. P4,344b. P6,501 d. P5,871 RPCPA 0594

Required Annual Before Tax Cash FlowEven Cash Flow90. Payback Company is considering the purchase of a copier machine for P42,825. The copier

machine will be expected to be economically productive for 4 years. The salvage value at the end of 4 years is negligible. The machine is expected to provide 15% internal rate of return. The company is subject to 40% income tax rate. The present value of an ordinary annuity of 1 for 4 periods is 2.85498. In order to realize the IRR of 15%, how much is the estimated before-tax cash inflow to be provided by the machine?(M)A. P17,860 C. P25,000B. P15,000 D. P35,700

91. Scott Corporation’s new project calls for an investment of $10,000. It has an estimated life of 10 years. The IRR has been calculated to be 15 percent. If cash flows are evenly distributed and the tax rate is 40 percent, what is the annual before-tax cash flow each year? (Assume depreciation is a negligible amount.) (M)a. $1,993 d. $4,983b. $3,321 e. $5,019c. $1,500 Brigham

Required Annual After-tax Cash FlowEven Cash Flow, Salvage Value92. Para Co. is reviewing the following data relating to an energy saving investment proposal:

Cost $50,000Residual value at the end of 5 years 10,000Present value of an annuity of 1 at 12% for 5 years 3.60

Present value of 1 due in 5 years at 12% 0.57What would be the annual savings needed to make the investment realize a 12% yield?(M)a. $8,189 c. $12,306b. $11,111 d. $13,889

Required Cash Flow for a Certain YearEven Cash Flow, Ignore Income Taxes, No Present Value Table39. (Ignore income taxes in this problem.) Hilltop Company invested $100,000 in a two-year

project. The cash flow was $40,000 for the first year. Assuming that the internal rate of return was exactly 12%, what was the cash flow for the second year of the project? (M)a. $51,247. c. $64,284.b. $60,000. d. $80,652. AICPA adapted

Required Investment & Annual Cash FlowQuestions 85 & 86 are based on the following information. G & N 9e(Ignore income taxes in this problem.) The Finney Company is reviewing the possibility of remodeling one of its showrooms and buying some new equipment to improve sales operations. The remodeling would cost $120,000 now and the useful life of the project is 10 years. Additional working capital needed immediately for this project would be $30,000; the working capital would be released for use elsewhere at the end of the 10-year period. The equipment and other materials used in the project would have a salvage value of $10,000 in 10 years. Finney's discount rate is 16%.

85. The immediate cash outflow required for this project would be: (E)a. $(120,000). c. $(90,000).b. $(150,000). d. $(130,000).

86. What would the annual net cash inflows from this project have to be in order to justify investing in remodeling? (M)a. $14,495 c. $16,147b. $35,842 d. $29,158

Required Break-even SalesPeso Sales14. Calculator Company proposes to invest $6 million in a new calculator making plant. Fixed

costs are $1 million a year. A calculator costs $4 million to manufacture and can be sold for $19. If the plant lasts for 3 years and the cost of capital is10%, what is the approximate break-even level of annual sales? (Assume no taxes.) (M)A. $227,550 C. $67,000B. $160,900 D. None of the above B & M

CMA EXAMINATION QUESTIONS Page 35 of 155

Page 36: P02 - Capital Budgeting

MANAGEMENT ADVISORY SERVICES CAPITAL BUDGETING

Unit Sales15. Financial Calculator Company proposes to invest $9 million in a new calculator making plant.

Fixed costs are $2 million a year. A financial calculator costs $8 per unit to manufacture and can be sold for $24 per unit. If the plant lasts for 4 years and the cost of capital is 20%, what is the break-even level of annual rates? (Assume no taxes.) (M)A. 342,500 units C. 125,000 unitsB. 217,500 units D. None of the above B & M

17. Taj Mahal Tour company proposes to invest $3 million in a new tour package. Fixed costs are $1 million per year. The tour package costs $500 and can be sold at $1500 per package to tourists. This tour package is expected to be attractive for the next five years. If the cost of capital is 20%, what is the break-even number of tourists per year? (Ignore taxes, give an approximation.) A. 2000 C. 15000B. 1000 D. None of the above B & M

18. Hammer Company proposes to invest $6 million in a new type of hammer-making equipment. The fixed costs are $0.5 million per year. The equipment is expected to last for 5 years. The manufacturing costs per hammer is $1. Calculate the break-even volume per year. (Ignore taxes.) A. 400,000 C. 250,000B. 500,000 D. None of the above B & M

Required Unit Sales & Selling PriceQuestions 55 and 56 are based on the following information. Pol BobadillaMoorman Products Company is considering a new product that will sell for P100 and have a variable cost of P60. Expected volume is 20,000 units. New equipment costing P1,500,000 and having a five-year useful life and no salvage value is needed, and will be depreciated using the straight-line method. The machine has cash operating costs of P20,000 per year. The firm is in the 40% tax bracket and has cost of capital of 12%. The present value of 1, end of five periods is 0.56743; present value of annuity of 1 for 5 periods is 3.60478.

55. How many units per year must the firm sell for the investment to earn 12% internal rate of return?A. 12,838 C. 8,225B. 10,403 D. 7,625

56. Suppose the 20,000 estimated volume is sound, but the price is in doubt, What is the selling price (rounded to nearest peso) needed to earn a 12% internal rate of return?A. P81 C. P70B. P85 D. P90

IRR Given Payback Period93. Smoot Automotive has implemented a new project that has an initial cost, and then generates

inflows of $10,000 a year for the next seven (7) years. The project has a payback period of 4.0 years. What is the project's internal rate of return (IRR)? A. 14.79% C. 18.54%B. 16.33% D. 15.61% Gleim

IRR Given Cash Flows, Ignore Income TaxesEven Cash Flows, No Present Value Table94. What is the approximate IRR for a project that costs $50,000 and provides cash inflows of

$20,000 for 3 years? A. 10% C. 22%B. 12% D. 27% Gleim

95. Pena Company is considering a project that calls for an initial cash outlay of $50,000. The expected net cash inflows from the project are $7,791 for each of 10 years. What is the IRR of the project? A. 6% C. 8%B. 7% D. 9% Gleim

96. Which of the following statements is most likely correct for a project costing $50,000 and returning $14,000 per year for 5 years? A. NPV = $36,274. C. IRR = 1.4%.B. NPV = $20,000. D. IRR is greater than 10%. Gleim

45. An investment opportunity costing $150,000 is expected to yield net cash flows of $45,000 annually for five years. The IRR of the investment is betweena. 10 and 12%. c. 14 and 16%.b. 12 and 14%. d. 16 and 18%. D, L & H 9e

45 An investment opportunity costing $180,000 is expected to yield net cash flows of $53,000 annually for five years. The IRR of the investment is betweena. 10 and 12% c. 14 and 16%.b. 12 and 14%. d. 16 and 18%. L & H 10e

49. An investment opportunity costing $400,000 is expected to yield net cash flows of $75,000 annually for eight years. The IRR of the investment is betweena. 10 and 12%. c. 14 and 16%.b. 12 and 14%. d. 16 and 18%. D, L & H 9e

CMA EXAMINATION QUESTIONS Page 36 of 155

Page 37: P02 - Capital Budgeting

MANAGEMENT ADVISORY SERVICES CAPITAL BUDGETING

49. An investment opportunity costing $200,000 is expected to yield net cash flows of $39,000 annually for eight years. The IRR of the investment is betweena. 10 and 12%. c. 14 and 16%.b. 12 and 14%. d. 16 and 18%. L & H 10e

*. MLF Corporation is evaluating the purchase of a P500,000 die attach machine. The cash inflows expected from the investment is P145,000 per year for five years with no equipment salvage value. The cost of capital is 12%. The net present value factor for five (5) years at 12% is 3.6048 and at 14% is 3.4331. The internal rate of return for this investment is (M)a. 3.45% c. 13.8%b. 2.04% d. 15.48% RPCPA 1097

36. (Ignore income taxes in this problem.) Sue Falls is the president of Sports, Inc. She is considering buying a new machine that would cost $14,125. Sue has determined that the new machine promises a internal rate of return of 12%, but Sue has misplaced the paper which tells the annual cost savings promised by the new machine. She does remember that the machine has a projected life of 10 years. Based on these data, the annual cost savings are: (M)a. it is impossible to determine from the data given.b. $1,412.50.c. $2,500.00.d. $1,695.00. G & N 9e

40. (Ignore income taxes in this problem.) Joe Flubup is the president of Flubup, Inc. He is considering buying a new machine that would cost $25,470. Joe has determined that the new machine promises a internal rate of return of 14%, but Joe has misplaced the paper which tells the annual cost savings promised by the new machine. He does remember that the machine has a projected life of 12 years. Based on these data, the annual cost savings are: (M)a. impossible to determine from the data given.b. $2,122.50.c. $4,500.00.d. $4,650.00. G & N 9e

97. A firm is considering a project requiring an investment of $13,500. The project would generate annual cash inflows of $3,148 per year for the next 7 years. The company uses the straight-line method of depreciation with no mid-year convention. Ignore income taxes. The approximate internal rate of return for the project is a. 6% d. 14%b. 8% e. 18%.c. 12% H & M

98. A firm is considering a project requiring an investment of $100,000. The project would generate annual cash inflows of $27,739 per year for the next 5 years. The company uses the straight-line method of depreciation with no mid-year convention. Ignore income taxes. The approximate internal rate of return for the project isa. 9% d. 16%b. 10% e. 28%c. 12% H & M

99. An insurance firm agrees to pay you $3,310 at the end of 20 years if you pay premiums of $100 per year at the end of each year for 20 years. Find the internal rate of return to the nearest whole percentage point. (E)a. 9% d. 3%b. 7% e. 11%c. 5% Brigham

100. Foster Company is considering the purchase of a new machine for $38,000. The machine would generate a net cash inflow of $11,607 per year for five years. At the end of five years, the machine would have no salvage value. The company’s cost of capital is 12 percent. The company uses straight-line depreciation with no mid-year convention.What is the internal rate of return for the machine rounded to the nearest percent, assuming no taxes are paid?a. 12% c. 14%b. 18% d. 16% H & M

101. Brown Corporation recently purchased a new machine for $339,013.20 with a ten-year life. The old equipment has a remaining life of ten years and no disposal value at the time of replacement. Net cash flows will be $60,000 per year. What is the internal rate of return? (E)a. 12% c. 20%b. 16% d. 24% Horngren

102. Soda Manufacturing Company provides vending machines for soft-drink manufacturers. The company has been investigating a new piece of machinery for its production department. The old equipment has a remaining life of three years and the new equipment has a value of $52,650 with a three-year life. The expected additional cash inflows are $25,000 per year. What is the internal rate of return? (E)a. 20% c. 10%b. 16% d. 8% Horngren

103. The Zeron Corporation recently purchased a new machine for its factory operations at a cost of $921,250. The investment is expected to generate $250,000 in annual cash flows for a period of six years. The required rate of return is 14%. The old machine has a remaining life of

CMA EXAMINATION QUESTIONS Page 37 of 155

Page 38: P02 - Capital Budgeting

MANAGEMENT ADVISORY SERVICES CAPITAL BUDGETING

six years. The new machine is expected to have zero value at the end of the six-year period. The disposal value of the old machine at the time of replacement is zero. What is the internal rate of return? (M)a. 15% c. 17%b. 16% d. 18% Horngren

57. (Ignore income taxes in this problem.) The following data pertain to an investment proposal:Present investment required $26,500Annual cost savings $ 5,000Projected life of the investment 10 yearsProjected salvage value $ -0-

The internal rate of return, interpolated to the nearest tenth of a percent, would be: (M)a. 11.6%. c. 13.6%.b. 12.8%. d. 12.4%. G & N 9e

58. (Ignore income taxes in this problem.) The following data are available on a proposed investment project:

Initial investment $142,500Annual cash inflows $30,000Life of the investment 8 yearsRequired rate of return 10%

The internal rate of return, interpolated to the nearest tenth of a percent, would be: (M)a. 13.3%. c. 15.3%.b. 12.1%. d. 12.7%. G & N 9e

59. (Ignore income taxes in this problem.) The following data pertain to an investment proposal:Present investment required $14,000Annual cost savings $ 2,500Projected life of the investment 8 yearsProjected salvage value $ -0-Required rate of return 6%

The internal rate of return, interpolated to the nearest tenth of a percent, would be: (M)a. 6.7%. c. 8.7%.b. 9.3%. d. 7.3%. G & N 9e

60. (Ignore income taxes in this problem.) Overland Company has gathered the following data on a proposed investment project:

Investment in depreciable equipment $150,000Annual cash flows $ 40,000Life of the equipment 10 yearsSalvage value -0-

Discount rate 10%The internal rate of return on this investment is closest to: (D)a. 23.4%. c. 22.7%b. 25.4%. d. 22.1% G & N 9e

61. (Ignore income taxes in this problem.) The following information concerns a proposed investment:

Investment required $14,150Annual savings $ 2,500Life of the project 12 years

The internal rate of return is (do not interpolate): (M)a. 14%. c. 10%.b. 12%. d. 5%. G & N 9e

104. Whitney Crane Inc. has the following independent investment opportunities for the coming year:

Project Cost Annual Cash Inflows Life (Years) IRRA $10,000 $11,800 1B 5,000 3,075 2 15C 12,000 5,696 3D 3,000 1,009 4 13

The IRRs for Projects A and C, respectively, are: (M)a. 16% and 14% d. 18% and 13%b. 18% and 10% e. 16% and 13%c. 18% and 20% Brigham

105. Genuine Products Inc. requires a new machine. Two companies have submitted bids, and you have been assigned the task of choosing one of the machines. Cash flow analysis indicates the following:

Year Machine A Cash Flow Machine B Cash Flow0 -$2,000 -$2,0001 0 8322 0 8323 0 8324 3,877 832

What is the internal rate of return for each machine? (M)a. IRRA = 16%; IRRB = 20% d. IRRA = 18%; IRRB = 24%b. IRRA = 24%; IRRB = 20% e. IRRA = 24%; IRRB = 26%c. IRRA = 18%; IRRB = 16% Brigham

CMA EXAMINATION QUESTIONS Page 38 of 155

Page 39: P02 - Capital Budgeting

MANAGEMENT ADVISORY SERVICES CAPITAL BUDGETING

Even Cash Flow, With Present Value Table*. What is the discounted rate of return, to the nearest percent of an investment of P100,000 that

gives an annual income of P12,000 over a 15-year period? (E)Abridged table of present value of P1.00 received annually for N years

N 6% 8% 10% 12%15 9.712 8.559 7.606 6.811

a. 6% c. 10%b. 8% d. none of these RPCPA 1087

*. Mr. Castillo is thinking to buy a lathe machine for P10,000. This machine will result in annual cash inflow of P2,000 a year for ten-year period. Below is an abridged table showing the present value of annuity of P1.00 for N periods.

Years

14% 15% 16%

9 4.946 4.772 4.60710 5.216 5.019 4.83311 5.453 5.234 5.029

Using the short-cut method, the discounted rate of return on the project is (E)a. 13% c. 15% (15.098%)b. 14% d. 16% RPCPA 0581

IRR Given After Tax Cash FlowsEven Cash Flows, No Present Value Table26. Valentine Company is considering investing in a new project. The project will need an initial

investment of $1,200,000 and will generate $600,000 (after-tax) cash flows for three years. Calculate the IRR for the project. (E)A. 14.5% C. 23.4%B. 18.6% D. 20.2% B & M

30. Elephant company is investing in a giant crane. It is expected to cost 2.2 million in initial investment and it is expected to generate an end of year cash flow of 1.0 million each year for three years. Calculate the IRR approximately. (E)A. 14.6 C. 22.1B. 16.4 D. 17.3 B & M

106. The capital budgeting director of Sparrow Corporation is evaluating a project which costs $200,000, is expected to last for 10 years and produce after-tax cash flows, including depreciation, of $44,503 per year. If the firm’s cost of capital is 14 percent and its tax rate is 40 percent, what is the project’s IRR? (E)a. 8% d. -5%b. 14% e. 12%

c. 18% Brigham

Even Cash Flow, With Present Value Table107. A project has a cost of $5,000 and is expected to produce a cash flow of $1,220 a year for five

years. Using the table given, what is the internal rate of return? (Note: Annuity factors are rounded to two places.) (E)

Future Value of an Annuity of $1 per Period for 5 Periods

Present Value of an Annuity of $1 per Period for 5 Periods

7% 5.75 4.108% 5.87 3.999% 5.98 3.8910% 6.11 3.79

A. 9% C. 7%B. 10% D. 8% CIA adapted

*. Progressive Corporation acquired an equipment at a cost of P40,500. It had an estimated life of ten years. Annual after tax net cash benefits are estimated to be P10,000 at the end of each year. The following amounts appear in the interest table for present value of an annuity of P1 at year end for ten years:

20% 4.1922% 3.9224% 3.68

What is the maximum rate that could be paid for the capital employed over the life of this asset without loss on this project? (M)a. 20% c. 22%b. 21% d. 23% RPCPA 1080

*. Jeroig, Inc. placed P300,000 in a ten-year project. The annual cash inflow after income taxes from this project was estimated to be P58,500. The company’s cut-off rate on investments of this type was 14%. Information on the present value factors is:

at 14% at 15%Present value of P1 for ten periods 0.270 0.247Present value of an annuity of P1 for ten periods 5.216 5.019

The company’s expected rate of return on this investment is (M) RPCPA 0583a. 14% c. Less than 14% but more than 0%b. 15% d. Less than 15% but more than 14%

*. Scott, Inc. is planning to invest $120,000 in a 10-year project. Scott estimates that the annual cash inflow, net of income taxes, from this project will be $20,000. Scott’s desired rate of return on investments of this type is 10%. Information on present value factors is as follows:

CMA EXAMINATION QUESTIONS Page 39 of 155

Page 40: P02 - Capital Budgeting

MANAGEMENT ADVISORY SERVICES CAPITAL BUDGETING

at 10% at 12%Present value of $1 for ten periods 0.386 0.322Present value of an annuity of $1 for ten periods 6.145 5.650

Scott’s expected rate of return on this investment isa. Less than 10% but more than 0% c. Less than 12% but more than 10%b. 10% d. 12% AICPA 1180 I-26

Uneven Cash Flow, No Present Value Table20. Given the following cash flows for Project M: C0 = -2,000, C1 = +500, C2 = +1,500, C3 =

+1455, calculate the IRR for the project. (E)A. 10% C. 28%B. 18% D. None of the above B & M

Multiple IRRs108. Two fellow financial analysts are evaluating a project with the following net cash flows:

Year Cash Flow 0 -$ 10,0001 100,0002 -100,000

One analyst says that the project has an IRR of between 12 and 13 percent. The other analyst calculates an IRR of just under 800 percent, but fears his calculator’s battery is low and may have caused an error. You agree to settle the dispute by analyzing the project cash flows. Which statement best describes the IRR for this project? (D)a. There is a single IRR of approximately 12.7 percent.b. This project has no IRR, because the NPV profile does not cross the X axis.c. There are multiple IRRs of approximately 12.7 percent and 787 percent.d. This project has two imaginary IRRs.e. There are an infinite number of IRRs between 12.5 percent and 790 percent that can

define the IRR for this project. Brigham

ComprehensiveQuestions 73 and 74 are based on the following information. BarfieldFordem Co. is considering an investment in a machine that would reduce annual labor costs by $30,000. The machine has an expected life of 10 years with no salvage value. The machine would be depreciated according to the straight-line method over its useful life. The company's marginal tax rate is 30 percent.

73. Assume that the company will invest in the machine if it generates an internal rate of return of 16 percent. What is the maximum amount the company can pay for the machine and still meet the internal rate of return criterion?a. $180,000 c. $187,500

b. $210,000 d. $144,996

74. Assume the company pays $250,000 for the machine. What is the expected internal rate of return on the machine?a. between 8 and 9 percent c. between 17 and 18 percentb. between 3 and 4 percent d. less than 1 percent

MODIFIED IRR27. Valentine Company is considering investing in a new project. The project will need an initial

investment of $1,200,000 and will generate $600,000 (after-tax) cash flows for three years. Calculate the MIRR (modified internal rate of return) for the project if the cost of capital is 15%. (E)A. 14.5% C. 23.4%B. 18.6% D. 20.2% B & M

31. Elephant company is investing in a giant crane. It is expected to cost 2.2 million in initial investment and it is expected to generate an end of year cash flow of 1.0 million each year for three years. Calculate the MIRR for the project if the cost of capital is approximately 12% APA. (E)A. 15.3% C. 23.8%B. 17.3% D. 22.1% B & M

109. A project requires an initial cash investment at its inception of $10,000, and no other cash outflows are necessary. Cash inflows from the project over its 3-year life are $6,000 at the end of the first year, $5,000 at the end of the second year, and $2,000 at the end of the third year. The future value interest factors for an amount of $1 at the cost of capital of 8% are

Period1 2 3 4

1.080 1.166 1.26 1.36The present value interest factors for an amount of $1 for three periods are as follows:

Interest Rate8% 9% 10% 12% 14%.794 .772 .751 .712 .675

The modified IRR (MIRR) for the project is closes toa. 8% c. 10%b. 9% d. 12% Gleim

110. Alyeska Salmon Inc., a large salmon canning firm operating out of Valdez, Alaska, has a new automated production line project it is considering. The project has a cost of $275,000 and is expected to provide after-tax annual cash flows of $73,306 for eight years. The firm’s management is uncomfortable with the IRR reinvestment assumption and prefers the modified

CMA EXAMINATION QUESTIONS Page 40 of 155

Page 41: P02 - Capital Budgeting

MANAGEMENT ADVISORY SERVICES CAPITAL BUDGETING

IRR approach. You have calculated a cost of capital for the firm of 12 percent. What is the project’s MIRR? (M)a. 15.0% d. 16.0%b. 14.0% e. 17.0%c. 12.0% Brigham

111. Below are the returns of Nulook Cosmetics and “the market” over a three-year period:Year Nulook Market

1 8% 6%2 9 93 32 22

Nulook finances internally using only retained earnings, and it uses the Capital Asset Pricing Model with a historical beta to determine its cost of equity. Currently, the risk-free rate is 7 percent, and the estimated market risk premium is 6 percent. Nulook is evaluating a project which has a cost today of $2,028 and will provide estimated cash inflows of $1,000 at the end of the next 3 years. What is this project’s MIRR? (M)a. 12.4% d. 20.0%b. 16.0% e. 22.9%c. 17.5% Brigham

INFLATIONARY ENVIRONMENTRate of ReturnMinimum Desired Rate of Return112. You just passed the CPA licensure examination and took your oath. As you started your

practice, Kon Fuse, Inc. came to you for help in establishing a minimum desired rate of return to be used in the evaluation of a capital project with a five year life. The following data were provided: (D)

Inflation rate for the past 5 years 13%Expected inflation rate for the next five years 9%“Risk-free” element 5%“Risk” premium demanded for the project 7%

You will advice the client to consider a minimum desired rate of return ofa. 20% c. 16%b. 21% d. 25% RPCPA 0596

Nominal Rate of Interest14. The real rate of interest is 3 % and the inflation is 4%. What is the nominal rate of interest?

A. 3% C. 7.12%B. 4% D. 1% B & M

16. The real interest rate is 3% and the inflation rate is 6%. What is the nominal interest rate?

A. 3% C. 9.2%B. 4% D. 1% B & M

Real Interest Rate17. If the nominal interest rate is 8.1% and the inflation rate is 4%, what is the real interest rate?

A. 3.85% C. 4%B. 8% D. None of the above B & M

Cash FlowReal Cash Flow15. A cash flow received in two years is expected to be $11,236. If the real rate of interest is 4%

and the inflation rate is 6%, what is the real cash flow for year 2? A. $11,236 C. $10,000B. $10,388 D. $9,246 B & M

Nominal Cash Flow19. Real cash flow occurring in year 2 is 50,000. If the inflation rate is 10% per year, calculate

nominal cash flow for year 2. A. 60,500 C. 55,000B. 50,000 D. None of the above B & M

Net Present Value – Infinite Life29. You have been asked to evaluate a project with infinite life. Sales and costs are projected to

be $1000 and $500 respectively. There is no depreciation and the tax rate is 30%. The real required rate of return is 10% The inflation rate is 4% and is expected to be 4% forever. Sales and costs will increase at the rate of inflation. If the project costs $3000, what is the NPV? A. $240.74 C. $500.00B. $1629.62 D. None of the above B & M

30. A project costs $100 today. It has sales of $100 per year forever. Costs will be $50 the first year and increase by 19% per year. Ignoring taxes calculate the NPV of the project at the discount rate of 10%. A. $11.62 C. $100.00B. $65.00 D. Cannot be calculated as g > g B & M

Investment Decision27. You own 100 acres of timberland, with young timber worth $20,000 if logged today. This

represents 500 cords of wood at $40 per cord. After logging, the land can be sold today for $10,000 ($100 per acre). The opportunity cost of capital is 10%. You have made the following estimates:i) The price of a cord of wood will increase by 5% per year.

CMA EXAMINATION QUESTIONS Page 41 of 155

Page 42: P02 - Capital Budgeting

MANAGEMENT ADVISORY SERVICES CAPITAL BUDGETING

ii) The price of land will increase by 3% per year.iii) The yearly growth rate of the mutual cords of wood on your land are: years 1-2: 15%;

years 3-4: 10%; years 5-8: 5%; years thereafter: 2%.The present value of the optimal decision is approximately: A. $30,000 C. $34,250B. $32,800 D. $34,315 B & M

28. You own 100 acres of timberland, with young timber worth $20,000 if logged today. This represents 500 cords of wood at $40 per cord. After logging, the land can be sold today for $10,000 ($100 per acre). The opportunity cost of capital is 10%. You have made the following estimates:i) The price of a cord of wood will increase by 5% per year.ii) The price of land will increase by 3% per year.iii) The yearly growth rate of the mutual cords of wood on your land are: years 1-2: 15%;

years 3-4: 10%; years 5-8: 5%; years thereafter: 2%.The optimal decision is to sell after: A. 8 years C. 4 yearsB. 5 years D. 3 years B & M

POINT OF INDIFFERENCE – COST OF CAPITALRequired Investment113. Athey Airlines is considering two mutually exclusive projects, Project A and Project B. The

projects have the following cash flows (in millions of dollars):Year Project A Cash Flow Project B Cash Flow0 -$4.0 ?1 2.0 $1.72 3.0 3.23 5.0 5.8

The crossover rate of the two projects’ NPV profiles is 9 percent. Consequently, when the WACC is 9 percent the projects have the same NPV. What is the cash flow for Project B at t = 0? (M)a. -$4.22 d. +$4.22b. -$3.49 e. -$4.51c. -$8.73 Brigham

Even Cash Flow16. PDLT Investment which has a weighted average cost of capital of 12% is evaluating two

mutually exclusive projects (X and Y), which have the following projections:Project X Project Y

Investment P48,000 P83,225After-tax cash inflow 12,000 15,200

Asset life 6 years 10 yearsThe fisher rate for the two projects isA. 12.64% C. 16.01%B. 12.00% D. 19.33% Pol Bobadilla

114. Suzie owns a computer reselling business and is expanding her business. Suzie is presented with one proposal, Proposal A, such that the estimated investment for the expansion project is $85,000, and it is expected to produce cash flows after taxes of $25,000 for each of the next 6 years. An alternate proposal, Proposal B, involves an investment of $32,000 and after-tax cash flows of $10,000 for each of the next 6 years. The cost of capital that would make Suzie indifferent between these two proposals lies betweena. 10% and 12% c. 16% and 18%b. 14% and 16% d. 18% and 20% Gleim

57. Berry Products is considering two pieces of machinery. The first machine costs P50,000 more than the second machine. During the two-year life of these two alternatives, the first machine has P155,000 more cash flow in year one and a P110,000 less cash flow in year two than the second machine. All cash flows occur at year-end. The present value of 1 at 15% end of 1 period and 2 periods are 0.86957 and 0.75614, respectively. The present value of 1 at 8% end of period 1 is 0.92593 and period 2 is 0.85734.At what discount rate would Machine 1 equally acceptable as machine 2? (D)A. 9% C. 11%B. 10% D. 12% RPCPA 0503

Questions 75 and 76 are based on the following information. BarfieldThe net after-tax cash flows associated with two projects under consideration by Novelle Co. follow:

Project 1 Project 2 Initial investment $(300,000) $(100,000)Cash flows years 1-5 80,000 30,000

75. What is the Fisher rate for these two projects? (D)a. less than 1 percent c. between 4 and 5 percentb. between 7 and 8 percent d. between 6 and 7 percent

76. Assume that the company can potentially accept both projects, one project, or neither project. Which project(s) would the company accept if it estimates its weighted average cost of capital is 9 percent?a. both projects c. Project 2b. Project 1 d. neither project

CMA EXAMINATION QUESTIONS Page 42 of 155

Page 43: P02 - Capital Budgeting

MANAGEMENT ADVISORY SERVICES CAPITAL BUDGETING

Uneven Cash Flow115. Two projects being considered are mutually exclusive and have the following projected cash

flows:Year Project A Cash Flow Project B Cash Flow

0 -$50,000 -$ 50,0001 15,990 02 15,990 03 15,990 04 15,990 05 15,990 100,560

At what rate (approximately) do the NPV profiles of Projects A and B cross? (D)a. 6.5%b. 11.5%c. 16.5%d. 20.0%e. The NPV profiles of these two projects do not cross. Brigham

EVALUATION OF INVESTMENT ALTERNATIVESNet Present Value116. The U.S. Postal Service is looking for a new machine to help sort the mail. Two companies

have submitted bids to Cliff Kraven, the postal inspector responsible for choosing a machine. A cash flow analysis of the two machines indicates the following:

Year Machine A Machine B0 -$30,000 -$30,000

1 0 13,0002 0 13,0003 0 13,0004 60,000 13,000

If the cost of capital for the Postal Service is 8%, which of the two mail sorters should Cliff choose and why? (M)A. Machine A, because NPVA > NPVB, by $1,044. B. Machine B, because NPVA < NPVB, by $22,000. C. Machine A, because NPVA > NPVB, by $8,000. D. Machine B, because IRRA < IRRB. Gleim

117. Rohan Transport is considering two alternative buses to transport people between cities that are in the Southeastern U.S., such as Baton Rouge and Gainesville. A gas-powered bus has a cost of $55,000, and will produce end-of-year net cash flows of $22,000 per year for 4 years. A new electric bus will cost $90,000, and will produce cash flows of $28,000 per year for 8 years. The company must provide bus service for 8 years, after which it plans to give up its franchise and to cease operating the route. Inflation is not expected to affect either costs or revenues during the next 8 years. If Rohan Transport's cost of capital is 17 percent, by what amount will the better project increase the company's value? (D)A. $5,350 C. $10,701B. -$17,441 D. $27,801 Gleim

Present Value of Costs118. Union Electric Company must clean up the water released from its generating plant. The

company's cost of capital is 11 percent for average projects, and that rate is normally adjusted up or down by 2 percentage points for high- and low-risk projects. Clean-Up Plan A, which is of average risk, has an initial cost of $10 million, and its operating cost will be $1 million per year for its 10-year life. Plan B, which is a high-risk project, has an initial cost of $5 million, and its annual operating cost over Years 1 to 10 will be $2 million. What is the approximate PV of costs for the better project? (VD)A. -$5.9 million. C. -$16.8 million.B. -$15.9 million. D. -$17.8 million. Gleim

Weighted-Average Cost of Capital (WACC)119. Mulva Inc. is considering the following five independent projects:

Project Required Amount of Capital IRRA $300,000 25.35%B 500,000 23.22%C 400,000 19.10%D 550,000 9.25%E 650,000 8.50%

CMA EXAMINATION QUESTIONS Page 43 of 155

Page 44: P02 - Capital Budgeting

MANAGEMENT ADVISORY SERVICES CAPITAL BUDGETING

The company has a target capital structure which is 40 percent debt and 60 percent equity. The company can issue bonds with a yield to maturity of 10 percent. The company has $900,000 in retained earnings, and the current stock price is $40 per share. The flotation costs associated with issuing new equity are $2 per share. Mulva's earnings are expected to continue to grow at 5 percent per year. Next year's dividend (D1) is forecasted to be $2.50. The firm faces a 40 percent tax rate. What is the size of Mulva's capital budget? (D)A. $1,200,000 C. $2,400,000B. $1,750,000 D. $800,000 Gleim

PROJECT SCREENING METHODSingle Project*. Universal Corp. is reviewing a capital budgeting decision regarding the acquisition of a capital

equipment. Below are the relevant information:Investment P300,000PV of net cash inflows 200,000Cash-flow tax shield from depreciation 100,000

The company is used to have as benchmark for similar projects an excess present value index of 0.50, that is, the project’s index should be no less than 0.50. Should this project be pursued? (M)a. No, since the excess present value index is 0.33b. Yes, since the excess present value index is 0.67c. No, since the excess present value index is less than 0.50d. Yes, since the excess present value index is 1.50 RPCPA 0597

*. A tax-exempt foundation, Sincerely Foundation, Inc. intends to invest P1 million in a five-year project. The foundation estimates that the annual savings from the project will amount to P325,000. The P1 million asset is depreciable over five (5) years on a straight-line basis. The foundation’s hurdle rate is 12% and as a consultant of the foundation, you are asked to determine the internal rate of return and advise if the project should be pursued.To facilitate computations, below are present value factors:

N=5 12% 14% 16%Present value of P1 0.57 0.52 0.48Present value of an annuity of P1 3.60 3.40 3.30

Your advice is (M)a. To proceed due to an estimated IRR of less than 14% but not more than 12%.b. To proceed due to an estimated IRR of less than 16% but not more than 14%.c. Not to proceed due to an estimated IRR of less than 12%.d. To proceed due to an estimated IRR of more than 16%. RPCPA 1095

Unlimited Capital*. SB Co. uses a 12% hurdle rate for all capital expenditures. It has lined up four projects and

below is the summary thereof.In Thousand Pesos

Project 1 Project 2 Project 3 Project 4Initial cash outflow 400 596 496 544Annual cash inflow

Year 1 130 200 160 190 2 140 270 190 250

CMA EXAMINATION QUESTIONS Page 44 of 155

Page 45: P02 - Capital Budgeting

MANAGEMENT ADVISORY SERVICES CAPITAL BUDGETING

3 160 180 180 180 4 80 130 160 120

Net present value (7.500) 8.552 28.128 29.324Profitability Index 98% 101% 106% 105%Internal rate of return

11% 13% 14% 15%

If the company has no budgetary limitations, which projects should be pursued? (E)a. Project 1. c. Projects 2, 3 and 4.b. Projects 3 and 4. d. All the four projects. RPCPA 1096

*. The following data relate to two capital-budgeting projects of equal risk:Present Value of Cash Flows

Period Project A Project B0 $(10,000) $(30,000)1 4,550 13,6502 4,150 12,4503 3,750 11,250

Which of the projects will be selected using the profitability index (PI) approach and the NPV approach?CIA 0586 IV-33 a. b. c. d.PI B Either Either BNPV A B A B

RANKING OF INVESTMENT ALTERNATIVES AND PROJECT SELECTIONCapital Budget120. A company's marginal cost of new capital (MCC) is 10% up to $600,000. MCC increases .5%

for the next $400,000 and another .5% thereafter. Several proposed capital projects are under consideration, with projected cost and internal rates of return (IRR) as follows:

Project Cost IRRA $100,000 10.5%B $300,000 14.0%C $450,000 10.8%D $350,000 13.5%E $400,000 12.0%

What should the company's capital budget be? A. $0 C. $1,500,000B. $1,050,000 D. $1,600,000 CIA 0589 IV-55

Mutually Exclusive40. Which of the following results of the net present value method in capital budgeting is the

LEAST acceptable? (E)a. $(10,000) c. $(18,000)b. $(7,000) d. $0 Horngren

34. The projects have the following NPVs and project lives.Project NPV LifeProject A $5,000 4 years

CMA EXAMINATION QUESTIONS Page 45 of 155

Page 46: P02 - Capital Budgeting

MANAGEMENT ADVISORY SERVICES CAPITAL BUDGETING

Project B $7,000 7 yearsIf the cost of capital is 12%, which project would you accept? A. A C. Both A and BB. B D. Reject both A and B B & M

56. A firm is considering two mutually exclusive projects with the following cash flows:Project A Project B

Year 1 $20,000 $60,000Year 2 $40,000 $40,000

Year 3 $60,000 $20,000Each project requires an investment of $50,000. The cost of capital is 10 percent.Which project will have the higher net present value?a. Project Ab. Project Bc. Project A and Project B will have the same net present value. H & Md. It is not possible to answer the question based upon the information provided.

121. If the investment projects listed below are mutually exclusive, which alternative should be accepted?

Project A B C DNPV $100,000 ($20,000) $60,000 $30,000

A. Project A. C. Projects A and C.B. Project B. D. Projects A, C, and D. CIA 0595 IV-38

122. Five mutually exclusive projects had the following information:A B C D E

NPV $500 $(200) $100 $200 $1,000IRR 12% 8% 11% 13% 10%

Which project is preferred? (M)a. A d. Db. B e. Ec. C H & M

123. Five mutually exclusive projects had the following information:V W X Y Z

NPV $5,000 $10,000 $(3,000) $20,000 $15,000IRR 10% 12% 8% 11% 13%

Which project is preferred?a. V d. Yb. W e. Zc. X H & M

124. As the director of capital budgeting for Denver Corporation, you are evaluating two mutually exclusive projects with the following net cash flows:

Year Project X Cash Flow Project Z Cash Flow0 -$100,000 -$100,0001 50,000 10,0002 40,000 30,0003 30,000 40,000

CMA EXAMINATION QUESTIONS Page 46 of 155

Page 47: P02 - Capital Budgeting

MANAGEMENT ADVISORY SERVICES CAPITAL BUDGETING

4 10,000 60,000If Denver’s cost of capital is 15 percent, which project would you choose? (E)a. Neither project. d. Project X, since it has the higher NPV.b. Project X, since it has the higher IRR. e. Project Z, since it has the higher IRR.c. Project Z, since it has the higher NPV. Brigham

125. Two projects being considered are mutually exclusive and have the following projected cash flows:

Year Project A Cash Flow Project B Cash Flow0 -$50,000 -$50,0001 15,625 02 15,625 03 15,625 04 15,625 05 15,625 99,500

If the required rate of return on these projects is 10 percent, which would be chosen and why? (E)a. Project B because it has the higher NPV.b. Project B because it has the higher IRR.c. Project A because it has the higher NPV.d. Project A because it has the higher IRR.e. Neither, because both have IRRs less than the cost of capital. Brigham

*. Each of three mutually exclusive projects costs $200. Using the table provided, rank the projects in descending NPV order. (M)

Present Value Interest Factor (10%)

Project’s Cash FlowYear A B C

1 .91 $300 $200 $ 02 .83 200 100 1003 .75 100 0 1004 .68 0 100 2005 .62 0 200 300

a. A, B, C. c. C, B, A.b. B, A, C. d. A, C, B. CIA 0585 IV-33

Questions 55 and 56 are based on the following information. CMA 1291 4-8 & 9Mercken Industries is contemplating four projects, Project P, Project Q, Project R, and Project S. The capital costs and estimated after-tax net cash flows of each mutually exclusive project are listed below. Mercken's desired after-tax opportunity cost is 12%, and the company has a capital budget for the year of $450,000. Idle funds cannot be reinvested at greater than 12%.

Project P Project Q Project R Project SInitial cost $200,000 $235,000 $190,000 $210,000Annual cash flows Year 1 $93,000 $90,000 $45,000 $40,000 Year 2 93,000 85,000 55,000 50,000 Year 3 93,000 75,000 65,000 60,000 Year 4 -0- 55,000 70,000 65,000 Year 5 -0- 50,000 75,000 75,000

CMA EXAMINATION QUESTIONS Page 47 of 155

Page 48: P02 - Capital Budgeting

MANAGEMENT ADVISORY SERVICES CAPITAL BUDGETING

Net present value $23,370 $29,827 $27,333 $(7,854)Internal rate of return 18.7% 17.6% 17.2% 10.6%Excess present value index 1.12 1.13 1.14 .96

126. During this year, Mercken will choose A. Projects P, Q, and R. C. Projects Q and R.B. Projects P, Q, R, and S. D. Projects P and Q.

127. If Mercken is able to accept only one project, the company would choose A. Project P. B. Project Q because it has the highest net present value. C. Project P because it has the highest internal rate of return. D. Project P because it has the shortest payback period.

*. A capital budgeting decision model has provided the following information:Proposal A Proposal B

Investment P1,000,000 P1,800,000Profitability Index 1.2 2.1Net present Value P600,000 P300,000

The best project is(situation is impossible)a. Proposal A because it has the highest present value.b. Proposal B because it has the highest profitability index.c. Proposal B because its profitability index is over 2.0d. Proposal A because it has the highest net present value even though its investment base

in smaller. RPCPA 0595

Excess Present Value Index*. Telephone Corp. is contemplating four projects: L, M, N, and O. The capital costs for the

initiation of each mutually-exclusive project and its estimated after-tax, net cash flow are listed below. The company’s desired after-tax opportunity costs is 12%. It has P900,000 capital budget for the year. Idle funds cannot be reinvested at greater than 12%.

In Thousand PesosL M N O

Initial cost 400 470 380 420Annual cash flows

Year 1 113 180 90 80 2 113 170 110 100 3 113 150 130 120 4 113 110 140 130 5 113 100 150 150

Net present value P7,540 P59,654 P54,666 P(15,708)Internal rate of return 12.7% 17.6% 17.2% 10.6%Excess present value index 1.02 1.13 1.14 0.96

The company will choose (E)a. Projects M, N and O. c. Projects L and N.b. Projects M and N. d. Projects L and M. RPCPA 0595

EAC35. Two machines, A and B, which perform the same functions, have the following costs and lives.

Type PV Costs LifeMachine A $6000 5Machine B $8000 7

Which machine would you choose? The two machines are mutually exclusive and the cost of capital is 15%. A. Machine A as the EAC is $1789.89 C. Don't buy either machineB. Machine B as the EAC is $1922.88 B & M

Internal Rate of ReturnMutually exclusive projectsInternal Rate of Return128. Two projects being considered by a firm are mutually exclusive and have the following

projected cash flows:Year Project A Cash Flow Project B Cash Flow

0 ($100,000) ($100,000)1 39,500 02 39,500 03 39,500 133,000

Based only on the information given, which of the two projects would be preferred, and why? (M)a. Project A, because it has a shorter payback period.b. Project B, because it has a higher IRR.c. Indifferent, because the projects have equal IRRs.d. Include both in the capital budget, since the sum of the cash inflows exceeds the initial

investment in both cases.e. Choose neither, since their NPVs are negative. Brigham

Payback PeriodQuestions 18 & 19 are based on the following information. GitmanA firm is evaluating two projects that are mutually exclusive with initial investments and cash flows as follows:

CMA EXAMINATION QUESTIONS Page 48 of 155

Page 49: P02 - Capital Budgeting

MANAGEMENT ADVISORY SERVICES CAPITAL BUDGETING

Project: A Project: BInitial

InvestmentEnd-of-Year Cash Flows

Initial Investment

End-of-Year Cash Flows

$40,000 $20,000 $90,000 $40,00020,000 40,00020,000 80,000

18. If the firm in Figure 901 has a required payback of two (2) years, they shouldA. accept projects A and B. C. reject project A and accept B.B. accept project A and reject B. D. reject both.

19. The new financial analyst does not like the payback approach and determines that the firm's required rate of return is 15%. His recommendation would be toA. accept projects A and B. C. reject project A and accept B.B. accept project A and reject B. D. reject both.

Replacement Chain33. OM Construction Company must choose between two types of cranes. Crane A costs

$600,000, will last for 5 years, and will require $60,000 in maintenance each year. Crane B costs $750,000 and will last for seven years. Maintenance costs for crane B are incurred at the end of each year. The appropriate discount rate is 12% per year. Which machine should OM Construction purchase? A. Crane A as EAC is $226,444 B. Crane B as EAC is $194,336 C. Crane A as the PV is $816,286 D. Cannot be calculated as the revenues for the project are not given B & M

129. Borden Books is interested in purchasing a computer system to use for the next 10 years. Currently, Borden is considering two mutually exclusive systems, System S and System L. System S has an up-front cost of $3 million at t = 0 and will produce positive cash flows of $2.5 million per year for two years (at t = 1 and 2). This system can be repeated forever. In other words, every two years the company can repurchase the system under exactly the same terms. System L has an up-front cost of $5 million at t = 0 and will produce positive cash flows of $2 million per year for five years (at t = 1, 2, 3, 4, and 5). This system can be replaced at a cost of $4 million at t = 5, after which time it will produce positive cash flows of $1.5 million per year for the subsequent five years (at t = 6, 7, 8, 9, and 10).Borden’s CFO has determined that the company’s WACC is 12 percent. Over a 10-year extended basis, which system is the better system and what is its NPV? (M)a. System L; $2.21 million d. System L; $4.41 millionb. System L; $3.01 million e. System S; $6.13 million

c. System S; $4.10 million Brigham

Capital RationingNet Present Value*. The Nativity Corporation has the following investment opportunities:

Proposal Profitability Index Initial Cash Outlay1 1.15 P200,0002 1.13 125,0003 1.11 175,0004 1.08 150,000

The firm has a budget constraint of P300,000.What proposal(s) should be accepted? (D)a. Proposal 1 because it has the highest profitability index.b. Proposal 4 because it has the lowest profitability index.c. Proposals 2 and 3 because their total net present values are the highest among all

possible proposal combinations.d. Proposals 1 and 2 because their total net present values are the highest among all

possible proposal combinations. RPCPA 0579

36. The following table gives the available projects for a firm.A B C D E F G90 20 60 50 150 40 20 Initial investment140 70 65 -10 30 32 10 NPV

If the firm has a limit of 200 million to invest, what is the maximum NPV the company can obtain? (M)

A. 200 C. 283B. 243 D. None of the above B & M

37. The following table gives the available projects for a firm.A B C D E F G

5.0 4.0 5.0 1.0 2.0 7.0 8.0 Initial investment1.5 -0.5 1.0 0.5 0.5 1.0 1.0 NPV

The firm has only twenty million to invest. What is the maximum NPV that the company can obtain? (M)A. 3.5 C. 4.0B. 4.5 D. None of the above B & M

Profitability Index*. Information on three (E) investment projects is given below:

Project Investment Required Net Present Value

CMA EXAMINATION QUESTIONS Page 49 of 155

Page 50: P02 - Capital Budgeting

MANAGEMENT ADVISORY SERVICES CAPITAL BUDGETING

X P150,000 P34,005G 100,000 22,670W 60,000 13,602

Rank the projects in terms of preference: (M)a. 1st W; 2nd G; 3rd X. c. 1st X; 2nd G; 3rd W.b. 1st G; 2nd W; 3rd X. d. The ranking is the same. RPCPA 1097

69. Perkins Company is considering several investment proposals, as shown below:Investment Proposal

A B C DInvestment required $80,000 $100,000 $60,000 $75,000Present value of future net cash flows 96,000 150,000 84,000 120,000

Rank the proposals in terms of preference using the profitability index: (E)a. D, B, C, A. c. B, D, A, C.b. B, D, C, A. d. A, C, B, D. G & N 9e

70. Information on four investment proposals is given below:

Proposal Investment Net Present Value1 $50,000 $30,0002 60,000 24,0003 30,000 15,0004 45,000 9,000

Rank the proposals in terms of preference according to the profitability index: (E)a. 3, 4, 1, 2. c. 1, 3, 2, 4.b. 1, 2, 3, 4. d. 2, 1, 4, 3. G & N 9e

*. The Sarmiento Company has the following investment opportunities:Proposal Profitability Index Initial Cash Outlay

1 1.25 P400,0002 1.19 100,0003 1.16 175,0004 1.14 125,0005 1.09 200,0006 1.05 100,000

CMA EXAMINATION QUESTIONS Page 50 of 155

Page 51: P02 - Capital Budgeting

MANAGEMENT ADVISORY SERVICES CAPITAL BUDGETING

7 0.97 150,000If the budget ceiling for initial outlays during the present period was P1,000,000 and the proposals were independent of each other, the Company should engage in proposal(s) (E)a. Proposal 1 to 6 because their profitability indeces are all greater than 1.b. Proposal 7 because its profitability index is less than 1.c. Proposal 1 to 5 because their profitability indeces are greater. RPCPA 0579d. Proposals 1, 3, 4, 5 and 6 because their profitability indeces are greater than 1.

Questions 26 and 27 are based on the following information. CIA 1196 IV-40 & 41Investment Project Cash Outlay Present Value of Cash Inflows

A $1,100,000 $ 980,000B 250,000 600,000C 1,400,000 1,830,000D 650,000 790,000

The company has $2 million of financing available for new investment projects.

130. The investment project with the highest profitability index is A. Project A. C. Project C.B. Project B. D. Project D.

131. If only one project may be selected, which should the company undertake? A. Project A. C. Project C.B. Project B. D. Project D.

Net Present Value & Internal Rate of ReturnQuestions 20 & 21 are based on the following information. GitmanA firm must choose from six capital budgeting proposals outlined below. The firm is subject to capital rationing and has a capital budget of $1,000,000; the firm's cost of capital is 15 percent.

Project Initial Investment IRR NPV1 $200,000 19% $100,0002 400,000 17 20,0003 250,000 16 60,0004 200,000 12 - 5,0005 150,000 20 50,0006 400,000 15 150,000

20. Using the internal rate of return approach to ranking projects, which projects should the firm accept?A. 1, 2, 3, 4, and 5 C. 2, 3, 4, and 6B. 1, 2, 3, and 5 D. 1, 3, 4, and 6

21. Using the net present value approach to ranking projects, which projects should the firm accept? A. 1, 2, 3, 4, and 5 C. 2, 3, 4, and 5B. 1, 2, 3, 5, and 6 D. 1, 3, 5, and 6

Comprehensive45. Investors, Inc. uses a 12% hurdle rate for all capital expenditures and has done the following

analysis for four projects for the upcoming year:Project 1 Project 2 Project 3 Project 4

Initial cash outlay P200,000 P298,000 P248,000 P272,000Annual net cash inflows Year 1 P 65,000 P100,000 P 80,000 P 95,000

CMA EXAMINATION QUESTIONS Page 51 of 155

Page 52: P02 - Capital Budgeting

MANAGEMENT ADVISORY SERVICES CAPITAL BUDGETING

Year 2 70,000 135,000 95,000 125,000 Year 3 80,000 90,000 90,000 90,000 Year 4 40,000 65,000 80,000 60,000Net present value ( 3,798) 4,276 14,064 14,662Profitability index 98% 101% 106% 105%Internal rate of return 11% 13% 14% 15%

Which project(s) should Investors, Inc. select during the upcoming year under each budgeted amount of funds? Pol Bobadilla

No Budget Restriction P600,000 Available Funds P300,000 Available Funds

a. Projects 2,3, & 4 Projects 3 & 4 Project 3b. Projects 1, 2, & 3 Projects 2, 3 & 4 Projects 3 & 4c. Projects 1, 3, & 4 Projects 2 & 3 Project 2d. Projects 3 & 4 Projects 2 & 4 Projects 2 & 4

Questions 63 through 65 are based on the following information. GleimMaloney Company uses a 12% hurdle rate for all capital expenditures and has done the following analysis for four projects for the upcoming year:

Project 1 Project 2 Project 3 Project 4

CMA EXAMINATION QUESTIONS Page 52 of 155

Page 53: P02 - Capital Budgeting

MANAGEMENT ADVISORY SERVICES CAPITAL BUDGETING

Initial outlay $4,960,000 $5,440,000 $4,000,000 $5,960,000Annual net cash inflows Year 1 1,600,000 1,900,000 1,300,000 2,000,000 Year 2 1,900,000 2,500,000 1,400,000 2,700,000 Year 3 1,800,000 1,500,000 1,600,000 1,800,000 Year 4 1,600,000 1,200,000 800,000 1,300,000Net present value 281,280 293,240 (75,960) 85,520Profitability index 106% 105% 98% 101%Internal rate of return 14% 15% 11% 13%

132. What project(s) should Maloney undertake during the upcoming year assuming it has no budget restrictions?a. All of the projects. c. Projects 1,2 and 4b. Projects 1, 2, and 3 d. Projects 1 and 2

133. Which projects should Maloney undertake during the upcoming year if it has only $12,000,000 of investment funds available?a. Projects 1 and 3 c. Projects 1 and 4.b. Projects 1, 2, and 4. d. Projects 1 and 2.

134. Which project(s) should Maloney undertake during the upcoming year if it has only $6,000,000 of funds available?a. Project 3. c. Project 1.b. Projects 1 and 2. d. Project 2.

Questions 60 through 62 are based on the following information. CMA 695 4-7 to 9Capital Invest Inc. uses a 12% hurdle rate for all capital expenditures and has done the following analysis for four projects for the upcoming year:

Project 1 Project 2 Project 3 Project 4Initial capital outlay $200,000 $298,000 $248,000 $272,000Annual net cash inflows Year 1 $ 65,000 $100,000 $ 80,000 $ 95,000 Year 2 70,000 135,000 95,000 125,000 Year 3 80,000 90,000 90,000 90,000 Year 4 40,000 65,000 80,000 60,000Net present value (3,798) 4,276 14,064 14,662Profitability index 98% 101% 106% 105%Internal rate of return 11% 13% 14% 15%

135. Which project(s) should Capital Invest Inc. undertake during the upcoming year assuming it has no budget restrictions?

A. All of the projects. C. Projects 2, 3, and 4.B. Projects 1, 2, and 3. D. Projects 1, 3, and 4.

136. Which project(s) should Capital Invest Inc. undertake during the upcoming year if it has only $600,000 of funds available? A. Projects 1 and 3. C. Projects 2 and 3.B. Projects 2, 3, and 4. D. Projects 3 and 4.

137. Which project(s) should Capital Invest Inc. undertake during the upcoming year if it has only $300,000 of capital funds available? A. Project 1. C. Projects 3 and 4.B. Projects 2, 3, and 4. D. Project 3.

Questions 73 thru 78 are based on the following information.A company that annually reviews its investment opportunities and selects appropriate capital expenditures for the coming year is presented with two projects, called Project A and Project B. Best estimates indicate that the investment outlay for Project A is $30,000 and for Project B is $1 million. The projects are considered to be equally risky. Project A is expected to generate cash inflows of $40,000 at the end of each year for 2 years. Project B is expected to generate cash inflows of $700,000 at the end of the first year and $500,000 at the end of the second year. The company has a cost of capital of 8%.

138. What is the net present value (NPV) of each project when the cost of capital is zero?CIA 0594 IV-40 A. B. C. D.Project A $30,000 $50,000 $80,000 $110,000Project B $1,000,000 $200,000 $1,200,000 $2,200,000

139. The internal rate of return of Project A, to the nearest full percentage point, isA. 10% C. 25%B. 15% D. 100% CIA 0594 IV-41

140. Net present value (NPV) and internal rate of return (IRR) differ in thatA. NPV assumes reinvestment of project cash flows at the cost of capital, whereas IRR

assumes reinvestment of project cash flows at the internal rate of return. CIA 0594 IV-42B. NPV and IRR make different accept or reject decisions for independent projects.C. IRR can be used to rank mutually exclusive investment projects, butt NPV cannot.D. NPV is expressed as a percentage, while IRR is expressed as a dollar amount.

141. If Projects A and B are mutually exclusive, which of the following statements is true? A. The company should undertake Project A only. B. The company should undertake Project B only.

CMA EXAMINATION QUESTIONS Page 53 of 155

Page 54: P02 - Capital Budgeting

MANAGEMENT ADVISORY SERVICES CAPITAL BUDGETING

C. The company should undertake both projects. D. The company should not undertake either project. CIA 0594 IV-43

142. If Projects A and B are independent, which of the following statements is true? A. The company should undertake Project A only. B. The company should undertake Project B only. C. The company should undertake both projects. D. The company should not undertake either project. CIA 0594 IV-44

143. If the net present value (NPV) of project A is known to be higher than the NPV of project B, it can be concluded that A. The internal rate of return (IRR) of project A will definitely be higher than the IRR of project

B. B. The IRR of project A will definitely be lower than the IRR of project B. C. The ranking of IRRs is indeterminate based on the information provided. D. The payback period for project A is definitely shorter than the payback period for project

B. CIA 1190 IV-54

PROJECT IMPLEMENTATION & REVIEWQuestions 28 and 29 are based on the following information. RPCPA 0593The construction of a waste treatment plant was arrived at after a careful cost-benefit analysis. During the construction period a status report was presented for your review:

completed cost as originally estimated, P5 million % of actual completion to date, 65% actual cost to date, P3.75 million

*. Assuming cost is evenly distributed throughout the construction period, how much will the completion cost be most likely? (E)a. The original cost estimate of P5 million.b. P5 million plus a cost overrun of about P769,000c. P500,000 less than the original cost at completion.d. About P100,000 above the original cost at completion.

*. What would be an appropriate action to take considering the situation in number 28?a. No need to take any action.b. Immediately stop further work on the project.c. Recommend immediate review with the project implementation team to determine the

cause of overrun and the corrective actions to be taken.d. Wait for the next quarterly status report on the project.

PROBABILITIES & EXPECTED VALUE ANALYSIS

144. What is the expected value of NPV (to the nearest dollar) for the following situation? The firm expects an NPV of $10,000 if the economy is exceptionally strong (40% probability), an NPV of $4,000 if the economy is normal (40% probability), and an NPV of -$2,000 if the economy is exceptionally weak (20% probability). (M)A. $5,600 C. $6,000B. $5,200 D. None of the answers are correct. Gleim

Questions 5 thru 7 are based on the following information. GitmanA corporation is assessing the risk of two capital budgeting proposals. The financial analysts have developed pessimistic, most likely, and optimistic estimates of the annual cash inflows which are given in the following table. The firm's cost of capital is 10 percent.

Project A Project BInitial

InvestmentAnnual

Cash inflow OutcomeInitial

InvestmentAnnual

Cash Inflow Outcome$20,000 $ 5,000 Pessimistic $100,000 $ 20,000 Pessimistic

10,000 Most likely 40,000 Most likely15,000 Optimistic 100,000 Optimistic

5. The range of the annual cash inflows for Project A is (E)A. $30,000. C. $5,000.B. $10,000. D. $0.

6. If the projects have five-year lives, the range of the net present value for Project B is approximately (M)A. $80,560. C. $255,410.B. $201,000. D. $303,280.

7. The expected net present value of project A if the outcomes are equally probable and the project has five-year life is (M)A. -$ 1,045. C. $36,865.B. $17,910. D. $93,730.

Questions 8 thru 12 are based on the following information. GitmanA firm is evaluating the relative riskiness of two capital budgeting projects. The following table summarizes the net present values and associated probabilities for various outcomes for the two projects.

Net Present ValueProbability Project A Project B

0.25 -$ 5,000 00.50 4,000 $ 2,0000.25 10,000 8,000

CMA EXAMINATION QUESTIONS Page 54 of 155

Page 55: P02 - Capital Budgeting

MANAGEMENT ADVISORY SERVICES CAPITAL BUDGETING

8. The expected net present value for projects A and B are A. $4,000 and $1,500, respectively. C. $3,250 and $3,000, respectively.B. $2,000 and $1,000, respectively. D. $3,000 and $3,300, respectively.

9. The standard deviation for projects A and B are A. $3,000 and $5,000, respectively. C. $2,106 and $0, respectively.B. $4,210 and $2,104, respectively. D. $5,356 and $3,000, respectively.

10. The coefficient of variations for projects A and B areA. 0.6 and 1, respectively. C. 0.8 and 2, respectively.B. 1.6 and 1, respectively. D. 1.2 and 1.5, respectively.

11. The two projects can best be characterized relative to one another by the statement, A. project A is more risky than project B.B. project B is more risky than project A.C. since project A has a higher expected net present value, it should be chosen.D. since project B has a higher standard deviation, it is more risky and should not be chosen.

12. The firm should A. choose project A since it has a higher net present value potential.B. choose project B since it has a lower standard deviation.C. choose project A since it has a lower relative risk.D. choose project B since it has a lower relative risk.

Questions 23 through 25 are based on the following information. RPCPA 0579The Suarez Corporation has the following incremental cash flows for a press machine it intends to buy:

Year 1 Year 2Cash Flow Initial Probability Cash Flow Initial Probability

P5,000 0.5 P 3,000 0.3 4,500 0.3 6,000 0.4

P10,000 0.5 P10,000 0.25 12,500 0.40 15,000 0.35

The firm has a required after tax rate of return of 10%. The abandonment value of the machine is as follows:

Year 1 Year 2Probability Amount Probability Amount

0.5 P8,000 0.5 P2,0000.3 5,000 0.3 1,5000.2 4,000 0.2 1,000

*. Based on the above data, what is the joint probability of the cash flow series that will yield P10,000 for the first year and P12,500 for the second year? (D)a. 0.125 c. 0.175b. 0.150 d. 0.200

*. The abandonment value for year 1 is (D)a. P5,000 c. P7,500b. P6,300 d. P10,000

*. The abandonment value for year 2 is (D)a. P1,650 c. P25,500b. P13,500 d. P37,500

COMPREHENSIVECash Flows & Discounted Cash FlowsQuestions 91 through 94 are based on the following information. CMA 1296 4-15 to 18In order to increase production capacity, Gunning Industries is considering replacing an existing production machine with a new technologically improved machine effective January 1, 1997. The following information is being considered by Gunning Industries: The new machine would be purchased for $160,000 in cash. Shipping, installation, and testing

would cost an additional $30,000. The new machine is expected to increase an annual sales by $20,000 at a sales price of $40

per unit. Incremental operating costs include $30 per unit in variable costs and total fixed costs of $40,000 per year.

The investment in the new machine will require an immediate increase in working capital of $35,000. This cash outflow will be recovered at the end of year 5.

Gunning uses straight-line depreciation for financial reporting and tax reporting purposes. The new machine has an estimated useful life of 5 years and zero salvage value.

CMA EXAMINATION QUESTIONS Page 55 of 155

Page 56: P02 - Capital Budgeting

MANAGEMENT ADVISORY SERVICES CAPITAL BUDGETING

Gunning is subject to a 40% corporate income tax rate.Gunning uses the net present value method to analyze investments and will employ the following factors and rates:

PeriodPresent Valueof $1 at 10%

Present Value of anOrdinary Annuity of $1 at 10%

1 0.909 0.9092 0.826 1.7363 0.751 2.4874 0.683 3.1705 0.621 3.791

145. Gunning Industries’ net cash outflow in a capital budgeting decision isa. $190,000 c. P204,525b. P195,000 d. P225,000

146. Gunning Industries’ discounted annual depreciation tax shield for the year 1997 isa. $13,817 c. $20,725b. $16,762 d. $22,800

147. The acquisition of the new production machine by Gunning Industries will contribute a discounted net-of-tax contribution margin ofa. $242,624 c. $363,936b. $303,280 d. $454,920

148. The overall discounted cash flow impact of Gunning Industries’ working capital investment for the new production machine would bea. $(7,959) c. $(13,265)b. $(10,080) d. $(35,000)

Net Investment & Payback PeriodQuestions 19 and 20 are based on the following information. RPCPA 0583Beta Company plans to replace its company car with a new one. The new car costs P120,000 and its estimated useful life is five years without scrap value. The old car has a book value of P15,000 and can be sold at P12,000. The acquisition of the new car will yield annual cash savings of P20,000 before income tax. Income tax rate is 25%. *. The net investment of the new car is (E)

a. P108,000 c. P107,250b. P108,750 d. P107,000

*. The payback period of the investment is (M)a. 5.14 years c. 5.11 years

b. 5.18 years d. 5.095 years

Payback & Net Present ValueQuestions 1 & 2 are based on the following information. H & MRandall Corp is considering the purchase of a new machine for $38,000. The machine would generate a net cash inflow of $11,607 per year for five years. At the end of five years, the machine would have no salvage value. The company’s cost of capital is 12 percent. The company uses straight-line method of depreciation with no mid-year convention.

149. What is the payback period in years for the machine approximated to two decimal points, assuming no taxes are paid?a. 3.00 c. 3.27b. 9.48 d. 4.00

150. What is the net present value for the machine, assuming no taxes are paid?a. $3,843 c. $38,000b. $0 d. $(92,750)

Questions 1 and 2 are based on the following information. H & MJD, Inc. is considering the purchase of production equipment that costs $400,000. The equipment is expected to generate annual cash inflows of $125,000 and have a useful life of 5 years with no salvage value. The firm’s cost of capital is 12 percent. The company uses the straight-line method of depreciation with no mid-year convention. There are no income taxes.

151. The payback period in years for the project is a. 2.90 years c. 3.25 yearsb. 3.20 years d. 4.20 years

152. Ignoring income taxes, the net present value of the project is a. $40,480 c. $50,625b. $48,625 d. $54,450

Questions 1 & 2 are based on the following information.Rogers Manufacturing Company is considering the following investment proposal:

Original investment $30,000Operations (per year for 4 years): Cash receipts $20,000 Cash expenditures 11,000Salvage value of equipment after 4 years $ 2,000Discount rate 10 percent

The firm uses the straight-line method of depreciation with no mid-year convention.

CMA EXAMINATION QUESTIONS Page 56 of 155

Page 57: P02 - Capital Budgeting

MANAGEMENT ADVISORY SERVICES CAPITAL BUDGETING

153. What is the payback period in years approximated to two decimal points assuming no taxes are paid?a. 3.33 d. 3.78b. 1.50 e. 4.00c. 1.70 H & M

154. What is the net present value for the investment, assuming no taxes are paid?a. $(1,000) d. $29,000b. $30,000 e. $(104)c. $29,896 H & M

Questions 79 and 80 are based on the following information. Pol BobadillaRisk Insurance Company’s management is considering an advertising program that would require an initial expenditure of P165,500 and bring in additional sales over the next five years. The cost of advertising is immediately recognized as expense. The projected additional sales revenue in Year 1 is P75,000, with associated expenses of P25,000. The additional sales revenue and expenses from the advertising program are projected to increase by 10% each year. Risk Insurance Company’s tax rate is 40%.The present value of 1 at 10%, end of each period:

Periods Present Value Factor1 0.909092 0.826453 0.751314 0.683015 0.62092

79. The payback period for the advertising program isA. 4.6 years. C. 3.9 yearsB. 3.0 years D. 2.5 years

80. The net present value of the advertising program would beA. P37,064 C. P(37,064)B. P29,136 D. P(29,136)

Net Income after Tax, Annual Cash Flow & Payback PeriodQuestions 15 through 17 are based on the following information. RPCPA 1083Plaridel Electronics is considering the feasibility of producing a new product to add to its existing line. The sales and cost estimates for this new product are as follows:

Annual sales 22,500 unitsSelling price P35 per unit

Variable manufacturing costs P21 per unitIncremental fixed manufacturing costs (depreciation not included) P37,500Incremental selling and administrative expenses P45,000

To produce the new product, the company must purchase an additional equipment costing P300,000. Since the product will have a life cycle of only five years, the equipment will also have a service life of 5 years with no salvage value. The straight-line method of depreciation is used. Cash basis for all revenues and expenses other than depreciation is assumed. Applicable income tax rate is 35%*. The estimated annual net income after taxes from the proposal to produce the new product is

(E)a. P86,250 c. P151,125b. P112,125 d. P172,500

*. The estimated net annual cash flow expected from the same proposal is (E)a. P112,125 c. P249,625b. P172,125 d. P232,500

*. The payback period of this same proposal is (E)a. 1.74 years c. 3.48 yearsb. 2.68 years d. 5 years

Payback & NPVQuestions 79 and 80 are based on the following information. BarfieldBlues Bros. Inc. is considering an investment in a computer that is capable of producing various images that are useful in the production of commercial art. The computer would cost $20,000 and have an expected life of eight years. The computer is expected to generate additional annual net cash receipts (before-tax) of $6,000 per year. The computer will be depreciated according to the straight-line method and the firm's marginal tax rate is 25 percent.

79. What is the after-tax payback period for the computer project?a. 7.62 years c. 4.44 yearsb. 3.90 years d. 3.11 years

80. (Present value tables needed to answer this question.) What is the after-tax net present value of the proposed project (using a 16 percent discount rate)?a. $2,261 c. $6,062b. $(454) d. $(4,797)

Questions 1 & 2 are based on the following information.Hancock Corporation is considering an investment in equipment for $30,000. Data related to the investment are as follows:

CMA EXAMINATION QUESTIONS Page 57 of 155

Page 58: P02 - Capital Budgeting

MANAGEMENT ADVISORY SERVICES CAPITAL BUDGETING

Year Income Before Depreciation and Taxes1 $20,0002 20,0003 20,0004 20,0005 20,000

Cost of capital is 18 percent.Hancock uses the straight-line method of depreciation with no mid-year convention. In addition, their tax rate is 40 percent and the life of the equipment is five years with no salvage value.

155. What is the payback period in years approximated to two decimal points?a. 1.00 d. 1.50b. 0.67 e. 2.08c. 0.48 H & M

156. What is the net present value of the investment?a. $45,029 d. $7,524b. $(15,029) e. $15,029c. $32,540 H & M

Questions 84 & 85 are based on the following information. BarfieldL&M Ironworks is considering a proposal to sell an existing lathe and purchase a new computer-operated lathe. Information on the existing lathe and the computer-operated lathe follow:

Existing Lathe Computer-operated latheCost $100,000 $300,000Accumulated depreciation 60,000 0Salvage value now 20,000

Salvage value in 4 years 0 60,000Annual depreciation 10,000 75,000Annual cash operating costs 200,000 50,000Remaining useful life 4 years 4 years

84. What is the payback period for the computer-operated lathe?a. 1.87 years c. 3.53 yearsb. 2.00 years d. 3.29 years

85. (Present value tables needed to answer this question.) If the company uses 10 percent as its discount rate, what is the net present value of the proposed new lathe purchase?a. $236,465 c. $195,485b. $256,465 d. $30,422

Questions 1 & 2 are based on the following information. H & MMarriott Services is considering an investment of $50,000. Data related to the investment are as follows:

Year Cash Inflows1 $20,0002 22,0003 16,0004 30,0005 30,000

Cost of capital is 14 percent.

157. What is the payback period in years approximated to two decimal points, assuming no taxes are paid?a. 2.12 c. 2.50b. 4.00 d. 3.00

158. What is the net present value of the investment, assuming no taxes are paid?a. $29,650 c. $50,000b. $78,588 d. $28,588

Net Investment and NPVProblem 15 and 16 are based on the following information. RPCPA 1086Daneche’s, a tax-exempt entity, plans to purchase a new machine which they project to depreciate over a ten-year period without salvage value. The new machine will cost P200,000 and is expected to generate cash savings of P60,000 per year in operating costs. Daneche's cost of capital is 12%.

CMA EXAMINATION QUESTIONS Page 58 of 155

Page 59: P02 - Capital Budgeting

MANAGEMENT ADVISORY SERVICES CAPITAL BUDGETING

For ten periods at 12%, the present value of P1 is P0.3220, while the present value of an ordinary annuity of P1 is P5.650.

*. What is the net present value of the proposed investment, assuming Daneche uses a 12% discount rate? (M)a. P185,640 c. P139,000b. P69,980 d. None of the above.

*. With the company’s initial investment on the new machine, the accounting rate of return is (M)a. 15% c. 25%b. 20% d. None of the above.

Questions 11 and 12 are based on the following information. RPCPA 1079Andalucia Machine Corporation invested in machine with a useful life of six years and no salvage value. The machine was depreciated using the straight-line method. The annual cash inflow from operations, net of income taxes was P2,000. The present value of an ordinary annuity of P1 in arrears for six years at 14% is 3.889. The present value of P1 for six periods at 14% is 0.456.*. Assuming that Andalucia used a time adjusted rate of return of 14%, what was the amount of

the original investment? (E)a. P7,778 c. P8,687b. P8,235 d. P9,048

*. Assuming that the minimum desired rate of return on this investment was 14% and the amount of the original investment was P7,500, what would be the net present value to the nearest peso? (E)a. P0 c. P278b. P196 d. P345

Net Investment, Annual Cash Flow & NPVQuestion 13 through 15 are based on the following information. RPCPA 1089A proposal to management of Celine Company has been made to effect the replacement of an old machine with a modern one, yielding a higher production capacity. The following relevant information was given for evaluation:

Net book value of the old machine P 23,000Resale value of the old machine 30,000Cost of the new machine 300,000Annual cash savings before tax, due to increase in efficiency 85,000Income tax rate 35%Estimated useful life of the new machine with a salvage value of P3,000 10 yearsMinimum desired rate of return on this investment 20%

The Company uses the straight-line method of depreciation. Information on present value factors follows:

Present value of P1.00 for 10 years at 20% 0.162Present value of ordinary annuity of P1.00 in arrears for 10 years 4.192

*. What would be the annual cash flow net of tax?

a. P65,645 c. P55,000b. P55,300 d. P67,750

*. What would be the net investment after income tax?a. P297,550 c. P270,000b. P272,450 d. P250,645

*. What would be the net present value of the investment? (rounded to the nearest peso)a. P2,734 c. P3,220b. P3,660 d. P(40,146)

Numbers 10, 11 and 12 are based on the following data. RPCPA 1088The management of PAMA, INC. is planning to replace an old machinery which has a net book value of P15,000 and a resale value of P20,000. The new machinery will cost P200,000 with an estimated useful life of 10 years and salvage value of P5,000. The expected increase in efficiency will result in annual cash savings of P55,000 before tax. The company is using the straight-line method of depreciation and income tax rate is 35%. The minimum desired rate of return on this investment is 18%.Present value of ordinary annuity of P1.00 in arrears for 10 years at 18% is 4.494.Present value of P1.00 for 10 years at 18% is 0.191.

*. The net investment after income tax isa. P180,000 c. P196,750b. P185,000 d. P181,750

*. The annual cash flow net of tax isa. P35,750 c. P35,500b. P42,750 d. P42,575

*. The net present value of the investment isa. (P7,712.95) c. P10,537.05b. P9,582.05 d. P12,287.05

Net Investment, Payback Period, NPV & IRRQuestions 17 through 20 are based on the following information. RPCPA 1084

CMA EXAMINATION QUESTIONS Page 59 of 155

Page 60: P02 - Capital Budgeting

MANAGEMENT ADVISORY SERVICES CAPITAL BUDGETING

Sta. Lucia Company is considering the purchase of a new equipment at a price of P400,000. If the new equipment is acquired, an old machine that is fully depreciated will be disposed for P50,000.The new equipment is expected to provide annual savings in operating costs of P150,000 before deducting depreciation or income tax. The new equipment has an estimated life of 5 years and will have no salvage value at the end of 5 years. Income tax is at the rate of 35% for both the ordinary income and on any gain from the sale of equipment. For similar type of investment, the company expects a minimum rate of 20%.The appropriate present value table that you may use in this problem is as follows:

Present value of P1 Present value of P1 received per period

Period

20% 22% 20% 22%

1 0.833 0.820 0.833 0.8202 0.694 0.672 1.528 1.4923 0.579 0.551 2.106 2.0424 0.482 0.451 2.589 2.4945 0.402 0.370 2.991 2.864

*. The amount of net investment in the new equipment isa. P350,000 c. P400,000b. P367,500 d. P450,000

*. The payback period using returns before depreciation and taxes isa. 2.45 years. c. 3.0 years.b. 2.67 years. d. 4.0 years.

*. The net present value of the investment of the new equipment isa. zero c. P25,870.00b. P7,870.50 d. P(24,630)

*. The discounted rate of return on the new equipment isa. 20% d. 22%b. less than 20% e. Answer not givenc. more than 20% but less than 22%

Annual Net Cash Inflow & PaybackQuestions 1 & 2 are based on the following information. H & MFinch Company is considering an investment in equipment for $30,000. Finch uses the straight-line method of depreciation with no mid-year convention. In addition, their tax rate is 40 percent and the life of the equipment is five years with no salvage value. The expected income before depreciation and taxes is projected to be $15,000 per year.

159. What is the payback period in years approximated to two decimal points?a. 1.00 d. 4.00b. 2.00 e. 0.38c. 2.63

160. What is the annual net cash inflow for year 1?a. $15,000 c. $11,400b. $9,000 d. $6,000

CMA EXAMINATION QUESTIONS Page 60 of 155

Page 61: P02 - Capital Budgeting

MANAGEMENT ADVISORY SERVICES CAPITAL BUDGETING

Annual Cash Flow, NPV & IRRQuestions 75 thru 77 are based on the following information. G & N 9e (Ignore income taxes in this problem.) Bugle's Bagel Bakery is investigating the purchase of a new bagel making machine. This machine would provide an annual operating cost savings of $3,650 for each of the next 4 years. In addition, this new machine would allow the production of one new type of bagel that would result in selling 1,500 dozen more bagels each year. The company earns a contribution margin of $0.90 on each dozen bagels sold. The purchase price of this machine is $13,450 and it will have a 4-year useful life. Bugle's discount rate is 14%.

75. The total annual cash inflow from this machine for capital budgeting purposes is: (E)a. $3,650. c. $4,750.b. $5,150. d. $5,000.

76. The internal rate of return for this investment is closest to: (M)a. 14%. c. 18%.b. 16%. d. 20%.

77. The net present value of this investment is closest to: (E)a. $1,120. c. $13,450.b. $6,550. d. $20,000.

ARR and PaybackQuestions 16 and 17 are based on the following information. RPCPA 1080A manufacturing firm is studying the purchase of a P50,000 equipment with an estimated life of five years. The new equipment is expected to give the company net cash income before income taxes of P15,000 a year. The equipment shall be depreciated on a straight-line basis with no salvage value. Assume that the income tax rate is 40%.*. The accounting (book value) rate of return on the initial increase in required investment is (E)

a. 6% c. 18%b. 10% d. 30%

*. The payback period is (E)a. 2 years c. 3.85 yearsb. 3.33 years d. 10 years

Questions 101 and 102 are based on the following information. BarfieldJimmy's Retail is considering an investment in a delivery truck. Jimmy has found a used truck that he can purchase for $8,000. He estimates the truck would last six years and increase his store's net cash revenues by $2,000 per year. At the end of six years, the truck would have no salvage value and would be discarded. Jimmy will depreciate the truck using the straight-line method.

101.What is the accounting rate of return on the truck investment (based on average profit and average investment)?a. 25.0% c. 16.7%b. 50.0% d. 8.3%

102.What is the payback period on the investment in the new truck?a. 12 years c. 4 yearsb. 6 years d. 2 years

Questions 99 & 100 are based on the following information. G & N 9e (Ignore income taxes in this problem.) Fast Food, Inc., has purchased a new donut maker. It cost $16,000 and has an estimated life of 10 years. The following annual donut sales and expenses are projected:

Sales $22,000Expenses:

Flour, etc., required in making donuts $10,000Salaries 6,000Depreciation 1,600 17,600

Net income $ 4,400

99. The payback period on the new machine is closest to:(E)a. 5 years. c. 3.6 years.b. 2.7 years. d. 1.4 years.

100.The simple rate of return for the new machine is closest to: (E)a. 20%. c. 27.5%.b. 37.5%. d. 80.0%.

Questions 101 & 102 are based on the following information. G & N 9e (Ignore income taxes in this problem.) Purvell Company has just acquired a new machine. Data on the machine follow:

Purchase cost $50,000Annual cost savings 15,000Life of the machine 8 years

The company uses straight-line depreciation and a $5,000 salvage value. (The company considers salvage value in making depreciation deductions.) Assume cash flows occur uniformly throughout a year.

101.The payback period would be closest to: (E)a. 3.33 years. c. 8.0 years.b. 3.0 years. d. 2.9 years.

CMA EXAMINATION QUESTIONS Page 61 of 155

Page 62: P02 - Capital Budgeting

MANAGEMENT ADVISORY SERVICES CAPITAL BUDGETING

102.The simple rate of return would be closest to: (E)a. 30.0%. c. 18.75%.b. 17.5%. d. 12.5%.

Questions 103 & 104 are based on the following information. AICPA adapted(Ignore income taxes in this problem.) Hanley Company purchased a machine for $125,000 that will be depreciated on the straight-line basis over a five-year period with no salvage value. The related cash flow from operations is expected to be $45,000 a year. These cash flows from operations occur uniformly throughout the year.

103.What is the payback period? (E)a. 2.1 years. c. 2.8 years.b. 2.3 years. d. 4.2 years.

104.What is the simple rate of return on the initial investment? (E)a. 16%. c. 28%.b. 24%. d. 36%.

ARR & NPVQuestions 8 and 9 are based on the following information. RPCPA 0585Information concerning two products or the Champaca Company is given below:Catleya Company, a tax-exempt entity, is planning to purchase a new machine which it will depreciate on a straight-line basis over a ten-year period with no salvage value. The new equipment costing P150,000 is expected to produce cash savings of P33,000 per year in operating costs. Catleya’s cost of capital is 16%. For ten periods at 16%, the present value of P1 is 0.227, while the present value of an annuity of P1 is P4.833.*. Assuming that Catleya uses a discount rate of 15%, the net present value of the proposed

investment is:a. P9,489 c. P75,000b. P34,050 d. P180,000

*. Based on the Company’s initial investment in the new equipment, the accounting rate of return is a. 8% c. 16%b. 12% d. 25%

Questions 1 & 2 are based on the following information.8. Johnson Company invests in a new place of equipment costing $20,000. The equipment is

expected to yield the following amounts per year for the equipment’s four-year useful life:

Cash revenues $30,000Cash expenses (16,000)Depreciation expenses ( 5,000)Income provided from equipment $ 9,000Cost of capital 14 percent

There is no salvage value at the end of four years.161. What is the accounting rate of return on average investment for the equipment, assuming no

taxes are paid?a. 100.0% d. 90.0%b. 6.3% e. 14.0%c. 70.0% H & M

162. What is the net present value of this investment in equipment assuming no taxes are paid?a. $40,796 d. $26,226b. $20,796 e. $17,760c. $(2,240) H & M

ARR & IRRItems 74 thru 76 are based on the following: Tam Co. is negotiating for the purchase of equipment that would cost $100,000, with the expectation that $20,000 per year could be saved in after-tax cash costs if the equipment were acquired. The equipment’s estimated useful life is 10 years, with no residual value, and would be depreciated by the straight-line method. Tam’s predetermined minimum desired rate of return is 12%. Present value of an annuity of 1 at 12% for 10 periods is 5.65. Present value of 1 due in 10 periods at 12% is 0.322.

163. The payback period is (E)a. 4.0 years. c. 4.5 years.b. 4.4 years. d. 5.0 years.

164. In estimating the internal rate of return, the factors in the table of present values of an annuity should be taken from the columns closest toa. 0.65 c. 5.00b. 1.30 d. 5.65

165. Accrual accounting rate of return based on initial investment isa. 30% c. 12%b. 20% d. 10%

ARR, Payback and NPVQuestions 1 through 3 are based on the following information.

CMA EXAMINATION QUESTIONS Page 62 of 155

Page 63: P02 - Capital Budgeting

MANAGEMENT ADVISORY SERVICES CAPITAL BUDGETING

A company is planning to acquire a new machine costing $120,000. It has an economic useful life of 3 years with no salvage value. It has a theoretical capacity of 100,000 machine hours. It can produce one unit per hour. It expects to sell 80,000 units per year. The machine will reduce labor costs from $2.50 to $1.50 per machine hour.

166. What is the net present value of this investment? Use the factors given below.Years PV $1 at 10%

1 0.912 0.833 0.754 0.685 0.626 0.56

A. $79,200 C. $129,000B. $120,000 D. $199,200 CIA 0589 IV-22

167. What is the accounting (book) rate of return on the average investment for the 3-year period? A. 10% C. 67%B. 33% D. 80% CIA 0589 IV-23

168. What is the payback period? A. 1.50 years. C. 2.00 years.B. 1.20 years. D. 3.00 years. CIA 0589 IV-24

Questions 83 through 85 are based on the following information. CMA 0696 4-23 to 25Jorelle Company’s financial staff has been requested to review a proposed investment in new capital equipment. Applicable financial data is presented below. There will be no salvage value at the end of the investment’s life and, due to realistic depreciation practices, it is estimated that the salvage value and net book value are equal at the end of each year. All cash flows are assumed to take place at the end of each year. For investment proposal, Jorelle uses a 12% after-tax target rate of return.

Investment Proposal

YearPurchase Costand Book Value

Annual Net After-Tax Cash Flows

Annual Net Income

0 $250,000 $ 0 . $ 0 .1 168,000 120,000 35,0002 100,000 108,000 39,0003 50,000 96,000 43,0004 18,000 84,000 47,0005 0 72,000 51,000

Year P.V. of $1 Receivedat the End of Each Period

P.V. of an Annuity of $1.00 Receivedat the End of Each Period

1 0.89 0.892 0.80 1.693 0.71 2.404 0.64 3.045 0.57 3.616 0.51 4.12

169. The accounting rate of return for the investment proposal isa. 12.0% c. 28.0%b. 17.2% d. 34.4%

170. The net present value for the investment proposal isa. $106,160 c. $356,160b. $(97,970) d. $96,560

171. The traditional payback period for the investment proposal isa. Over 5 years. c. 1.65 years.b. 2.23 years. d. 2.83 years

Questions 86 through 88 are based on the following information. CIA 0593 IV-22 to 24A company purchased a new machine to stamp the company logo on its products. The cost of the machine was $250,000, and it has an estimated useful life of 5 years with an expected salvage value at the end of its useful life of $50,000. The company uses the straight-line depreciation method.The new machine is expected to save $125,000 annually in operating costs. The company’s tax rate is 40%, and it uses a 10% discount rate to evaluate capital expenditure.

Year Present Value of $1 Present Value of an Ordinary Annuity of $11 .909 .9092 .826 1.7363 .751 2.4874 .683 3.1705 .621 3.791

172. What is the traditional payback period for the new stamping machine?a. 2.00 years. c. 2.75 years.b. 2.63 years. d. 2.94 years.

173. What is the accounting rate of return based on the average investment in the new stamping machine?

CMA EXAMINATION QUESTIONS Page 63 of 155

Page 64: P02 - Capital Budgeting

MANAGEMENT ADVISORY SERVICES CAPITAL BUDGETING

a. 20.4% c. 40.8%b. 34.0% d. 51.0%

174. What is the net present value (NPV) of the new stamping machine?a. $125,940 c. $250,000b. $200,000 d. $375,940

Questions 122 through 125 are based on the following information. CMA 1291 4-1 to 4Yipann Corporation is reviewing an investment proposal. The initial cost as well as other related data for each year are presented in the schedule below. All cash flows are assumed to take place at the end of the year. The salvage value of the investment at the end of each year is equal to its net book value, and there will be no salvage value at the end of the investment's life.

Investment Proposal

YearInitial Cost

and Book ValueAnnual Net After-Tax Cash Flows

AnnualNet Income

0 $105,000 $0 $01 70,000 50,000 15,0002 42,000 45,000 17,0003 21,000 40,000 19,0004 7,000 35,000 21,0005 0 30,000 23,000

Yipann uses a 24% after-tax target rate of return for new investment proposals. The discount figures for a 24% rate of return are given.

YearPresent Value of $1.00

Received at the End of PeriodPresent Value of an Annuity of

$1.00Received at the End of Each Period1 .81 .812 .65 1.463 .52 1.984 .42 2.405 .34 2.746 .28 3.027 .22 3.24

175. The traditional payback period for the investment proposal is (E)A. .875 years. C. 2.250 years.B. 1.933 years. D. Over 5 years.

176. The average annual cash inflow at which Yipann would be indifferent to the investment (rounded to the nearest dollar) is (E)

A. $21,000. C. $38,321.B. $30,000. D. $46,667.

177. The accounting rate of return for the investment proposal over its life using the initial value of the investment is (E)A. 36.2%. C. 28.1%.B. 18.1%. D. 38.1%.

178. The net present value of the investment proposal is (E)A. $4,600. C. $(55,280).B. $10,450. D. $115,450.

Questions 80 thru 82 are based on the following information. G & N 9e (Ignore income taxes in this problem.) Oriental Company has gathered the following data on a proposed investment project:

Investment in depreciable equipment ..... $200,000Annual net cash flows ................... $ 50,000Life of the equipment ................... 10 yearsSalvage value ........................... -0-Discount rate ........................... 10%

The company uses straight-line depreciation on all equipment.

80. The payback period for the investment would be: (E)a. 2.41 years. c. 10 years.b. 0.25 years. d. 4 years.

81. The simple rate of return on the investment would be: (E)a. 10%. c. 15%.b. 35%. d. 25%.

82. The net present value of this investment would be: (E)a. ($14,350). c. $77,200.b. $107,250. d. $200,000.

ARR, Discounted Payback & NPVQuestions 68 through 70 are based on the following information. GleimRex Company is considering an investment in a new plant which will entail an immediate capital expenditure of $4,000,000. The plant is to be depreciated on a straight-line basis over 10 years to zero salvage value. Operating income (before depreciation and taxes) is expected to be $800,000 per year over the 10-year life of the plant. The opportunity cost of capital is 14%. Assume that there are no taxes.

CMA EXAMINATION QUESTIONS Page 64 of 155

Page 65: P02 - Capital Budgeting

MANAGEMENT ADVISORY SERVICES CAPITAL BUDGETING

179. What is the book (or accounting) rate of return for the investment? A. 10% C. 28%B. 20% D. 35%

180. What is the discounted payback period for the investment? A. 5.5 years. C. 9.2 years.B. 7.1 years. D. 11.7 years.

181. What is the NPV for the investment? A. $172,800 C. $312,475B. $266,667 D. $428,956

ARR, NPV & IRRQuestions 1 thru 3 are based on the following information. H & MOrow Print Shop is considering the purchase of a used printing press costing $19,200. The printing press would generate a net cash inflow of $8,000 per year for three years. At the end of three years, the press would have no salvage value. The company’s cost of capital is 10 percent. The company uses straight-line depreciation with no mid-year convention.

182. What is the accounting rate of return on the original investment in the press to the nearest percent, assuming no taxes are paid?a. 41.67% d. 10.00%b. 8.33% e. 12.00%c. 75.00%

183. What is the net present value for the press, assuming no taxes are paid?a. $4,800 d. $696b. $19,896 e. $0c. $19,200

184. What is the internal rate of return to the nearest percent for the press, assuming no taxes are paid?a. 10% d. 8%b. 12% e. 14%c. 42%

ARR, NPV, IRR & Annual Cash FlowQuestions 1 thru 4 are based on the following information. H & MRandall Corp is considering the purchase of a new machine for $40,000. The machine would generate a net cash inflow before depreciation and taxes of $14,389 per year for five years. At the

end of five years, the machine would have no salvage value. The company’s cost of capital is 12 percent. The company uses straight-line method of depreciation with no mid-year convention and has a 40 percent tax rate.

185. What is the accounting rate of return on the original investment in the machine approximated to two decimal points?a. 35.97% d. 9.58%b. 19.17% e. 0.00%c. 15.97%

186. What is the net present value for the machine?a. $2,659 c. $42,659b. $0 d. $11,872

187. What is the internal rate of return for the machine rounded to the nearest percent?a. below 12 percent d. between 16 and 18 percentb. between 12 and 14 percent e. above 18 percentc. between 14 and 16 percent

188. What is the annual net after-tax cash inflow per year (rounded off)?a. $14,389 c. $3,200b. $8,633 d. $11,833

Questions 1 thru 4 are based on the following information. H & MHarrison Company is considering the purchase of a new machine for $80,000. The machine would generate net cash inflow before depreciation and taxes of $31,294 per year for four years. At the end of four years, the machine would have no salvage value. The company’s cost of capital is 12 percent. The company uses straight-line depreciation with no mid-year convention and has a 40 percent tax rate.

189. What is the accounting rate of return on the original investment in the machine approximated to two decimal points?a. 14.12% c. 39.12%b. 8.47% d. 16.92%

190. What is the net present value for the machine?a. $81,320 c. $15,040b. $1,320 d. $(22,976)

191. What is the internal rate of return for the machine rounded to the nearest percent?a. below 12 percent d. between 16 and 18 percent

CMA EXAMINATION QUESTIONS Page 65 of 155

Page 66: P02 - Capital Budgeting

MANAGEMENT ADVISORY SERVICES CAPITAL BUDGETING

b. between 12 and 14 percent e. above 18 percentc. between 14 and 16 percent

192. What is the annual net after-tax cash inflow per year?a. $31,294 d. $29,667b. $8,000 e. $26,76c. $18,776

ARR, Payback, NPV & IRRQuestions 118 through 121 are based on the following information. GleimDon Adams Breweries is considering an expansion project with an investment of $1,500,000. The equipment will be depreciated to zero salvage value on a straight-line basis over 5 years. The expansion will produce incremental operating revenue of $400,000 annually for 5 years. The company's opportunity cost of capital is 12%. Ignore taxes.

193. What is the payback period of the project? A. 2 years. C. 3.75 years.B. 2.14 years. D. 5 years.

194. What is the book (accounting) rate of return of the investment? A. 6.67% C. 16.67%B. 13.33% D. 26.67%

195. What is the NPV of the investment? A. $0 C. - $116,000B. - $58,000 D. $1,442,000

196. What is the IRR of the investment? A. 10.43% C. 16.32%B. 12.68% D. 19.17%

Questions 71 thru 74 are based on the following information. G & N 9e(Ignore income taxes in this problem.) Shields Company has gathered the following data on a proposed investment project:

Investment required in equipment $400,000Annual cash inflows $80,000Salvage value $-0-Life of the investment 10 yearsDiscount rate 10%

71. The payback period for the investment is closest to: (E)

a. 0.2 years. c. 3.0 years.b. 1.0 years. d. 5.0 years.

72. The simple rate of return on the investment is closest to: (E)a. 5%. c. 15%.b. 10%. d. 20%.

73. The net present value on this investment is closest to: (E)a. $400,000. c. $91,600.b. $80,000. d. $76,750.

74. The internal rate of return on the investment is closest to: (M)a. 11%. c. 15%.b. 13%. d. 17%.

ARR, Payback, NPV, PI & IRRQuestions 106 through 110 are based on the following information. GleimA proposed investment is not expected to have any salvage value at the end of its 5-year life. For present value purposes, cash flows are assumed to occur at the end of each year. The company uses a 12% after-tax target rate of return.

YearPurchase Cost and Book Value

Annual Net After-Tax Cash Flows

Annual Net Income

0 $500,000 $ 0 $ 01 336,000 240,000 70,0002 200,000 216,000 78,0003 100,000 192,000 86,0004 36,000 168,000 94,0005 0 144,000 102,000

Discount Factors for a 12% Rate of Return

YearPresent Value of $1 at

The End of Each PeriodPresent Value of an Annuity of$1 at the End of Each Period

1 .89 .892 .80 1.693 .71 2.404 .64 3.045 .57 3.616 .51 4.12

197. The accounting rate of return based on the average investment is (E)a. 84.9% c. 40.8%

CMA EXAMINATION QUESTIONS Page 66 of 155

Page 67: P02 - Capital Budgeting

MANAGEMENT ADVISORY SERVICES CAPITAL BUDGETING

b. 34.4% d. 12%

198. The net present value is (E)a. $304,060 c. $(70,000)b. $212,320 d. $712,320

199. The traditional payback period is (E)a. Over 5 years. c. 1.65 years.b. 2.23 years. d. 2.83 years.

200. The profitability index is (E)a. 0.61 c. 0.86b. 0.42 d. 1.425

201. Which statement about the internal rate of return of the investment is true? (E)a. The IRR is exactly 12%.b. The IRR is over 12%.c. The IRR is under 12%.d. No information about the IRR can be determined

Discount Rate, Profitability Index, IRRQuestions 88 through 90 are based on the following information. BarfieldR Co. is involved in the evaluation of a new computer-integrated manufacturing system. The system has a projected initial cost of $1,000,000. It has an expected life of six years, with no salvage value, and is expected to generate annual cost savings of $250,000. Based on R Co.'s analysis, the project has a net present value of $57,625.

88. (Present value tables needed to answer this question.) What discount rate did the company use to compute the net present value?a. 10% c. 12%b. 11% d. 13%

89. What is the project's profitability index?a. 1.058 c. .945b. .058 d. 1.000

90. (Present value tables needed to answer this question.) What is the project's internal rate of return?a. between 12.5 and 13.0 percent c. between 11.5 and 12.0 percentb. between 11.0 and 11.5 percent d. between 13.0 and 13.5 percent

Payback Period and NPVQuestions 13 and 14 are based on the following information. RPCPA 1082The Sta. Clara Company is planning to replace one of its machines. The annual operating cost of the machine is P150,000, excluding depreciation, while that of the new machine is estimated at P90,000. The new machine will cost P180,000 net of trade-in allowance with a useful life of 10 years, without salvage value. Assume cost of capital to be 12% and a straight-line depreciation charge. The old machine is fully depreciated. The present value of an annuity of P1.00 for 10 years at 12% is 5.6502. Ignore income tax implications.*. The payback period will be (E)

a. 2.5 years c. 4.29 yearsb. 3 years d. 4.5 years

*. The net present value is (E)a. P57,308.40 d. P237,308.40b. P90,000 e. None of the thesec. P180,000

Questions 66 and 67 are based on the following information. CMA 1293 4-18 & 19The Keego Company is planning a $200,000 equipment investment which has an estimated 5-year life with no estimated salvage value. The company has projected the following annual cash flows for the investment.

Year Projected Cash Inflows Present Value of $11 $120,000 .912 60,000 .763 40,000 .634 40,000 .535 40,000 .44

Totals $300,000 3.27

202. Assuming that the estimated cash inflows occur evenly during each year, the payback period for the investment isa. 1.67 years. c. 2.50 years.b. 4.91 years. d. 1.96 years.

203. The net present value for the investment is A. $18,800. C. $196,200.B. $218,800. D. $91,743.

Questions 76 and 77 are based on the following information. CMA 695 4-5 & 6McLean Inc. is considering the purchase of a new machine that will cost $160,000. The machine has an estimated useful life of 3 years. Assume that 30% of the depreciable value base will be

CMA EXAMINATION QUESTIONS Page 67 of 155

Page 68: P02 - Capital Budgeting

MANAGEMENT ADVISORY SERVICES CAPITAL BUDGETING

depreciated in the first year, 40% in the second year, and 30% in the third year. The new machine will have a $10,000 resale value at the end of its estimated useful life. The machine is expected to save the company $85,000 per year in operating expenses. McLean uses a 40% estimated income tax rate and a 16% hurdle rate to evaluate capital projects.Discount rates for a 16% rate are as follows:

Present Value of $1 Present Value of an Ordinary Annuity of $1Year 1 .862 .862Year 2 .743 1.605

Year 3 .641 2.246

204. What is the net present value of this project?a. $3,278 c. $(568)b. $5,842 d. $30,910

205. The payback period for this investment would bea. 1.88 years. d. 2.23 years.b. 3.00 years. e. 1.62 years.c. 2.23 years.

Questions 78 and 79 are based on the following information. CMA 1295 4-12 & 13Willis Inc. has a cost of capital of 15% and is considering the acquisition of a new machine which costs $400,000 and has a useful life of 5 years. Willis projects that earnings and cash flow will increase as follows:

Year Net Earnings After-Tax Cash Flow1 $100,000 $160,0002 100,000 140,0003 100,000 100,0004 100,000 100,0005 200,000 100,000

15% Interest Rate FactorsPeriod Present Value of $1 Present Value of an Annuity of $1

1 0.87 0.872 0.76 1.633 0.66 2.294 0.57 2.865 0.50 3.36

206. The net present value of this investment is a. Negative, $64,000 c. Positive, $18,600b. Negative, $14,000 d. Positive, $200,000

207. What is the payback period of this investment?a. 1.50 years. c. 3.33 years.b. 3.00 years. d. 4.00 years.

Questions 83 & 84 are based on the following information. AICPA adapted

CMA EXAMINATION QUESTIONS Page 68 of 155

Page 69: P02 - Capital Budgeting

MANAGEMENT ADVISORY SERVICES CAPITAL BUDGETING

(Ignore income taxes in this problem.) Apex Corp. is planning to buy production machinery costing $100,000. This machinery's expected useful life is five years, with no residual value. Apex uses a discount rate of 10% and has calculated the following data pertaining to the purchase and operation of this machinery:

Year Estimated annual net cash inflow1 $ 60,0002 30,0003 20,0004 20,0005 20,000

83. The payback period is: (E)a. 2.50 years. c. 3.00 years.b. 2.75 years. d. 5.00 years.

84. The net present value is closest to: (E)a. $20,400. c. $80,000.b. $28,400. d. $50,000.

Payback, Present Value of Cash Flow, Profitability Index Questions 8 through 10 are based on the following information. RPCPA 1079Caloocan Manufacturing Co. which has a 14% cost of capital, is planning a project that will cost P80,000. The annual cash inflow, net of income taxes, together with the present value factors (14%) are as follows:

Year Cash Flow Present Value of P1 at 14%1 P16,000 0.8772 P32,000 0.7693 P32,000 0.6754 P48,000 0.592

*. The present value of the cash flow generated by the project is (E)a. P88,656 c. P91,456b. P90,056 d. P92,856

*. The profitability index of the project (rounded to the nearest hundredth) is (E)a. 1.02 c. 1.10b. 1.07 d. 1.15

*. Payback period for the project is (E)a. 2.75 years c. 3.5 yearsb. 3 years d. 4 years.

Payback, NPV & Annual Cash InflowQuestions 1 thru 3 are based on the following information.James Corporation is considering an investment in equipment for $50,000. Data related to the investment are as follows:

Year Income Before Depreciation and Taxes1 $25,0002 25,0003 25,000

CMA EXAMINATION QUESTIONS Page 69 of 155

Page 70: P02 - Capital Budgeting

MANAGEMENT ADVISORY SERVICES CAPITAL BUDGETING

4 25,000James uses the straight-line method of depreciation with no mid-year convention. In addition, their tax rate is 40 percent and the life of the equipment is four years with no salvage value. Cost of capital is 12 percent.

208. What is the payback period in years approximated to two decimal points?a. 2.00 d. 2.50b. 0.40 e. 0.50c. 3.33 H & M

209. What is the net present value of the investment?a. $60,740 c. $25,925b. $(4,445) d. $10,740 H & M

210. What is the annual net cash inflow for year 1?a. $20,000 c. $25,000b. $15,000 d. $5,000 H & M

Payback, NPV & PIQuestions 80 through 82 are based on the following information. GleimTonya, Inc. has a cost of capital of 15% and is considering the acquisition of a new machine that costs $800,000 and has a useful life of 5 years. Tonya projects that earnings and cash flow will increase as follows:

Year Net Earnings After-Tax Cash Flow1 $200,000 $320,0002 200,000 280,0003 200,000 200,0004 200,000 200,0005 200,000 200,000

Interest rate factors at 15% are as follows:Period Present Value of $1 Present Value of an Annuity

1 0.87 0.872 0.76 1.633 0.66 2.294 0.57 2.865 0.50 3.36

211. The net present value of this investment isa. $(128,000) c. $37,200b. $200,000 d. $400,000

212. What is the profitability index for the investment?a. 0.05 c. 1.05b. 0.96 d. 1.25

213. What is the payback period of this investment?a. 1.5 years. c. 3.3 years.b. 3.0 years. d. 4.0 years.

Questions 102 through 105 are based on the following information. GleimMS Trucking is considering the purchase of a new piece of equipment that has a net initial investment with a present vale of $300,000. The equipment has an estimated useful life of 3 years. For tax purposes, the equipment will be fully depreciated at rates of 30%, 40%, and 30%, in years one, two, and three, respectively. The new machine is expected to have a $20,000 salvage value. The machine is expected to save the company $170,000 per year in operating expenses. MS Trucking has a 40% marginal income tax rate and a 1^% cost of capital. Discount rates for a 16% rate are:

Present Value of an Ordinary Annuity of $1 Present Value of $1Year 1 0.862 0.862Year 2 1.605 0.743Year 3 2.246 0.641

214. What is the net present value of this project?a. $31,684 c. $94,640b. $26,556 d. $18,864

215. What is the profitability index for the project?a. 1.089 c. 1.315b. 1.106 d. 1.063

216. The payback period for this investment isa. 2.84 years. c. 2.08 years.b. 1.76 years. d. 3.00 years

217. Assume the same facts as above, except that the salvage value at the end of the investment’s useful life is zero. What is the new payback period?a. 2.84 years. c. 2.08 years.b. 1.76 years. d. 2.09 years.

Expected Annual Cost Savings & IRRQuestions 86 and 87 are based on the following information. Barfield

CMA EXAMINATION QUESTIONS Page 70 of 155

Page 71: P02 - Capital Budgeting

MANAGEMENT ADVISORY SERVICES CAPITAL BUDGETING

The Allendale Co. has recently evaluated a proposal to invest in cost-reducing production technology. According to the evaluation, the project would require an initial investment of $17,166 and would provide equal annual cost savings for five years. Based on a 10 percent discount rate, the project generates a net present value of $1,788. The project is not expected to have any salvage value at the end of its five-year life.

86. (Present value tables needed to answer this question.) What are the expected annual cost savings of the project?a. $3,500 c. $4,500b. $4,000 d. $5,000

87. (Present value tables needed to answer this question.) What is the project's expected internal rate of return?a. 10% c. 13%b. 11% d. 14%

Payback Period, NPV & IRRQuestions 14 through 16 are based on the following information. RPCPA 1085Bernie Co. evaluates 2 alternative investment opportunities. All capital investment in this company are expected to yield a discount rate of return of less than 12%. The following data on the 2 investment proposals are:

Proposal M Proposal NRequired investment P 440,000 P 480,000Estimated service life 5 years 6 yearsEstimated salvage value P 20,000 -Estimated annual cash flow 170,000 P 120,000Depreciation straight-line 84,000 80,000Estimated annual income 36,000 40,000

*. Rate of all investment computed for Proposal M and Proposal N, respectively:a. 8.2% and 8.3% c. 16.4% and 17.4%b. 15.7% and 16.7% d. 18.6% and 18.6%

*. Payback period a. 3.0 for M and 3.5 for N c. 5.0 for M and 6.0 for Nb. 3.7 for M and 4.0 for N d. 12.2 for M and 12.0 for N

*. Net present value discounted at an annual rate of 12% of proposal N if the present value to be received for 5 periods is 3.605 and for 6 periods 4.111.a. P3,940 c. P13,320b. P7,880 d. P26,640

Questions 81 through 83 are based on the following information. BarfieldHefty Investment Co. is considering an investment in a labor-saving machine. Information on this machine follows:

Cost $30,000Salvage value in five years $0Estimated life 5 yearsAnnual depreciation $6,000

CMA EXAMINATION QUESTIONS Page 71 of 155

Page 72: P02 - Capital Budgeting

MANAGEMENT ADVISORY SERVICES CAPITAL BUDGETING

Annual reduction in existing costs $8,000

81. (Present value tables needed to answer this question.) What is the internal rate of return on this project (round to the nearest 1/2%)?a. 37.5% c. 10.5%b. 25.0% d. 13.5%

82. (Present value tables needed to answer this question.) Assume for this question only that Hefty Co. uses a discount rate of 16 percent to evaluate projects of this type. What is the project's net present value?a. $(6,283) c. $(23,451)b. $(3,806) d. $(22,000)

83. What is the payback period on this investment?a. 4 years c. 3.75 yearsb. 2.14 years d. 5 years

Payback, NPV, IRR & Cash FlowsQuestions 1 thru 3 are based on the following information. H & M78. Frank Drewer is considering the purchase of a computer-aided manufacturing system. The

after-tax cash benefits/savings associated with the system are as follows:Decreased waste $150,000Increased quality 200,000Decrease in operating costs 300,000Increase in on-time deliveries 100,000

The system will cost $4,500,000 and will last ten years. The company’s cost of capital is 12 percent.

218. What is the payback period for the computer-aided manufacturing system?a. 10.00 years d. 6.92 yearsb. 15.00 years e. 6.00 yearsc. 11.25 years

219. What is the NPV for the computer-aided manufacturing system?a. $4,500,000 d. $4,237,500b. $(262,500) e. $3,000,000c. $(2,805,000)

220. Which of the following best describes the IRR for this project?a. between 8 and 10 percent d. between 14 and 16 percentb. between 10 and 12 percent e. above 16 percent.

c. between 12 and 14 percent

Questions 114 through 117 are based on the following information. CIA 1195 IV-38 to 41An organization has four investment proposals with the following costs and expected cash inflows:

Expected Cash InflowsProject Cost End of Year 1 End of Year 2 End of Year 3

A Unknown $10,000 $10,000 $10,000B $20,000 $ 5,000 $10,000 $15,000

CMA EXAMINATION QUESTIONS Page 72 of 155

Page 73: P02 - Capital Budgeting

MANAGEMENT ADVISORY SERVICES CAPITAL BUDGETING

C $25,000 $15,000 $10,000 $ 5,000D $30,000 $20,000 Unknown $20,000

Additional information:

Discount Rate

Number of Periods

Present Value of $1 Due at the End of n Periods

(PVIP)

Present Value of an Annuity of $1 per Period for n Periods

(PVIFA)5% 1 0.9524 0.9524

5% 2 0.9070 1.85945% 3 0.8638 2.723210% 1 0.9091 0.909110% 2 0.8264 1.735510% 3 0.7513 2.486915% 1 0.8696 0.869615% 2 0.7561 1.625715% 3 0.6575 2.2832

221. If Project A has an internal rate of return (IRR) of 15%, it has a cost ofa. $8,696 c. $24,869b. $22,832 d. $27,232

222. If the discount rate is 10%, the net present value (NPV) of Project B isa. $4,079 c. $9,869b. $6,789 d. $39,204

223. The payback period of Project C isa. 0 years c. 2 years.b. 1 year. d. 3 years.

224. If the discount rate is 5% and the discounted payback period of Project D is exactly 2 years, then the year 2 cash inflow for Project D isa. $5,890 c. $12,075b. $10,000 d. $14,301

Questions 113 through 116 are based on the following information. CIA 0595 IV-41 to 44A company is evaluating three investment projects.Project A costs $100,000, has a 10% cost of capital, and has equal end-of-period cash inflows each year for five years.Project B costs $50,000 and has equal end-of-period cash inflows of $40,000 per year for two years.Project C costs $60,000 and has cash inflows of $20,000 at the end of year one, $35,000 at the end of year two, and an unknown cash inflow at the end of year three.

Number of Years

Discount Rate (Percent)

Present Value of $1 Due at the end of n Periods (PVIF)

Present Value of $1 per Period for n Periods (PVIFA)

1 10 0.9091 0.90911 12 0.8929 0.89292 10 0.8264 1.73552 12 0.7972 1.69013 10 0.7513 2.4869

CMA EXAMINATION QUESTIONS Page 73 of 155

Page 74: P02 - Capital Budgeting

MANAGEMENT ADVISORY SERVICES CAPITAL BUDGETING

3 12 0.7118 2.40184 10 0.683 3.16994 12 0.6355 3.03735 10 0.6209 3.79085 12 0.5674 3.6048

225. If the net present value of Project A is $12,405, then the size of each equal, end-of-period cash inflow, to the nearest dollar, is A. $19,979 C. $29,652B. $21,863 D. $31,182

226. If the cost of capital is 12%, the net present value of Project B, to the nearest dollar, is A. ($18,112) C. $17,604B. ($16,944) D. $19,420

227. If Project C has an internal rate of return of 12%, then the cash inflow at the end of year three, to the nearest dollar, is A. $17,162 C. $24,126B. $20,006 D. $24,981

228. If the year three cash inflow for Project C is $46,000, then the payback period for <List A> is <List B> year(s).

A. B. C. D.List A Project B Project B Project C Project CList B One Two One Two

Questions 24 thru 29 are based on the following information. GitmanNuff Folding Box Company, Inc. is considering purchasing a new gluing machine. The gluing machine costs $50,000 and requires installation costs of $2,500. This outlay would be partially offset by the sale of an existing gluer. The existing gluer originally cost $10,000 and is four years old. It is being depreciated under MACRS using a five-year recovery schedule and can currently be sold for $15,000. The existing gluer has a remaining useful life of five years. If held until year 5, the existing machine's market value would be zero. Over its five-year life, the new machine should reduce operating costs (excluding depreciation) by $17,000 per year. Training costs of employees who will operate the new machine will be a one-time cost of $5,000 which should be included in the initial outlay. The new machine will be depreciated under MACRS using a five-year recovery period. The firm has a 12 percent cost of capital and a 40 percent tax on ordinary income and capital gains.

24. The payback period for the project is A. 2 years. C. between 3 and 4 years.

B. 3 years. D. between 4 and 5 years.

25. The tax effect of the sale of the existing asset isA. a tax liability of $2,340. C. a tax liability of $3,320.B. a tax benefit of $1,500. D. a tax liability of $5,320.

26. The initial outlay for this project isA. $42,820. C. $47,820.B. $40,320. D. $35,140.

27. The present value of the project's annual cash flows is A. $ 47,820. C. $ 51,694.B. $ 42,820. D. $100,563.

28. The net present value of the project isA. $3,874. C. $5,614.B. $2,445. D. $7,500.

29. The internal rate of return for the project isA. between 7 and 8 percent. C. greater than 12 percent.B. between 9 and 10 percent. D. between 10 and 11 percent.

Payback, NPV, IRR & Investment DecisionQuestions 109 through 112 are based on the following information. CIA 1194 IV-31 to 33A company is evaluating two investment proposals. Project A costs $700,000 and Project B costs $500,000. Expected cash inflows for the two projects are as follows:

Year Cash Inflow at End of Year for Project A Cash Inflow at End of Year for Project B1 $400,000 $250,0002 $300,000 $250,0003 $600,000 $250,000

Both projects are assessed as being equally risky and having an appropriate cost of capital of 10%.For a 10% cost of capital, the following present value factors apply:

Number of Years

Present Value of $1 Received at End of Year n [PVIF]

Present Value of an Annuity of $1 Received at End of Each Year for n Years [PVIFA]

1 .9091 .90912 .8264 1.73553 .7513 2.4873

For a 3-year annuity, the following present value factors apply:Discount Rate Present Value of a 3-Year Annuity of $1 (PVIFA)

CMA EXAMINATION QUESTIONS Page 74 of 155

Page 75: P02 - Capital Budgeting

MANAGEMENT ADVISORY SERVICES CAPITAL BUDGETING

16 percent 2.245920 percent 2.106524 percent 1.981328 percent 1.8684

229. Project A has a payback period of A. 0 years. C. 2 years.B. 1 year. D. 3 years. CIA 1194 IV-31

230. Project A has a net present value (to the nearest dollar) of A. $362,340 C. $1,676,670 B. $1,062,340 D. $2,376,670 CIA 1194 IV-32

231. Which of the following most closely approximates the internal rate of return of Project B? A. 16% C. 24%B. 20% D. 28% CIA 1194 IV-33

232. If projects A and B are independent, the company will undertake A. Both projects. C. Only Project B.B. Only Project A. D. Neither project. CIA 1194 IV-34

NPV & IRRQuestions 78 & 79 are based on the following information. G & N 9e (Ignore income taxes in this problem.) Treads Corporation is considering the replacement of an old machine that is currently being used. The old machine is fully depreciated but can be used by the corporation for five more years. If Treads decides to replace the old machine, Picco Company has offered to purchase the old machine for $60,000. The old machine would have no salvage value in five years.The new machine would be acquired from Hillcrest Industries for $1,000,000 in cash. The new machine has an expected useful life of five years with no salvage value. Due to the increased efficiency of the new machine, estimated annual cash savings of $300,000 would be generated.Treads Corporation uses a discount rate of 12%.

78. The net present value of the project is closest to: (M)a. $171,000. c. $141,500.b. $136,400. d. $560,000.

79. The internal rate of return of the project is closest to: (M)a. 14%. c. 18%.b. 16%. d. 20%.

Questions 89 and 90 are based on the following information. CIA 0597 IV-40 & 41A firm with an 18% cost of capital is considering the following projects (on January 1, year 1):

January 1, Year 1Cash Outflow

(000’s Omitted)

December 31, Year 5Cash Inflow

(000’s Omitted)Project InternalRate of Return

Project A $3,500 $7,400 16%Project B 4,000 9,950 ?

Present Value of $1 due at the End of “N” PeriodsCMA EXAMINATION QUESTIONS Page 75 of 155

Page 76: P02 - Capital Budgeting

MANAGEMENT ADVISORY SERVICES CAPITAL BUDGETING

N 12% 14% 15% 16% 18% 20% 22%4 .6355 .5921 .5718 .5523 .5153 .4823 .42305 .5674 .5194 .4972 .4761 .4371 .4019 .34116 .5066 .4556 .4323 .4104 .3704 .3349 .2751

233. Using the net-present-value (NPV) method, project A’s net present value isa. $316,920 c. $(265,460)b. $23,140 d. $(316,920)

234. Project B’s internal rate of return is closed toa. 15% c. 18%b. 16% d. 20%

Questions 14 and 15 are based on the following information. RPCPA 0581V. Ortega, Inc. is evaluating two alternative investment proposals, X & Y. Each project has estimated life of ten years. All capital investments in this company are expected to yield a discounted rate of return of not less than 15%. Additional information on the two projects and a present value (Present Value table are shown below.)

Project X Project YNet investment P60,000 P135,000Annual return for each of 10 years P16,295 P 32,204

Present Value of P1 Received Annually for 10 yearsInt. Rate Present Value Int. Rate Present Value

12% 5.650 20% 4.19214% 5.216 22% 3.92315% 5.019 24% 3.68216% 4.833 25% 3.57118% 4.494 26% 3.465

*. The discounted rate of return for each of the two projects are: (M)a. b. c. d.

Project X 14% 22% 24% 24%Project Y 18% 25% 20% 24%

*. Based on the minimum rate of return requirement of 15%, the net or excess present value of returns for each of the two projects are (E)

a. b. c. d.Project X P16,295 P21,785 P 81,785 P102,950Project Y P32,204 P26,632 P161,632 P187,040

Questions 111 through 113 are based on the following information. CIA 0594 IV-40 to 42A company that annually reviews its investment opportunities and selects appropriate capital expenditures for the coming year is presented with two projects, called Project A and Project B. Best estimates indicate that the investment outlay for Project A is $30,000 and for Project B is $1 million. The projects are considered to be equally risky. Project A is expected to generate cash inflows of $40,000 at the end of each year for 2 years. Project B is expected to generate cash inflows of $700,000 at the end of the first year and $500,000 at the end of the second year. The company has a cost of capital of 8%.

235. What is the net present value (NPV) of each project when the cost of capital is zero?a. b. c. d.

Project A $30,000 $50,000 $80,000 $110,000Project B $1,000,000 $200,000 $1,200,000 $2,200,000

236. The internal rate of return of Project A, to the nearest full percentage point isa. 10% c. 25%b. 15% d. 100%

237. Net present value (NPV) and internal rate of return (IRR) differ in thata. NPV assumes reinvestment of project cash flows at the cost of capital, whereas IRR

assumes reinvestment of project cash flows at the internal rate of return.b. NPV and IRR make different accept or reject decisions for independent projects.c. IRR can be used to rank mutually exclusive investment projects, but NPV cannot.d. NPV is expressed as a percentage, while IRR is expressed as a dollar amount.

Net Income, Incremental Cash Inflow, Payback Period & Discounted Rate of Return*. The composite life of the company’s fixed assets was determined to be nine years at the end

of 1993. One-tenth of the 1993 balance is the net book value of an equipment line used for bottling operations. An investment opportunity to replace this line was presented in the beginning of 1994 with the following additional data:

Disposal value of old line is equal to its net book value. Cost of new line inclusive of civil works, installation and commissioning costs, P35,000.

Estimated useful life is 15 years. Immediate repairs needed on old line is P1,000 Income tax rate is 35% Annual out-of-pocket operating costs: old line, P9,000; new line, P3,000 The table of present values include the following:

14% 16% 18% 20%Present Value of P1 due 15 years 0.140 0.108 0.084 0.065Present Value of P1 received annually

CMA EXAMINATION QUESTIONS Page 76 of 155

Page 77: P02 - Capital Budgeting

MANAGEMENT ADVISORY SERVICES CAPITAL BUDGETING

for 15 years 6.142 5.575 5.092 4.675Determine the net investment (NI), incremental cash inflow (CI), payback period (PP), and the discounted rate of return (DRR) of the new investment. (D)

a. b. c. d.NI P25,114,40 P25,764.40 P24,764.40 P25,414.40CI P4,343.89 P3,075.63 P4,343.89 P4,343.89PP 4.5 years 8.4 years 5.7 years 5.9 yearsDRR 22.22% 11.91% 15.56% 16.95%

Investment DecisionsQuestions 1 through 5 are based on the following information. RPCPA 1076YOU are the management adviser of Masugid Company, a management company. It is presently considering the putting in of certain concessions in the main lobby of an office building which it manages. Your study brought out the following estimates on an average annual basis.

Salaries P 7,000Licences and taxes 200Cost of merchandise sold 40,000Share of air conditioning and light 500Pro rata building depreciation 1,000Concession advertising 100Share of company administrative costs 400Sale of merchandise 49,000

The investment in equipment, which would last 10 years (no salvage value) would be P5,000. As an alternative, a catering company has offered to lease for P750 per year, for 10 years, and to put in and operate by the office building at no additional charge. In formulating your advice to the management of the company, you determine that:*. The annual cash flow which you consider the incremental advantage in the ownership

alternative isa. P1,700 c. P1,900b. P750 d. P9,000

*. Using the Present Value Table for 10 years (the period of the lease offer) and 10%, the yield of alternative opportunities for this project (given that PV factors for 1 to 10 are 1.00; 0.909; 0.751; 0.683; 0.621, 0.564; 0.513; 0.467; 0.424; and 0.386) the present value of the lease alternatives isa. P4,609 c. P750b. P5,445 d. P1,700

*. The total of the costs irrelevant to the decision is a. P9,000 d. P800

b. P1,000 e. none of thesec. P900

*. The net present value of the ownership alternative is a. P9,000 c. P5,445b. P1,000 d. P4,609

*. The total of the pertinent costs that entered your cash flow considerations isa. P4,609 c. P47,300b. P49,200 d. P9,200

Optimal Project Selection238. Jackson Corporation is evaluating the following four independent, investment opportunities:

Project Cost Rate of ReturnA $300,000 14%B 150,000 10C 200,000 13D 400,000 11

Jackson’s target capital structure is 60 percent debt and 40 percent equity. The yield to maturity on the company’s debt is 10 percent. Jackson will incur flotation costs for a new equity issuance of 12 percent. The growth rate is a constant 6 percent. The stock price is currently $35 per share for each of the 10,000 shares outstanding. Jackson expects to earn net income of $100,000 this coming year and the dividend payout ratio will be 50 percent. If the company’s tax rate is 30 percent, which of the projects will be accepted? (M)a. Project Ab. Projects A and Cc. Projects A, C, and D d. All of the investment projects will be taken.e. None of the investment projects will be taken. Brigham

Optimal Capital Budget239. Gibson Inc. is considering the following five independent projects:

Project Cost IRRA $200,000 20%B 600,000 15C 400,000 12D 400,000 11E 400,000 10

The company has a target capital structure that consists of 40 percent debt and 60 percent equity. The company can issue bonds with a yield to maturity of 11 percent. The company has $600,000 in retained earnings, and the current stock price is $42 per share. The flotation costs

CMA EXAMINATION QUESTIONS Page 77 of 155

Page 78: P02 - Capital Budgeting

MANAGEMENT ADVISORY SERVICES CAPITAL BUDGETING

associated with issuing new equity are $2 per share. Gibson’s earnings are expected to continue to grow at 6 percent per year. Next year’s dividend (D1) is forecasted to be $4.00. The firm faces a 40 percent tax rate. What is the size of Gibson’s capital budget? (D)a. $ 200,000 d. $1,600,000b. $ 800,000 e. $2,000,000c. $1,200,000 Brigham

240. Photon Corporation has a target capital structure that consists of 60 percent equity and 40 percent debt. The firm can raise an unlimited amount of debt at a before-tax cost of 9 percent. The company expects to retain earnings of $300,000 in the coming year and to face a tax rate of 35 percent. The last dividend (D0) was $2 per share and the growth rate of the company is constant at 6 percent. If the company needs to issue new equity, then the flotation cost will be $5 per share. The current stock price (P0) is $30. Photon has the following investment opportunities:

Project Cost IRR1 $100,000 10.5%2 200,000 13.03 100,000 12.04 150,000 14.0

5 75,000 9.0What is the company’s optimal capital budget? (D)a. $625,000 d. $550,000b. $450,000 e. $150,000c. $350,000 Brigham

241. Atlee Associates has a capital structure that consists of 40 percent debt and 60 percent common stock. The yield to maturity on the company’s debt is 8 percent, the cost of retained earnings is 12 percent, and the cost of issuing new equity is 13 percent. The company expects its net income to be $500,000, the dividend payout is expected to be 40 percent, and its tax rate is 40 percent. The company is considering five projects, all with the same risk. The size and estimated returns of the proposed projects are listed below:

Project Cost IRRA $200,000 11.00%B 100,000 10.00C 100,000 9.95D 200,000 9.85E 200,000 9.25

On the basis of this information, what is Atlee’s optimal capital budget? (D)a. $800,000 d. $300,000b. $600,000 e. $200,000c. $400,000 Brigham

Questions 79 through 82 are based on the following information. (06-13-61 to 64)A company has the following three investment projects available:

Project Cost Internal Rate of ReturnA $ 50 million 14%B $ 75 million 12%C $125 million 8%

The company has a 40% debt and 60% equity capital structure. Each dollar of investment funds will be raised in these proportions (40 cents of debt and 60 cents of equity.)The marginal cost of financing increases with the amount of new funds raised, as follows:

Interval Amount Raised Weighted-Average Cost of Capital1 First $ 75 million 6%2 Next $100 million 10%3 Over $175 million 12%

These investment opportunities and financing costs are shown in the graph below:Percent15 A14

CMA EXAMINATION QUESTIONS Page 78 of 155

Page 79: P02 - Capital Budgeting

MANAGEMENT ADVISORY SERVICES CAPITAL BUDGETING

13 B 312 MCC11 2109 C8 IOS7 16543210 | | | | | | | | | 25 50 75 100 125 150 175 200 225 $ MillionMCC – Margin cost of capitalIOS – Investment opportunity schedule

242. The investment opportunity schedule (IOS) shows, in rank order, how much money the company would invest at different rates of return. Such schedules can be drawn only for a set of projects that a. Have the same investment cost. c. Have the same net present value.b. Are mutually exclusive. d. Are independent.

243. The company should invest in Project(s) <List A> and has an optimal capital budget of <List B> million dollars.

a. b. c. d.List A B only A and B only A and C only C onlyList B 75 125 175 125

244. Without prejudice to your answer to question 68, assume that the optimal capital budget for the company is $150 million. The marginal cost of capital and the appropriate discount rate to use in evaluating investment proposals for this company would bea. 6% c. 10%b. 8% d. 12%

245. The marginal cost of capital (MCC) curve for this company rises twice, first when the company has raised $75 million and again when $175 million of new funds has been raised. These increases in the MCC caused by thea. Increases in the returns on the additional investments undertaken.

b. Decreases in the returns on the additional investments undertaken.c. Decreases in the cost of at least one of the financing sources.d. Increases in the cost of at least one of the financing sources.

Capital LeaseQuestions 92 thru 94 are based on the following information. Pol BobadillaA firm must choose between leasing a new asset or purchasing it with funds from a term loan. Under the purchase option, the firm will pay five equal principal payments of P1,000 each and 6% interest on the unpaid balance. Principal and interest are due at the end of each year for five years. Alternatively, the firm can lease the asset for five years at an annual rental cost of P1,400 with payments due at the beginning of each year. The corporate tax rate is 35% and the appropriate after-tax cost of capital is 12%.

92. Which of the following is closes to the PV of the after-tax interest payment?A. P360 C. P640B. P453 D. P726

93. Which of the following is closest to the present value of cost if leasing the asset?A. P3,674 C. P3,849B. P3,779 D. P3,992

94. Which of the following is closest to the PV of cost of purchasing the new asset with a term loan?A. 3,777 C. 4,058B. 3,952 D. 4,153

ComprehensiveQuestions 99 thru 101 are based on the following information. Pol BobadillaLogo Co. is planning to buy a coin-operated machine costing P40,000. For book and tax purposes, this machine will be depreciated P8,000 each year for five years. Logo estimates that this machine will yield an annual cash inflow, net of depreciation and income taxes, of P12,000. Logo’s desired rate of return on its investment is 12%. At the following discount rates, the NPVs of the investment in this machine are:

Discount rate 12% 14% 16% 18%NPV +P3,258 +P1,197 – P708 – P2,474

99. Logo’s accounting rate of return on its initial investment in this machine is expected to beA. 30% C. 12%B. 15% D. 10%

100.Logo’s expected payback period for its investment in this machine isCMA EXAMINATION QUESTIONS Page 79 of 155

Page 80: P02 - Capital Budgeting

MANAGEMENT ADVISORY SERVICES CAPITAL BUDGETING

A. 2.0 years. C. 3.3 years.B. 3.0 years. D. 5.0 years

101.Logo’s expected IRR on its investment in this machine isA. 3.3% C. 12.0%B. 10.0% D. 15.3%

Questions 95 through 101 are based on the following information. CMA 1285 5-1 to 7At the beginning of 1996, Garrison Corporation is considering the replacement of an old machine that is currently being used. The old machine is fully depreciated but can be used by the corporation for an additional 5 years, that is, through 2000. If Garrison decides to replace the old machine, Picco Company has offered to purchase it for $60,000 on the replacement date. The old machine would have no salvage value in 2000.If the replacement occurs, a new machine will be acquired from Hillcrest Industries on January 2, 1996. The purchase price of $1,000,000 for the new machine will be paid in cash at the time of replacement. Because of the increased efficiency of the new machine, estimated annual cash savings of $300,000 will be generated through 2000, the end of its expected useful life. The new machine is not expected to have any salvage value at the end of 2000.All operating cash receipts, operating cash expenditures, and applicable tax payments and credits are assumed to occur at the end of the year. Garrison employs the calendar year for reporting purposes.Discount tables for several different interest rates that are to be used in any discounting calculations are given below.

Present Value of $1.00 Received at End of PeriodPeriod 9% 12% 15% 18% 21%

1 .92 .89 .87 .85 .832 .84 .80 .76 .72 .683 .77 .71 .65 .61 .564 .71 .64 .57 .51 .475 .65 .57 .50 .44 .39

Present Value of an Annuity of $1.00 Received at the End of Each PeriodPeriod 9% 12% 15% 18% 21%

1 .92 .89 .87 .85 .832 1.76 1.69 1.63 1.57 1.513 2.53 2.40 2.28 2.18 2.074 3.24 3.04 2.85 2.69 2.545 3.89 3.61 3.35 3.13 2.93

For questions 96 through 98 only, assume that Garrison is not subject to income taxes.

246. If Garrison requires investments to earn 12% return, the NPV for replacing the old machine with the new machine isa. $171,000 c. $143,000b. $136,400 d. $83,000 CMA 1285 5-1

247. The IRR, to the nearest percent, to replace the old machine isa. 9% c. 17%b. 15% d. 18% CMA 1285 5-2

248. The payback period to replace the old machine with the new machine isa. 1.14 years. c. 3.13 years.b. 2.78 years. d. 3.33 years. CMA 1285 5-3

Questions 98 through 101 are based on the following additional information.The assumptions are Garrison requires all investments to earn a 12% after-tax rate of return to be accepted. Garrison is subject to a marginal income tax rate of 40% on all income and gains (losses). The new machine will have depreciation as follows:

Year Depreciation1996 $ 250,0001997 380,0001998 370,000

$1,000,000

249. The present value of the depreciation tax shield for 1997 isa. $182,400 c. $109,440b. $121,600 d. $114,304 CMA 1285 5-7

250. The present value of the after-tax cash flow associated with the salvage of the old machine is (M)a. $38,640 c. $32,040b. $36,000 d. $27,960 CMA 1285 5-5

251. The present value of the annual after-tax cash savings that arise from the increased efficiency of the new machine throughout its life (calculated before consideration of any depreciation tax shield) isa. $563,400 c. $433,200b. $375,600 d. $649,800 CMA 1285 5-6

252. If the new machine is expected to be sold for $80,000 on December 31, 2000, the present value of the additional after-tax cash flow is

CMA EXAMINATION QUESTIONS Page 80 of 155

Page 81: P02 - Capital Budgeting

MANAGEMENT ADVISORY SERVICES CAPITAL BUDGETING

a. $18,240 c. $45,600b. $27,360 d. $46,000 CMA 1285 5-8

Questions 3 through 11 are based on the following information. RPCPA 0579The Burgos Corporation is considering investing in a project. It requires an immediate cash outlay of P100,000. It has a life of four years and will be depreciated on a straight-line basis (no salvage value). The firm’s tax rate is 25% and requires a return of 10%. Income before depreciation is projected to be:

YEAR 1 2 3 4Income before depreciation P30,000 P30,000 P40,000 P40,00

0

The present value factors for P1 at 10% isYear 1 2 3 4Present Value Factor 0.909 0.826 0.751 0.683

*. The net cash flow for year 1 is (M)a. P25,850 c. P31,250b. P28,750 d. P34,450

*. The net cash flow for year 4 is (M)a. P35,850 c. P30,150b. P35,950 d. P36,250

*. The payback period for the project is (M)a. 3 years c. 3.5 yearsb. 3.17 years d. 4 years.

*. The accounting rate of return of the project is253 (M)a. 7% c. 12%b. 9% d. 15% (?)

*. The present value of year two’s cash flow is (M)a. P23,747.50 c. P26,100.75b. P25,856.25 d. P29,750.75

*. The present value of the project’s net cash flow is (M)a. P95,650.15 c. P101,863.75b. P98,151.25 d. P104,750.25

*. The profitability index of the project (rounded to the nearest hundredth) is (M)a. 0.96 c. 1.02

b. 0.98 d. 1.05

*. The project would be accepted on the basis of the (M)a. Payback and present value results.b. Accounting rate of return and profitability index results.c. Payback results onlyd. a and b combined

*. The project would be rejected on the basis of the (M)a. Payback and present value results.b. Accounting rate of return and profitability index results.c. Payback results onlyd. a and b combinede. None of the above.

CMA EXAMINATION QUESTIONS Page 81 of 155

Page 82: P02 - Capital Budgeting

MANAGEMENT ADVISORY SERVICES CAPITAL BUDGETING

ANSWER EXPLANATIONSACCOUNTING RATE OF RETURN254. Jensen Manufacturing is considering buying an automated machine that costs $250,000. It

requires working capital of $25,000. Annual cash savings are anticipated to be $103,000 for five years. The company uses straight-line depreciation. The salvage value at the end of five years is expected to be $10,000. The working capital will be recovered at the end of the machine's life.

Required: (M) HorngrenCompute the accrual accounting rate of return based on the initial investment.

CASH FLOWSCash Flow from Sale of Old Equipment255. Darwin Company is considering the sale of a machine with the following characteristics.

Book value $110,000Remaining useful life 5 yearsAnnual straight-line depreciation $ ???Current market value $120,000

If the company sells the machine its cash operating expenses will increase by $20,000 per year. The tax rate is 40%.

Required: D, L & H 9ea. Find the cash flow from selling the machine.b. Calculate the increase in annual net cash outflows as a result of selling the machine.

Estimating Pretax Cash Inflows256. (Ignore income taxes in this problem.) AB Company is considering the purchase of a machine

that promises to reduce operating costs by the same amount for every year of its 6-year useful life. The machine will cost $83,150 and has no salvage value. The machine has a 20% internal rate of return.

Required: (Difficult) N & GWhat is the annual cost savings promised by the machine?

257. Skyway Corporation is considering purchasing a new machine to be used to manufacture a new product, called Jax, which will sell for $15 a unit. Variable manufacturing cost is expected to be $5 for each unit of Jax manufactured, and variable marketing cost, $2 for each unit sold. The machine being considered could produce 10,000 units a year, all of which the Marketing Department believes could be sold for $15 a unit. The proposed machine would cost $250,000. Although the machine would probably last 8 years, management believes that the product's life cycle would be only 5 years. The salvage value of the new machine at the end of

the product's 5-year life cycle is expected to be $50,000. Management does not believe the machine could be used to manufacture any of the company's other products.

Required: Carter & UsryCompute the pretax net cash inflows expected from the capital expenditure proposal for each year, and ignoring the effect of income taxes, determine the excess of cash inflows from all sources over the cost of the machine.

Cash Flow from Sale of Old Machine & Incremental Cash Flow258. Stockholm Company is considering the sale of a machine with the following characteristics.

Book value $120,000Remaining useful life 5 yearsAnnual straight-line depreciation $ 24,000Current market value $ 70,000

If the company sells the machine its cash operating expenses will increase by $30,000 per year due to an operating lease. The tax rate is 40%.

Required: D, L & H 9ea. Find the cash flow from selling the machine.b. Calculate the increase in annual net cash outflows as a result of selling the machine.

Net Investment, Incremental Cash Flow & Income Tax259. Rusk Company is considering replacing a machine that has the following characteristics.

Book value $200,000Remaining useful life 4 yearsAnnual straight-line depreciation $ ???Current market value $160,000

The replacement machine would cost $300,000, have a four-year life, and save $37,500 per year in cash operating costs. It would be depreciated using the straight-line method. The tax rate is 40%.

Required: D, L & H 9ea. Find the net investment required to replace the existing machine.b. Compute the increase in annual income taxes if the company replaces the machine.c. Compute the increase in annual net cash flows if the company replaces the machine.

260. Pepin Company is considering replacing a machine that has the following characteristics.Book value $100,000Remaining useful life 5 yearsAnnual straight-line depreciation $ ???Current market value $ 60,000

CMA EXAMINATION QUESTIONS Page 82 of 155

Page 83: P02 - Capital Budgeting

MANAGEMENT ADVISORY SERVICES CAPITAL BUDGETING

The replacement machine would cost $150,000, have a five-year life, and save $50,000 per year in cash operating costs. It would be depreciated using the straight-line method. The tax rate is 40%.

Required: D, L & H 9ea. Find the net investment required to replace the existing machine.b. Compute the increase in annual income taxes if the company replaces the machine.c. Compute the increase in annual net cash flows if the company replaces the machine.

PAYBACK PERIODPayback PeriodNET PRESENT VALUE261. ABC Boat Company is interested in replacing a molding machine with a new improved model.

The old machine has a salvage value of $20,000 now and a predicted salvage value of $4,000 in six years, if rebuilt. If the old machine is kept, it must be rebuilt in one year at a predicted cost of $40,000.The new machine costs $160,000 and has a predicted salvage value of $24,000 at the end of six years. If purchased, the new machine will allow cash savings of $40,000 for each of the first three years, and $20,000 for each year of its remaining six-year life.

Required: (D) HorngrenWhat is the net present value of purchasing the new machine if the company has a required rate of return of 14%?

262. (Ignore income taxes in this problem.) Jim Bingham is considering starting a small catering business. He would need to purchase a delivery van and various equipment costing $125,000 to equip the business and another $60,000 for inventories and other working capital needs. Rent for the building used by the business will be $35,000 per year. Jim’s marketing studies indicate that the annual cash inflow from the business will amount to $120,000. In addition to the building rent, annual cash outflow for operating costs will amount to $40,000. Jim wants to operate the catering business for only six years. He estimates that the equipment could be sold at that time for 4% of its original cost. Jim uses a 16% discount rate.

Required: (Moderate) N & GWould you advise Jim to make this investment?

263. (Ignore income taxes in this problem.) General Manufacturing Company consists of several divisions, one of which is the Transportation Division. The company has decided to dispose of this division since it no longer fits the company's long-term strategy. An offer of $9,000,000 has been received from a prospective buyer. If General retained the division, the company would operate the division for only nine years, after which the division would no longer be needed

and would be sold for $600,000. If the company retains the division, an immediate investment of $500,000 would need to be made to update equipment to current standards. Annual net operating cash flows would be $1,805,000 if the division is retained. The company’s discount rate is 12%.

Required: (Moderate) N & GUsing the net present value method, determine whether General Manufacturing should accept or reject the offer made by the potential buyer.

264 (Ignore income taxes in this problem.) Vernon Company has been offered a 7-year contract to supply a part for the military. After careful study, the company has developed the following estimated data relating to the contract:

Cost of equipment needed ............................. $300,000Working capital needed ............................... $ 50,000Annual cash receipts from the delivery of parts, less cash operating costs $ 70,000Salvage value of equipment at termination of the contract $ 5,000

It is not expected that the contract would be extended beyond the initial contract period. The company's discount rate is 10%.

Required: (Moderate) N & GUse the net present value method to determine if the contract should be accepted. Round all computations to the nearest dollar.

265. (Ignore income taxes in this problem.) Mark Stevens is considering opening a hobby and craft store. He would need $100,000 to equip the business and another $40,000 for inventories and other working capital needs. Rent on the building used by the business will be $24,000 per year. Mark estimates that the annual cash inflow from the business will amount to $90,000. In addition to building rent, annual cash outflow for operating costs will amount to $30,000. Mark plans to operate the business for only six years. He estimates that the equipment and furnishings could be sold at that time for 10% of their original cost. Mark uses a discount rate of 16%.

Required: (Moderate) N & GWould you advise Mark to make this investment? Use the net present value method.

266 (Ignore income taxes in this problem.) Bradley Company's required rate of return is 14%. The company has an opportunity to be the exclusive distributor of a very popular consumer item. No new equipment would be needed, but the company would have to use one-fourth of the space in a warehouse it owns. The warehouse cost $200,000 new. The warehouse is currently half-empty and there are no other plans to use the empty space. In addition, the company would have to invest $100,000 in working capital to carry inventories and accounts receivable

CMA EXAMINATION QUESTIONS Page 83 of 155

Page 84: P02 - Capital Budgeting

MANAGEMENT ADVISORY SERVICES CAPITAL BUDGETING

for the new product line. The company would have the distributorship for only 5 years. The distributorship would generate a $17,000 net annual cash inflow.

Required: (Moderate) N & G

What is the net present value of the project at a discount rate of 14%? Should the project be accepted?

267. (Ignore income taxes in this problem.) The following data concern an investment project:Investment in equipment $16,000Net annual cash inflows $ 3,600Working capital required $ 4,500Salvage value of the equipment $ 2,000Life of the project 12 yearsDiscount rate 14%

The working capital will be released for use elsewhere at the conclusion of the project.

Required: N & GCompute the project's net present value.

268 (Ignore income taxes in this problem.) Monson Company is considering three investment opportunities with cash flows as described below:

Project A: Cash investment now $15,000Cash inflow at the end of 5 years $21,000Cash inflow at the end of 8 years $21,000

Project B: Cash investment now $11,000Annual cash outflow for 5 years $ 3,000Additional cash inflow at the end of 5 years $21,000

Project C: Cash investment now ..................... $21,000Annual cash inflow for 4 years .......... $11,000Cash outflow at the end of 3 years ...... $ 5,000Additional cash inflow at the end of 4 years $15,000

Required: (Moderate) N & GCompute the net present value of each project assuming Monson Company uses a 12% discount rate.

COMPREHENSIVEAccounting Rate of Return & Payback269. (Ignore income taxes in this problem.) Ferris Company has an old machine that is fully

depreciated but has a current salvage value of $5,000. The company wants to purchase a new machine that would cost $60,000 and have a 5-year useful life and zero salvage value. Expected changes in annual revenues and expenses if the new machine is purchased are:

Increased revenues ............... $63,000Increased expenses: Salary of additional operator .. $20,000 Supplies ....................... 9,000 Depreciation ................... 12,000 Maintenance .................... 4,000 45,000Increased net income .............. $18,000

Required: (Easy) N & Ga. Compute the payback period on the new equipment.b. Compute the simple rate of return on the new equipment.

Accounting Rate of Return, Payback & NPV270. Scottso has an investment opportunity costing $180,000 that is expected to yield the following

cash flows over the next five years:Year One $ 30,000Year Two $ 60,000Year Three $ 90,000Year Four $ 60,000Year Five $ 30,000

Required: D, L & H 9eb. Find the book rate of return of the investment.c. Find the NPV of the investment at a cutoff rate of 12%.

271. Scottso has an investment opportunity costing $300,000 that is expected to yield the following cash flows over the next six years:

Year One $75,000Year Two $90,000Year Three $115,000Year Four $130,000Year Five $100,000Year Six $90,000

Required: D, L & H 9e

CMA EXAMINATION QUESTIONS Page 84 of 155

Page 85: P02 - Capital Budgeting

MANAGEMENT ADVISORY SERVICES CAPITAL BUDGETING

b. Find the book rate of return of the investment.c. Find the NPV of the investment at a cutoff rate of 10%.

272. The management of Elite Cookies Inc. is considering the purchase of a new shaping machine. The machine will cost $100,000 and will have a useful life of 10 years with a salvage value of $10,000 at the end of ten years. The investment will result in cost savings of $16,000 per year for each year of the machine's life. The tax rate is zero, and the appropriate discount rate for the company is 10%. (The present value factor for $1 received at the end of 10 years is .386, and the factor for $1 received annually for 10 years is 6.145.)

Required: Carter & Usry

(1) Compute the payback period.(2) Compute the accounting rate of return on the average investment.(3) Compute the net present value.(Round answers to two decimal places.)

Payback & NPV273. (Present value tables needed to answer this question.) Wood Productions is considering the

purchase of a new movie camera, which will be used for major motion pictures. The new camera will cost $30,000, have an eight-year life, and create cost savings of $5,000 per year. The new camera will require $700 of maintenance each year. Wood Productions uses a discount rate of 9 percent.

REQUIRED: Barfielda. Compute the net present value of the new camera.b. Determine the payback period.

274. Treble Co. is considering an investment in a new product line. The investment would require an immediate outlay of $100,000 for equipment and an immediate investment of $200,000 in working capital. The investment is expected to generate a net cash inflow of $100,000 in year 1, $150,000 in year 2, and $200,000 in years 3 and 4. The equipment would be scrapped (for no salvage) at the end of the fourth year and the working capital would be liquidated. The equipment would be fully depreciated by the straight-line method over its four-year life.

REQUIRED: Barfield(a) (Present value tables needed to answer this question.) If Treble uses a discount rate of 16

percent, what is the NPV of the proposed product line investment?(b) What is the payback period for the investment?

Payback & IRR275. Supply the missing data for each of the following proposals. (D) Horngren

Proposal A Proposal B Proposal CInitial investment (a) $62,900 $226,000Annual net cash inflow $60,000 (c) (e)Life, in years 10 6 10Salvage value $0 $10,000 $0Payback period in year (b) (d) 5.65Internal rate of return 12% 24% (f)

NPV & PI276. Racine Co. has the opportunity to introduce a new product. Racine expects the project to sell

for $200 and to have per-unit variable costs of $130 and annual cash fixed costs of $6,000,000. Expected annual sales volume is 125,000 units. The equipment needed to bring out the new product costs $7,200,000, has a four-year life and no salvage value, and would be depreciated on a straight-line basis. Working capital of $500,000 would be necessary to support the increased sales. Racine's cost of capital is 12% and its income tax rate is 40%.

Required: D, L & H 9ea. Compute the NPV of this opportunity.b. Compute the profitability index of this opportunity.

NPV & IRR277. The Zero Machine Company is evaluating a capital expenditure proposal that requires an initial

investment of $20,960 and has predicted cash inflows of $5,000 per year for 10 years. It will have no salvage value.

Required: (M) Horngrena. Using a required rate of return of 16%, determine the net present value of the investment

proposal.b. Determine the proposal's internal rate of return.

278. Network Service Center is considering purchasing a new computer network for $82,000. It will require additional working capital of $13,000. Its anticipated eight-year life will generate additional client revenue of $33,000 annually with operating costs, excluding depreciation, of $15,000. At the end of eight years, it will have a salvage value of $9,500 and return $5,000 in working capital. Taxes are not considered.

Required: (D) Horngrena. If the company has a required rate of return of 14%, what is the net present value of the

proposed investment?CMA EXAMINATION QUESTIONS Page 85 of 155

Page 86: P02 - Capital Budgeting

MANAGEMENT ADVISORY SERVICES CAPITAL BUDGETING

b. What is the internal rate of return?

Payback, NPV & IRR279. An investment opportunity costing $600,000 is expected to yield net cash flows of $120,000

annually for ten years.

Required: D, L & H 9ea. Find the NPV of the investment at a cutoff rate of 12%.b. Find the payback period of the investment.c. Find the IRR on the investment.

280. An investment opportunity costing $180,000 is expected to yield net cash flows of $60,000 annually for five years.

Required: D, L & H 9ea. Find the NPV of the investment at a cutoff rate of 12%.b. Find the payback period of the investment.c. Find the IRR on the investment.

281. Book & Bible Bookstore desires to buy a new coding machine to help control book inventories. The machine sells for $36,586 and requires working capital of $4,000. Its estimated useful life is five years and will have a salvage value of $4,000. Recovery of working capital will be $4,000 at the end of its useful life. Annual cash savings from the purchase of the machine will be $10,000.

Required: (D) Horngrena. Compute the net present value at a 14% required rate of return.b. Compute the internal rate of return.c. Determine the payback period of the investment.

282. The president of Eradicator Corp. is considering the purchase of new demolition equipment costing $100,000, with a useful life of five years and no salvage value. The new equipment would yield an annual after-tax cash flow of $29,129. An appropriate discount rate for this type of equipment is 12%. (The present value of an annuity of a dollar @ 12% for five years is 3.605. The present value of a dollar @ 12% received at the end of the fifth year is .567.)

Required: Carter & Usry(1) Compute the payback period to the nearest tenth of a year.(2) Compute the net present value to the nearest whole dollar.(3) Compute the internal rate of return on the purchase.(4) Should the equipment be purchased?

Accounting Rate of Return, Payback, NPV & IRR283. Ignore income taxes in this problem.) Ursus, Inc., is considering a project that would have a

ten-year life and would require a $1,000,000 investment in equipment. At the end of ten years, the project would terminate and the equipment would have no salvage value. The project would provide net income each year as follows:

Sales $2,000,000Less variable expenses 1,400,000Contribution margin 600,000Less fixed expenses 400,000Net income $ 200,000

All of the above items, except for depreciation of $100,000 a year, represent cash flows. The depreciation is included in the fixed expenses. The company’s required rate of return is 12%.

Required: (Moderate) N & Ga. Compute the project’s net present value.b. Compute the project’s internal rate of return, interpolating to the nearest tenth of a

percent.c. Compute the project’s payback period.d. Compute the project’s simple rate of return.

Required Investment, Payback Period & NPV284. Denali Company is evaluating a capital budgeting proposal, requiring an initial investment of

$45,000. The project will have a five-year life. The after-tax annual cash inflow from this investment is $12,000. The cost of capital is 10%. (The present value of $1 @ 10% received at the end of five years is .621. (The present value of $1 @ 10% received each year for five years is 3.791.)

Required: Carter & Usry(1) What is the payback period?(2) Compute the net present value of the project.(3) What amount should Denali have invested five years ago, at 10% compounded annually,

to have $45,000 now?

Required Investment, NPV & IRR285 (Ignore income taxes in this problem.) Prince Company’s required rate of return is 10%. The

company is considering the purchase of three machines, as indicated below. Consider each machine independently.

Required: (Moderate) N & G

CMA EXAMINATION QUESTIONS Page 86 of 155

Page 87: P02 - Capital Budgeting

MANAGEMENT ADVISORY SERVICES CAPITAL BUDGETING

a. Machine A will cost $25,00 and have a life of 15 years. Its salvage value will be $1,000, and cost savings are projected at $3,500 per year. Compute the machine’s net present value.

b. How much will Prince Company be willing to pay for Machine B if the machine promises annual cash inflows of $5,000 per year for 8 years?

c. Machine C has a projected life of 10 years. What is the machine's internal rate of return if it costs $30,000 and will save $6,000 annually in cash operating costs? Interpolate to the nearest tenth of a percent. Would you recommend purchase? Explain.

Required Investment, Profitability Index & IRR286. (Present value tables needed to answer this question.) Jane has an opportunity to invest in a

project that will yield four annual payments of $12,000 with no salvage. The first payment will be received in exactly one year. On low-risk projects of this type, Jane requires a return of 6 percent. Based on this requirement, the project generates a profitability index of 1.03953.

REQUIRED: Barfielda. How much is Jane required to invest in this project?b. What is the internal rate of return on Jane's project?

Net Cash Flow & Payback287. Marquette Company is considering the purchase of a machine with the following

characteristics.Cost $150,000Estimated useful life 10 yearsExpected annual cash cost savings $35,000

Marquette's tax rate is 40%, its cost of capital is 12%, and it will use straight-line depreciation for the new machine.

Required: D, L & H 9ea. Compute the annual after-tax cash flows for this project. b. Find the payback period for this project.

Net Cash Flow & NPV288. Tofte is considering the purchase of a machine. Data are as follows:

Cost $100,000Useful life 10 yearsAnnual straight-line depreciation $ 10,000Expected annual savings in cash operation costs $ 18,000

Tofte's cutoff rate is 12% and its tax rate is 40%.

Required: D, L & H 9e

a. Compute the annual net cash flows for the investment.b. Compute the NPV of the project.

Net Cash Flow, Payback & NPV289. Bilt-Rite Co. has the opportunity to introduce a new product. Bilt-Rite expects the product to

sell for $60 and to have per-unit variable costs of $40 and annual cash fixed costs of $3,000,000. Expected annual sales volume is 250,000 units. The equipment needed to bring out the new product costs $5,000,000, has a four-year life and no salvage value, and would be depreciated on a straight-line basis. Bilt-Rite's cost of capital is 10% and its income tax rate is 40%.

Required: D, L & H 9ea. Find the increase in annual after-tax cash flows for this opportunity. b. Find the payback period on this project.c. Find the NPV for this project.

290. Reno Company is considering the purchase of a machine with the following characteristics.Cost $160,000Estimated useful life 5 yearsExpected annual cash cost savings $56,000Expected salvage value none

Reno's tax rate is 40%, its cost of capital is 12%, and it will use straight-line depreciation for the new machine.

Required: D, L & H 9Ea. Compute the annual after-tax cash flows for this project. b. Find the payback period for this project.c. Compute the NPV for this project.

291. Whitehall Co. has the opportunity to introduce a new product. Whitehall expects the project to sell for $40 and to have per-unit variable costs of $27 and annual cash fixed costs of $1,500,000. Expected annual sales volume is 200,000 units. The equipment needed to bring out the new product costs $3,500,000, has a four-year life and no salvage value, and would be depreciated on a straight-line basis. Whitehall's cutoff rate is 10% and its income tax rate is 40%.

Required: D, L & H 9Ea. Find the increase in annual after-tax cash flows for this opportunity.b. Find the payback period on this project.c. Find the NPV for this project.

CMA EXAMINATION QUESTIONS Page 87 of 155

Page 88: P02 - Capital Budgeting

MANAGEMENT ADVISORY SERVICES CAPITAL BUDGETING

Net Cash Flow & PI292. Zmolek Company is considering the purchase of a machine costing $700,000 with a useful life

of 10 years. Annual cash cost savings are expected to be $200,000. Zmolek's income tax rate is 40% and its cost of capital is 12%. Zmolek expects to use straight-line depreciation for tax purposes.

Required: D, L & H 9Ea. Compute the expected increase in annual net cash flow for this project. b. Compute the profitability index for the project.

293. Cable Company is considering the purchase of a machine with the following characteristics.Cost $100,000Useful life 10 yearsExpected annual cash cost savings $30,000

Cable's income tax rate is 40% and its cost of capital is 12%. Cable expects to use straight-line depreciation for tax purposes.

Required: D, L & H 9Ea. Compute the expected increase in annual net cash flow for this project. b. Compute the profitability index for the project. c. How would the profitability index for this project be affected if Cable were to use MACRS

depreciation for tax purposes and the machine fell into the 7-year MACRS class? (increase decrease not affected) Circle the appropriate answer.

Net Cash Flow, NPV & PI294. Zenex is considering the purchase of a machine. Data are as follows:

Cost $240,000Useful life 10 yearsAnnual straight-line depreciation $ ???Expected annual savings in cash operation costs $ 80,000Additional working capital needed $100,000

Zenex's cutoff rate is 12% and its tax rate is 40%.

Required: D, L & H 9Ea. Compute the annual net cash flows for the investment.b. Compute the NPV of the project.c. Compute the profitability index of the project.

295. Seiler is considering the purchase of a machine. Data are as follows:Cost $2,000,000Useful life 8 years

Annual straight-line depreciation $ ???Expected annual savings in cash operation costs $ 750,000Additional working capital needed $ 500,000

Seiler's cutoff rate is 12% and its tax rate is 40%.

Required: D, L & H 9Ea. Compute the annual net cash flows for the investment.b. Compute the NPV of the project.c. Compute the profitability index of the project.

Net Cash Flow, NPV & Sensitivity Analysis296. Frank Co. has the opportunity to introduce a new product. Frank expects the product to sell

for $60 and to have per-unit variable costs of $35 and annual cash fixed costs of $4,000,000. Expected annual sales volume is 275,000 units. The equipment needed to bring out the new product costs $6,000,000, has a four-year life and no salvage value, and would be depreciated on a straight-line basis. Frank's cost of capital is 14% and its income tax rate is 40%.

Required: D, L & H 9Ea. Compute the annual net cash flows for the investment.b. Compute the NPV of the project.c. Suppose that some of the 275,000 units expected to be sold would be to customers who

currently buy another of Frank's products, the X-10, which has a $12 per-unit contribution margin. Find the sales of X-10 that can Frank lose per year and still have the investment in the new product return at least the 14% cost of capital.

d. Suppose that selling the new product has no complementary effects but that Frank's production engineers anticipate some production problems in making the new product and are not confident of the $35 estimate of per-unit variable costs for the new product. Find the amount by which Frank's estimate of per-unit variable cost could be in error and the investment still have a return at least equal to the 14% cost of capital.

Net Cash Flow, NPV & IRR297. Acme is considering the purchase of a machine. Data are as follows:

Cost $160,000Useful life 10 yearsAnnual straight-line depreciation $ ???Expected annual savings in cash operation costs $ 33,000

Acme's cutoff rate is 12% and its tax rate is 40%.

Required: D, L & H 9Ea. Compute the annual net cash flows for the investment.b. Compute the NPV of the project.

CMA EXAMINATION QUESTIONS Page 88 of 155

Page 89: P02 - Capital Budgeting

MANAGEMENT ADVISORY SERVICES CAPITAL BUDGETING

c. Compute the IRR of the project.

Annual Cash Inflow, ARR, NPV, PI & IRR298. The Sun Corp. is contemplating the acquisition of an automatic car wash. The following

information is relevant: The cost of the car wash is $160,000 The anticipated revenue from the car wash is $100,000 per annum. The useful life of the car wash is 10 years. Annual operating costs are expected to be:

Salaries $30,000Utilities 9,600Water usage 4,400Supplies 6,000Repairs/maintenance 10,000

The firm uses straight-line depreciation. The salvage value for the car wash is zero. The company's cutoff points are as follows:

Payback 3 yearsAccounting rate of return 18%Internal rate of return 18%

Ignore income taxes.

REQUIRED: Barfielda. Compute the annual cash inflow.b. Compute the net present value.c. Compute internal rate of return.d. Compute the payback period.e. Compute the profitability index.f. Should the car wash be purchased?

PROJECT EVALUATION & RANKING299. EIF Manufacturing Company needs to overhaul its drill press or buy a new one. The facts

have been gathered, and are as follows:Current Machine New Machine

Purchase Price, New $80,000 $100,000Current book value 30,000Overhaul needed now 40,000Annual cash operating costs 70,000 40,000Current salvage value 20,000Salvage value in five years 5,000 20,000

Required: (D) HorngrenWhich alternative is the most desirable with a current required rate of return of 20%? Show computations, and assume no taxes.

300. Gavin and Alex, baseball consultants, are in need of a microcomputer network for their staff. They have received three proposals, with related facts as follows:

Proposal A Proposal B Proposal CInitial investment in equipment $90,000 $90,000 $90,000Annual cash increase in operations: Year 1 80,000 45,000 90,000 Year 2 10,000 45,000 0 Year 3 45,000 45,000 0Salvage value 0 0 0Estimated life 3 yrs 3 yrs 1 yr

The company uses straight-line depreciation for all capital assets.

Required: (D) Horngrena. Compute the payback period, net present value, and accrual accounting rate of return with

initial investment, for each proposal. Use a required rate of return of 14%.b. Rank each proposal 1, 2, and 3 using each method separately. Which proposal is best?

Why?

301. (Present value tables needed to answer this question.) The Ruth Company has been operating a small lunch counter for the convenience of employees. The counter occupies space that is not needed for any other business purpose. The lunch counter has been managed by a part-time employee whose annual salary is $3,000. Yearly operations have consistently shown a loss as follows:

Receipts $20,000 Expenses for food, supplies (in cash) $19,000Salary 3,000 22,000 Net Loss $(2,000)

A company has offered to sell Ruth automatic vending machines for a total cost of $12,000. Sales terms are cash on delivery. The old equipment has zero disposal value.The predicted useful life of the equipment is 10 years, with zero scrap value. The equipment will easily serve the same volume that the lunch counter handled. Z catering company will completely service and supply the machines. Prices and variety of food and drink will be the same as those that prevailed at the lunch counter. The catering company will pay 5 percent of gross receipts to the Ruth Company and will bear all costs of food, repairs, and so forth. The part-time employee will be discharged. Thus, Ruth's only cost will be the initial outlay for the machines.

CMA EXAMINATION QUESTIONS Page 89 of 155

Page 90: P02 - Capital Budgeting

MANAGEMENT ADVISORY SERVICES CAPITAL BUDGETING

Consider only the two alternatives mentioned.

REQUIRED: Barfielda. What is the annual income difference between alternatives?b. Compute the payback period.c. Compute:

1. The net present value if relevant cost of capital is 20 percent.2. Internal rate of return.

d. Management is very uncertain about the prospective revenue from the vending equipment. Suppose that the gross receipts amounted to $14,000 instead of $20,000. Repeat the computation in part c.1.

e. What would be the minimum amount of annual gross receipts from the vending equipment that would justify making the investment? Show computations.

Payback, Net Present Value & Profitability Index302. (Present value tables needed to answer this question.) XYZ Co. is interested in purchasing a

state-of-the-art widget machine for its manufacturing plant. The new machine has been designed to basically eliminate all errors and defects in the widget-making production process. The new machine will cost $150,000, and have a salvage value of $70,000 at the end of its seven-year useful life. XYZ has determined that cash inflows for years 1 through 7 will be as follows: $32,000; $57,000; $15,000; $28,000; $16,000; $10,000, and $15,000, respectively. Maintenance will be required in years 3 and 6 at $10,000 and $7,000 respectively. XYZ uses a discount rate of 11 percent and wants projects to have a payback period of no longer than five years.

REQUIRED: Barfielda. Compute the net present value of the new machine.b. Compute the firm's profitability index.c. Compute the payback period.d. Evaluate this investment proposal for XYZ Co.

Point Of Indifference303. (Present value tables needed to answer this question.) Managers of the Jonathan Co. realize

that the present value of the depreciation tax benefit is affected by the discount rate, the tax rate, and the depreciation rate. They have recently purchased a machine for $100,000 and they are trying to decide which depreciation method to use. There are only two alternatives available, and they must make an irrevocable selection of one method or the other right now. They have no uncertainty about the company's discount rate (it is 10 percent), but they are highly uncertain about the direction of future tax rates. The company's uncertainty stems from the fact that the existing tax rate is 30 percent, but congress is presently debating tax

legislation that would dramatically increase the rate. If the legislation is passed it would go into affect in two years (after the Jonathan Co. has claimed two years of depreciation).

Method 1 Method 2 DifferenceYear 1 $30,000 $10,000 $(20,000)Year 2 $40,000 $15,000 $(25,000)Year 3 $10,000 $25,000 $ 15,000 Year 4 $10,000 $25,000 $ 15,000 Year 5 $10,000 $25,000 $ 15,000

REQUIRED: BarfieldHow high would tax rates need to be in two years for the Jonathan Co. to be indifferent between depreciation Method 2 and depreciation Method 1 below?

.304. XL Corp. is considering an investment that will require an initial cash outlay of $200,000 to purchase non-depreciable assets. The project promises to return $60,000 per year (after-tax) for eight years with no salvage value. The company's cost of capital is 11 percent.(Present value tables needed to answer this question.) The company is uncertain about its estimate of the life expectancy of the project.

REQUIRED: BarfieldHow many years must the project generate the $60,000 per year return for the company to at least be indifferent about its acceptance? (Do not consider the possibility of partial year returns.) Barfield

INFLATIONEffect of Inflation on Investment Decision305. Ranchero Company is evaluating a capital budgeting proposal that will require an initial cash

investment of $100,000. The project will have a 3-year life. The net after-tax cash inflows from the project, before any adjustment for the effects of inflation, are expected to be as follows:

Year Unadjusted Estimate of Cash Inflows 1 $50,000 2 40,000 3 30,000No salvage is expected at the end of the project. The anticipated inflation rate is 10% each year. The company's cost of capital rate is 16%.

Required: Carter & Usry

(1) Compute the estimated cash inflow for each year, adjusted for the anticipated effect of inflation.

CMA EXAMINATION QUESTIONS Page 90 of 155

Page 91: P02 - Capital Budgeting

MANAGEMENT ADVISORY SERVICES CAPITAL BUDGETING

(2) Determine the net present value of the cash flows before and after the adjustment for the anticipated effects of inflation.

(The present values of $1 @ 16% at the end of years 1, 2, and 3 respectively are .862, .743, and .641. The present value of an annuity of $1 @ 16% for 3 years is 2.246.)

Estimating Pretax Cash Inflows With Inflation306. Speedi Corporation is considering a capital expenditure proposal which will require an initial

cash outlay of $50,000. The project life is expected to be 6 years. The estimated salvage value for the equipment (based on today's market price for similar used 6-year old equipment) is $2,500. Estimated annual net cash inflows from operations during the life of the project follow:

Year Estimated Annual Cash Inflow1 $10,0002 15,0003 15,0004 15,0005 10,0006 5,000

Required: Carter & UsryCompute the excess of cash inflows over cash outflows assuming management expects a constant 4% rate of inflation during the 6-year period. (Round your price level index to three decimal places.)

Effect of Inflation and Taxes on Investment Decision307. Weighout Company is evaluating a capital expenditure proposal that will require an initial cash

investment of $100,000. The project will have a 6-year life; however, the property will qualify as 5-year property for income-tax depreciation purposes. The income tax rate is 40%. The annual cash inflows from the project, before any adjustment for the effects of inflation or income taxes, are expected to be as follows:

Year Unadjusted Estimate of Cash Inflows1 $25,0002 27,0003 29,0004 23,0005 20,0006 15,000

The expected salvage value of the property is zero. Cash inflows are expected to increase at the anticipated inflation rate of 4% each year.

Required: Carter & Usry

Compute the inflation adjusted after-tax cash inflow from the proposal for each year, and the excess of total net cash inflows over the initial cash outlay. (Use the MACRS depreciation rates provided below to compute tax depreciation, and round the price-level index to three decimal places.)

Year MACRS 5-year Recovery Rate1 0.2002 0.3203 0.1924 0.1155 0.1156 0.058

1.000

COST OF CAPITAL308. Molloy Company wishes to compute a weighted-average cost of capital for use in evaluating

capital expenditure proposals. Earnings, capital structure, and current market prices of the company's securities are:

Earnings:Earnings before interest and tax $400,000Interest expense on bonds 100,000Pretax earnings $300,000Income tax (40%) 120,000After-tax earnings $180,000Preferred stock dividends 75,000Earnings available to common stockholders $105,000Common stock dividends 50,000Retained earnings $ 55,000

Capital structure:Mortgage bonds, 12%, 20 years $500,000Preferred stock, 15%, $100 par 500,000Common stock, no par, 25,000 shares 300,000Retained earnings (equity of common stockholders) 700,000

$ 2,000,000

Market price of the company's securities:Preferred stock $100Common stock 30

Required: Carter & Usry

CMA EXAMINATION QUESTIONS Page 91 of 155

Page 92: P02 - Capital Budgeting

MANAGEMENT ADVISORY SERVICES CAPITAL BUDGETING

Determine the company's cost of capital to the nearest hundredth of a percent.

CMA EXAMINATION QUESTIONS Page 92 of 155

Page 93: P02 - Capital Budgeting

MANAGEMENT ADVISORY SERVICES CAPITAL BUDGETING

8 . Answer (C) is correct. For capital budgeting purposes, the net investment is the net outlay or cash requirement. This amount includes the cost of the new equipment, minus any cash recovered from the trade or sale of existing assets. The investment required also includes funds to provide for increases in working capital, for example, the additional receivables and inventories resulting from the acquisition of a new manufacturing plant. The investment in working capital is treated as an initial cost of the investment, although it will be recovered at the end of the project (its salvage value equals its initial cost). For Kline, the additional current assets will be 30% of sales, but current liabilities can be used to fund assets to the extent of 10% of sales. Thus, the initial investment in working capital will equal 20% of the $6 million in sales, or $1,200,000. The total initial cash outlay will consist of the $8 million in new equipment plus $1,200,000 in working capital, a total of $9.2 million. Answer (A) is incorrect because $6.8 million subtracted the net investment in working capital from the cost of the equipment. Answer (B) is incorrect because $8.6 million assumes current assets will increase by 10% of new sales but that current liabilities will not change. Answer (D) is incorrect because $9.8 million ignores the financing of incremental current assets with accounts payable.

10 . Answer (A) is correct. The concept of present value gives greater value to inflows received earlier in the stream. Thus, the declining inflows would be superior to increasing inflows, or even inflows. Answer (B) is incorrect because it involves lower inflows in the earlier years. Answer (C) is incorrect because it involves lower inflows in the earlier years. Answer (D) is incorrect because it involves lower inflows in the earlier years.

11 . Answer (C) is correct. The increase in pre-tax net profit is 140,000 (400,000 cash sales increase - 180,000 nondepreciation expenses increase - 80,000 depreciation). Thus, taxes will increase by 47,600 (34% x 140,000), and the increase in net cash inflows will be 172,400 (400,000 - 180,000 - 47,600). Answer (A) is incorrect because 92,400 equals the increase in after-tax net profit. Answer (B) is incorrect because 140,000 is the increase in pre-tax net profit. Answer (D) is incorrect because 220,000 equals cash sales minus expenses other than depreciation.

12 . Answer (D) is correct. Depreciation expense is not a cash outflow and is not considered in the cash analysis except for its effects on taxes paid. Thus, the expected annual cash flow is $14,000 {(1.0 - .4 tax rate)[$50,000 cash sales - $10,000 fixed cost - (.4 x $50,000) variable costs - $5,000 depreciation] + $5,000 depreciation}. Answer (A) is incorrect because $15,000 is the pre-tax income from the project. Answer (B) is incorrect because $9,000 includes depreciation as a cash outflow. Answer (C) is incorrect because $19,000 equals expected annual cash flow plus depreciation.

13 . $50,000 x 5 = $250,000 x (1- 0.4) = $150,000 net cash flow

14 . Answer (B) is correct. The project will have an $11,000 before-tax cash inflow from operations in the tenth year ($40,000 - $29,000). Also, $9,000 will be generated from the sale of the equipment. The entire $9,000 will be taxable because the basis of the asset was reduced to zero in the 7th year. Thus, taxable income will be $20,000 ($11,000 + $9,000), leaving a net after-tax cash inflow of $12,000 [(1.0 - .4) x $20,000]. To this $12,000 must be added the $12,000 tied up in working capital ($7,000 + $5,000). The total net cash flow in the 10th year will therefore be $24,000. Answer (A) is incorrect because $32,000 omits the $8,000 outflow for income taxes. Answer (C) is incorrect because taxes will be $8,000, not $12,000. Answer (D) is incorrect because $11,000 is the net operating cash flow.

CMA EXAMINATION QUESTIONS Page 93 of 155

Page 94: P02 - Capital Budgeting

MANAGEMENT ADVISORY SERVICES CAPITAL BUDGETING

15 . Answer (C) is correct. The tax basis of $75,000 and the $40,000 cost to remove can be written off. However, the $10,000 scrap value is a cash inflow. Thus, the taxable loss is $105,000 ($75,000 loss on disposal + $40,000 expense to remove - $10,000 of inflows). At a 40% tax rate, the $105,000 loss will produce a tax savings (inflow) of $42,000. The final cash flows will consist of an outflow of $40,000 (cost to remove) and inflows of $10,000 (scrap) and $42,000 (tax savings), or a net inflow of $12,000. Answer (A) is incorrect because $45,000 ignores income taxes and assumes that the loss on disposal involves a cash inflow. Answer (B) is incorrect because $27,000 assumes that the loss on disposal involves a cash inflow. Answer (D) is incorrect because $(18,000) ignores the tax loss on disposal.

16 . Answer (C) is correct. The tax basis of $150,000 and the $80,000 cost to remove are deductible expenses, but the $20,000 scrap value is an offsetting cash inflow. Thus, the taxable loss is $210,000 ($150,000 + $80,000 - $20,000). At a 40% tax rate, the $210,000 loss will produce a tax savings (inflow) of $84,000. Accordingly, the final cash flows will consist of an outflow of $80,000 (cost to remove) and inflows of $20,000 (scrap) and $84,000 (tax savings), a net inflow of $24,000. Answer (A) is incorrect because $90,000 assumes that the loss on disposal is a cash inflow. It also ignores income taxes. Answer (B) is incorrect because $54,000 assumes that the loss on disposal involves a cash inflow. Answer (D) is incorrect because $(36,000) assumes that the tax basis is $0.

17 . $50,000 x 40% = $20,000

18 . $80,000 x 40% = $32,000

19

Additional depreciation on the new machine$(40,000)Loss on sale of old machine(45,000)Operating cost savings 125,000Increase in income$ 40,00020 . (A) ($8,000 x .97087) + ($12,000 x .94260) + ($10,000 x .91514) + ($15,000 x .88849) =

$7,767 + 11,311 + 9,151 + 13,327 = $41,556

21 . $27,000 x 0.641 (PVIF, n = 3, 16%) = $17,307

22 . $6,000 x 0.404 (PVIF, n = 8, 12%) = $2,424

23.Revenues$48,100Less: Variable costs(16,000)Fixed out-of-pocket costs(10,000)Annual cash inflows$22,000PVAF, n = 8, 12%x 4.968Present value$109,296

24 . Answer (D) is correct. Initially, the company must invest $105,000 in the machine, consisting of the invoice price of $90,000, the delivery costs of $6,000, and the installation costs of $9,000. Answer (A) is incorrect because $(85,000) erroneously includes salvage value but ignores delivery and installation costs. Answer (B) is incorrect because $(90,000) ignores the outlays needed for delivery and installation costs, both of which are an integral part of preparing the new asset for use. Answer (C) is incorrect because $(96,000) fails to include installation costs in the total.

CMA EXAMINATION QUESTIONS Page 94 of 155

Page 95: P02 - Capital Budgeting

MANAGEMENT ADVISORY SERVICES CAPITAL BUDGETING

26 . Answer (D) is correct. The company will receive net cash inflows of $50 per unit ($500 selling price - $450 of variable costs), or a total of $100,000 per year. This amount will be subject to taxation, as will the $5,000 gain on sale of the investment, bringing taxable income to $105,000. No depreciation will be deducted in the tenth year because the asset was fully depreciated after 5 years. Because the asset was fully depreciated (book value was zero), the $5,000 salvage value received would be fully taxable. After income taxes of $42,000 ($105,000 x 40%), the net cash flow in the tenth year is $63,000 ($105,000 - $42,000). Answer (A) is incorrect because $100,000 overlooks the salvage proceeds and the taxes to be paid. Answer (B) is incorrect because $81,000 miscalculates income taxes. Answer (C) is incorrect because $68,400 assumes that depreciation is deducted; it also overlooks the receipt of the salvage proceeds.

27 . Answer (D) is correct. Delivery and installation costs are essential to preparing the machine for its intended use. Thus, the company must initially pay $210,000 for the machine, consisting of the invoice price of $180,000, the delivery costs of $12,000, and the $18,000 of installation costs. Answer (A) is incorrect because $(170,000) includes salvage value and ignores delivery and installation costs. Answer (B) is incorrect because $(180,000) ignores the outlays needed for delivery and installation. Answer (C) is incorrect because $(192,000) excludes installation costs.

28 . Answer (A) is correct. The company will receive net cash inflows of $50 per unit ($500 selling price - $450 variable costs), a total of $200,000 per year for 4,000 units. This amount will be subject to taxation. However, for the first 5 years, a depreciation deduction of $42,000 per year ($210,000 cost ÷ 5 years) will be available. Thus, annual taxable income will be $158,000 ($200,000 - $42,000). At a 40% tax rate, income tax expense will be $63,200, and the net cash inflow will be $136,800 ($200,000 - $63,200). Answer (B) is incorrect because $136,000 results from subtracting salvage value when calculating depreciation expense. Answer (C) is incorrect because $128,400 assumes depreciation is recognized over 10 years. Answer (D) is incorrect because $107,400 assumes that depreciation is recognized over 10 years and that it requires a cash outlay.

29 . Answer (D) is correct. The company will receive net cash inflows of $50 per unit ($500 selling price - $450 of variable costs), a total of $200,000 per year for 4,000 units. This amount will be subject to taxation, as will the $10,000 gain on sale of the investment, resulting in taxable income of $210,000. No depreciation will be deducted in the tenth year because the asset was fully depreciated after 5 years. Because the asset was fully depreciated (book value was $0), the $10,000 received as salvage value is fully taxable. At 40%, the tax on $210,000 is $84,000. After subtracting $84,000 of tax expense from the $210,000 of inflows, the net inflows amount to $126,000. Answer (A) is incorrect because $200,000 overlooks the salvage proceeds and the taxes to be paid. Answer (B) is incorrect because $158,000 equals annual taxable income for each of the first 5 years. Answer (C) is incorrect because $136,800 is the annual net cash inflow in the second through the fifth years.

30 . Answer (B) is correct. When annual cash inflows are uniform, the payback period is calculated by dividing the initial investment ($210,000) by the annual net cash inflows ($136,800). Dividing $210,000 by $136,800 produces a payback period of 1.54 years. Answer (A) is incorrect because 1.05 years fails to subtract income taxes. Answer (C) is incorrect because 1.33 years includes taxable income in the denominator instead of cash flows. Answer (D) is incorrect because 2.22 years subtracts depreciation from cash flows.

31 . Answer (C) is correct. The accounting rate of return method (ARR) computes an approximate rate of return which ignores the time value of money. It is computed as follows:ARR = expected increase in net income Average investment.

CMA EXAMINATION QUESTIONS Page 95 of 155

Page 96: P02 - Capital Budgeting

MANAGEMENT ADVISORY SERVICES CAPITAL BUDGETING

25 . Answer (A) is correct. The company will receive net cash inflows of $50 per unit ($500 32 . [$16,000 – ($80,000/8)]/$80,000 = 7.5%

33 . {$12,000 – [($30,000 - $3,000)/5)]}/$30,000 = 22%

34 . Net Income After Tax (P280,000 x 20%) P 56,000Divide by (1 – 0.30) 0.70Net Income before Tax P 80,000Add Depreciation 35,000Cash Flow before Tax P115,000

35 . Net Income after Tax 7.5% of InvestmentDepreciation 10.0% of InvestmentAfter-tax Cash Flow 17.5% of Investment = P175,000Investment (P175,000 17.5%) = P1,000,000

36 . Answer (B) is correct. Payback is the number of years required to complete the return of the original investment. Given a periodic constant cash flow, the payback period equals net investment divided by the constant expected periodic after-tax cash flow. The desired payback period is 4 years, so the constant after-tax annual cash flow must be $90,000 ($360,000 ÷ 4). Assuming that the company has sufficient other income to permit realization of the full tax savings, depreciation of the machine will shield $60,000 ($360,000 ÷ 6) of income from taxation each year, an after-tax cash savings of $24,000 (40% x $60,000). Thus, the machine must generate an additional $66,000 ($90,000 - $24,000) of after-tax cash savings from operations. This amount is equivalent to $110,000 [$66,000 ÷ (1.0 - .4)] of before-tax operating cash savings. Answer (A) is incorrect because $90,000 is the total desired annual after-tax cash savings. Answer (C) is incorrect because $114,000 results from adding, not subtracting, the $24,000 of tax depreciation savings to determine the minimum annual after-tax operating savings. Answer (D) is incorrect because $150,000 assumes that depreciation is not tax deductible.

37 . Answer (D) is correct. The payback period is calculated by dividing cost by the annual cash inflows, or cash savings. To achieve a payback period of 3 years, the annual increment in net cash inflow generated by the investment must be $150,000 ($450,000 ÷ 3-year targeted payback period). This amount equals the total reduction in cash operating costs minus related taxes. Depreciation is $90,000 ($450,000 ÷ 5 years). Because depreciation is a noncash deductible expense, it shields $90,000 of the cash savings from taxation. Accordingly, $60,000 ($150,000 - $90,000) of the additional net cash inflow must come from after-tax net income. At a 40% tax rate, $60,000 of after-tax income equals $100,000 ($60,000 ÷ 60%) of pre-tax income from cost savings, and the outflow for taxes is $40,000. Thus, the annual reduction in cash operating costs required is $190,000 ($150,000 additional net cash inflow required + $40,000 tax outflow). Answer (A) is incorrect because $60,000 is after-tax net income from the cost savings. Answer (B) is incorrect because $100,000 is the pre-tax income from the cost savings. Answer (C) is incorrect because $150,000 ignores the impact of depreciation and income taxes.

CMA EXAMINATION QUESTIONS Page 96 of 155

Page 97: P02 - Capital Budgeting

MANAGEMENT ADVISORY SERVICES CAPITAL BUDGETING

Therefore, $40,000 (as stated in problem) is the expected increase in annual income.

selling price - $450 of variable costs), or a total of $100,000 per year. This amount will be subject to taxation, but, for the first 5 years, there will be a depreciation deduction of $21,000 per year ($105,000 cost divided by 5 years). Therefore, deducting the $21,000 of depreciation expense from the $100,000 of contribution margin will result in taxable income of $79,000. After income taxes of $31,600 ($79,000 x 40%), the net cash flow in the third year is $68,400

38 . $1,000,000/$200,000 = 5.0 years

39 . REQUIRED: The payback period in years.DISCUSSION: (A) The payback period equals the original investment divided by the constant net cash inflow per year. Net cash inflow per year is $2,000, and the payback period is 2.5 years ($5,000 investment ÷ $2,000).Annual revenue$10,000CGS (60%) (6,000)Cash flow before fixed costs$4,000Fixed costs (2,000)Annual net cash inflow$ 2,000Answer (B) is incorrect because 5.00 assumes a $10,000 investment. Answer (C) is incorrect because 2.00 assumes a $10,000 investment and a $5,000 constant annual net cash inflow. Answer (D) is incorrect because 1.25 equals $5,000 divided by $4,000.

40 . Step 1: Calculate the PV of the cash flows:PVA5 = $60,000(PVIFA10%,5) = $60,000(3.7908) = $227,448.

Step 2: Calculate the Year 0 outflow:The outflow at t = 0 is X where $227,448 - X = $75,000. X or CF0 = -$152,448.

Step 3: Calculate the regular payback:YearCFCumulative CF 0-$152,448-$152,448 160,000-92,448 260,000-32,448 360,00027,552 460,00087,552 560,000147,552

So the payback is 2 + = 2.54 years.

41 . $600,000/$200,000 = 3 years

42 . $15,000 + $20,000 + $25,000 = $60,000 or 3 years

CMA EXAMINATION QUESTIONS Page 97 of 155

Page 98: P02 - Capital Budgeting

MANAGEMENT ADVISORY SERVICES CAPITAL BUDGETING

43. Payback period Answer: b Diff: E

Using the even cash flow distribution assumption, the project will completely recover initial investment after 30/35 = 0.86 of Year 5:

Payback = 4 + = 4.86 years.44

?. Payback period Answer: c Diff: M

Payback = 5 + = 5.928 years 6 years.

45 . REQUIRED: The payback period given the annual cash flows.DISCUSSION: (B) Because $700 will be received in the first 2 years, only $300 will remain to be recovered in year 3. The payback period is therefore 2.6 years [2 years + ($300 + $500) of year 3].Answer (A) is incorrect because after 2 years $300 ($1,000 - $700) remains to be recovered. Answer (C) is incorrect because the payback period is determined by adding 2 plus the percentage of year 3 cash flows necessary to recover the balance of the $1,000 original investment. Answer (D) is incorrect because, at the end of 3 years, $200 ($1,200 - $1,000) more than the original cost will have recovered.

46 . 2 + ($8,000/$15,000) = 2.53

47 . 4 + $10,000/$30,000 = 4.33

48 . $9,000 + $12,000 + $15,000 + $9,000 = $45,000 or 4 yearsCMA EXAMINATION QUESTIONS Page 98 of 155

Page 99: P02 - Capital Budgeting

MANAGEMENT ADVISORY SERVICES CAPITAL BUDGETING

($100,000 - $31,600). 50 . $200,000/($125,000 - $75,000 + $15,000) = 3.1

51 . Answer (A) is correct. The payback period for an investment, ignoring the time value of money, can be found by accumulating each year's net cash flows until the initial investment is recovered. The amount accumulated after 3 years is $450,000. Thus, 50% of year 4 cash flows is needed to recover the initial investment. The payback period is 3.5 years. Answer (B) is incorrect because 4 years includes the additional $50,000 of Year 4. Answer (C) is incorrect because 4.2 years takes the average inflow of all 8 years and divides that into the $500,000 initial investment. Answer (D) is incorrect because 5 years uses only the cash flows from the remaining 5 years.

52 . Answer (A) is correct. The payback reciprocal for an investment is found by dividing 1 by the payback time. The payback time for this investment is 3.5 years, and the payback reciprocal is 1 divided by 3.5, or 29%. Answer (B) is incorrect because 25% includes the additional $50,000 of Year 4 in the payback time. Answer (C) is incorrect because 24% takes the average inflow of all 8 years and divides that into the $500,000 initial investment in the payback time. Answer (D) is incorrect because 20% uses only the cash flows from the remaining 5 years in the payback time.

53 . Answer (C) is correct. The payback period for an investment, ignoring the time value of money, can be found by accumulating each year's net cash flows until the initial investment is recovered. Therefore, dividing the $500,000 initial investment by the annual $130,000 inflow gives a payback time of 3.85 years. Answer (A) is incorrect because 3.85 years is the length of the payback period. Answer (B) is incorrect because 3.85 years is the length of the payback period. Answer (D) is incorrect because 4 years does not prorate the final $130,000.

54 . REQUIRED: The bailout period for an investment in a new machine.DISCUSSION: (C) The bailout period is the time required for the sum of the cumulative net cash inflow and the salvage value to equal the original investment. During years 1, and 2, cost minus salvage value is $70,000, and the annual net cash inflow is $36,000. Hence, the incremental amount to be recovered during year 2 is $34,000 ($70,000 – $36,000). Interpolating in year 2 therefore yields a bailout period of 1.9 years [1 + ($34,000 ÷ $38,000)].Answer (A) is incorrect because the annual cash flows are $36,000, not $50,000. Answer (B) is incorrect because cost minus salvage value in year 2 is $70,000, not $80,000. Answer (D) is incorrect because the incremental amount to be recovered during year 2 is $34,000 not $14,000.

55 . The PV of the outflows is -$700 million. To find the discounted payback you need to keep adding cash flows until the cumulative PVs of the cash inflows equal the PV of the outflow:

YearCash FlowDiscounted Cash FlowCumulative PV0-$700 million-$700.0000-$700.00001200 million181.8182-518.18182370 million305.7851-212.39673225 million169.0458-43.35094700 million478.1094434.7585The payback occurs somewhere in Year 4. To find out exactly where we calculate $43.3509/$478.1094 = 0.0907 through the year. Therefore, the discounted payback is 3.091

years.

CMA EXAMINATION QUESTIONS Page 99 of 155

Page 100: P02 - Capital Budgeting

MANAGEMENT ADVISORY SERVICES CAPITAL BUDGETING

56.Discounted

Year Cash Flow Cash Flow Cumulative PV0 -$3,000 -3,000.00 -3,000.001 1,000 909.09 -2,090.912 1,000 826.45 -1,264.463 1,000 751.31 -513.154 1,000 683.01 169.86

After Year 3, you can see that you won’t need all of Year 4 cash flows to break even. To find the portion that you need, calculate $513.15/ $683.01 = 0.75. Therefore, the discounted payback is 3.75 years.

57 .

Discounted CF Cumulative CF0 -200,000.00 -200,000.001 45,454.55 -154,545.452 82,644.63 -71,900.82

Payback3 112,697.22 +40,796.404 27,320.54 +68,116.945 15,523.03 +83,639.97

Payback period = 2 years + = 2.638 years.58

CMA EXAMINATION QUESTIONS Page 100 of 155

Page 101: P02 - Capital Budgeting

MANAGEMENT ADVISORY SERVICES CAPITAL BUDGETING

61 . 3.037 x ACI - $800,000 = $0 = $ 263,418

62 . REUIRED: To determine the present value of the future cash savings resulting from purchase of the new machine.Answer (C) is correct. The present value of the $30,000 savings per year for the first 2 years is calculated using the present value of an annuity for 2 periods. Since the amount of the cash savings drops to $20,000 in year 3, this amount must be calculated separately. The PV of an annuity for 2 periods, equals the PV of an amount to be received 3 years in the future. The total present value of the cash savings is calculated as follows:PV of $30,000 for 2 periods = $30,000 x 1.65 = $49,500PV of $20,000 in period 3 = $20,000 x (2.32 – 1.65) = 13,400Total pesent value of cash savings $62,900 Alternatively, $20,000 could have been treated as an annuity for 3 years and an additional $10,000 for 2 years.

63 . REQUIRED: The present value of the cash flow generated by the project.DISCUSSION: (B) If the cash inflow, net of taxes, at the end of each of 7 years is $5,000, and if the discount rate is 12%, the present value of this series of cash flows will e equal to the present value of an ordinary annuity of $5,000 for 7 years at 12%. The interest factor for the present value of an ordinary annuity is equal to the sum of the interest factors for the present value of $1 for the same period. The interest factor for an ordinary annuity of $5,000 for seven periods is 4.564. The present value is $22,820 ($5,000 x 4.564).The alternative is to calculate the present value of each $5,000 cash flow using the interest factor for the present value of $1 at 12% for each of the periods one though seven. The sum of these products is equal to the present value of an ordinary annuity of $5,000 for seven periods at 12%.

$5,000 x 0.893 = $ 4,465 5,000 x 0.797 = 3,985 5,000 x 0.712 = 3,560 5,000 x 0.636 = 3,180 5,000 x 0.567 = 2,835 5,000 x 0.507 = 2,535 5,000 x 0.452 = 2,260 5,000 x 4.565 = $22,820

Answers (A), (C), and (D) are incorrect because the present value of the cash flow is equal to the annual inflow times the sum of the present value factors.

64

?. NPV, payback, and missing cash flow Answer: b Diff: MFirst, find the missing t = 0 cash flow. If payback = 2.5 years, this implies t = 0 cash flow must be -$2,000 - $3,000 + (0.5)$3,000 = -$6,500.

CMA EXAMINATION QUESTIONS Page 101 of 155

Page 102: P02 - Capital Budgeting

MANAGEMENT ADVISORY SERVICES CAPITAL BUDGETING

65

?. NPV Answer: d Diff: M

Tabular solution:NPV = $30,000(PVIFA10%,4) + $35,000(PVIFA10%,5)(PVIF10%,4) + $40,000(PVIF10%,10) - $150,000 = $30,000(3.1699) + $35,000(3.7908)(0.6830) + $40,000(0.3855) - $150,000 = $51,136.07 $51,136.

Financial calculator solution (in thousands):Inputs: CF0 = -150; CF1 = 30; Nj = 4; CF2 = 35; Nj = 5; CF3 = 40; I = 10.Output: NPV = $51.13824 = $51,138.24 $51,138.

66

?. NPV Answer: b Diff: M

Tabular solution:PV = $5,000(PVIFA14%,5) + $3,000(PVIFA14%,3)(PVIF14%,5) + $2,000(PVIFA14%,2)(PVIF14%,8) = $5,000(3.4331) + $3,000(2.3216)(0.5194) + $2,000(1.6467)(0.3506) = $17,165.50 + $3,617.52 + $1,154.67 = $21,937.69.

Financial calculator solution (in thousands):Inputs: CF0 = 0; CF1 = 5; Nj = 5; CF2 = 3; Nj = 3; CF3 = 2; Nj = 2; I = 14.

CMA EXAMINATION QUESTIONS Page 102 of 155

Page 103: P02 - Capital Budgeting

MANAGEMENT ADVISORY SERVICES CAPITAL BUDGETING

Output: NPV = 21.93726 = $21,937.26.72.

- $32,000 - $160,000 + $16,000= $(176,000)Yr 1 = $72,000 x 0.833= 59,976Yr 2 = $72,000 x 0.694= 49,968Yr 3 = $72,000 x 0.579= 41,688Yr 4 = $72,000 x 0.482= 34,704$ 10,33673 . Answer (C) is correct. The problem requires finding the present value of $500,000 a year for each of 5 years, using the discount rate of 14 percent. The cash outlay is then deducted from the present

value of that stream of cash to arrive at the NPV. NPV = -1,500,000 + (t=1 to 5) $500,000/(1.14)t = -$1,500,000 + ($500,000 x 3.433) = -$1,500,000 + $1,716,500 = $216,500. Answer (A) is incorrect because ($43,150) would be the result of using 2.9137 as the annuity factor, which is actually the factor for 14% at 4 periods (not 5). Answer (B) is incorrect because $108,250 is half of the correct NPV. Answer (D) is incorrect because this NPV would be the result of using 3.8887 as the annuity factor, which is actually the factor for 14% at 6 periods (not 5).

74 . ($70,000 x 4.111) - $240,000 = $47,770

75 . Answer (C) is correct. Annual cash outflow for taxes is $12,000 {[$70,000 inflows - $20,000 cash operating expenses - ($100,000 ÷ 5) depreciation] x 40%}. The annual net cash inflow is therefore $38,000 ($70,000 - $20,000 - $12,000). The present value of these net inflows for a 5-year period is $136,990 ($38,000 x 3.605 present value of an ordinary annuity for 5 years at 12%), and the NPV of the investment is $36,990 ($136,990 - $100,000 investment). Answer (A) is incorrect because $28,840 is the present value of the depreciation tax savings. Answer (B) is incorrect because $8,150 ignores the depreciation tax savings. Answer (D) is incorrect because $80,250 ignores taxes.

76 . Answer (B) is correct. The following table derives the cash flows and NPV.Item Year 0 Years 1 to 5Investment -80,000,000Revenue144,000,000Variable cost 84,000,000Fixed cost 25,000,000Depreciation 16,000,000Pre-tax profit19,000,000Tax @ 36% 6,840,000Net profit 12,160,000Net cash flow 28,160,000Present value @ 12% (28,160,000 x 3.6048)101,511,160NPV = 21,511,160.Answer (A) is incorrect because $17,225,000 results from failing to deduct depreciation in calculating taxes. Answer (C) is incorrect because $26,780,000 results from failing to consider that depreciation is a noncash expense. Answer (D) is incorrect because $56,117,000 is based on annual sales of 15,000 units, rather than 12,000 units.

77 . Cash flow = ($15,000 x 0.6) + ($30,000/5 x 0.4) = $11,400NPV = $11,400 x 3.127 - $30,000 = $5,648

78 . Answer (B) is correct. The first step is to calculate the annual cash flows from the project for the base case (the expected values). These may be calculated as shown: DESCRIPTION HOW CALCULATEDVALUE ($ in millions)1. Revenues90,000 x 0.30 x $800 21.6002. Variable cost 90,000 x 0.30 x $400 10.8003. Fixed cost $4,000,000 4.0004. Depreciation $20,000,000 ÷ 8 2.5005. Pretax profit Item 1 - (Items 2 + 3 + 4) 4.3006. Tax Item 5 x 0.35 1.5057. Net profit Item 5 - Item 6 2.7958. Net cash flow Item 7 + Item 4 5.295This level of cash flow occurs for each of the 8 years of the project. The present value of an 8-year, $1 annuity is 4.639 at 14%. The NPV of the project is therefore given by:

NPV = $5,295,000 x 4.639 - $20,000,000 = $4,563,505Answer (A) is incorrect because $2,626,415 used the wrong discount factor. Answer (C) is incorrect because it failed to consider depreciation. Answer (D) is incorrect because it failed to consider depreciation and other fixed costs.

CMA EXAMINATION QUESTIONS Page 103 of 155

Page 104: P02 - Capital Budgeting

MANAGEMENT ADVISORY SERVICES CAPITAL BUDGETING

80 . DISCUSSION: The net present value of a project equals:NPV = (PV of future cash flows) – (Investments)

Since this problem involves a lease requiring only annual payments there is no initial investment in this case: Lease amortization must be subtracted from cash inflows to determine income tax expense.$7,,500Annual cash inflow – 5,000 Tax basis lease amortization$2,500Taxable lease income x 40%$1,000Tax expense per yearHowever, lease amortization is not a cash outflow and it thus excluded from the calculation of NPV. The after-tax present value of the lease equals:

$7,,500Annual cash inflow – 1,000 Cash outflow for taxes$6,500 x 1.74PV factor for 2 years at 10%$11,31081 . REQUIRED: The interest rate implicit in a lease.

DISCUSSION: (D) Present value tables may be used to determine the interest rate if the present value is already known. The present value is divided by the periodic amount to find the interest factor. The interest factor may then be found in the present-value-of-an-ordinary-annuity table in the row corresponding to the number of periods over which the annuity is payable.In this question, the series of payments is an annuity due, and an adjustment must be made before using the ordinary annuity table. The first payment is due immediately, so its present value (face amount) of $4,000 is deducted from the given present value of the annuity. The row for n – 1 periods (4) should then be used.Present value$15,192Minus first payment (4,000)$11,192Divided by annuity amount 4,000Present value factor2,798The present value factor of 2.798 is found under 16% in the row for four periods.Answers (A), (B), and (C) are incorrect because the present value factor used falls under 16%.

82 . Answer (B) is correct. At a 10% hurdle rate, the present value of the future inflows is: 20,000 x 2.48685 = $49,737Thus, the net present value is $4,737 (49,737 - 45,000). The profitability index calculation is:

49,737= 1.105345,000Answer (A) is incorrect because it uses the wrong present value factors. Answer (C) is incorrect because it uses the wrong present value factors. Answer (D) is incorrect because it uses the wrong present value factors.

83 . $100,000 x 4.868 (PVAF, n = 7, 10%) = $486,800

84 . $40,000 x 6.710 (PVAF, n = 10, 8%) = $268,400

85 . $120,000 X 4.968 (PVAF, n = 8, 12%) = $596,160

86 . 4.355 x $2,000 = $8,710

CMA EXAMINATION QUESTIONS Page 104 of 155

Page 105: P02 - Capital Budgeting

MANAGEMENT ADVISORY SERVICES CAPITAL BUDGETING

88 . (b) The maximum amount that Kern Co. should invest now to obtain a 15% internal rate of return is the present value of the project’s total net cash flows as computer below.YearNet cash flowsxPV of an ordinary annuity=PV of net cash flows1$50,000x0.870=$ 43,5002$80,000x0.756=$ 60,480Total present value$103,980

89 . ($100,000 x .833) + ($200,000 x .694) = $222,100

90 .Cash Flow Before TaxP17,863Depreciation (P43,825 4)10,706Net Income Before Tax7,157Income Tax (40%)2,863Net Income4,294Depreciation10,706Cash Flow After Tax (P42,825 2.85498P15,000

91

?. Before-tax cash flows Answer: b Diff: M

Tabular solution:X = after-tax cash flow.Y = before-tax cash flow.X = Y(1 - T).

$10,000 = X(PVIFA15%,10) $10,000 = X(5.0188) X = $1,992.51.$1,992.51 = Y(1 - 0.40) Y = $3,320.85 $3,321.

Financial calculator solution:Inputs: N = 10; I = 15; PV = -10,000; FV = 0. Output: PMT = $1,992.52.Before-tax CF = $1,992.52/0.6 = $3,320.87 $3,321.

92 . REQUIRED: To determine the annual savings needed for an investment to realize a 12% yield.DISCUSSION: Answer (C) is correct. The internal rate of return method of capital budgeting determines the rate of return at which the present value of the cash flows will exactly equal the investment outlay. In this problem, the desired IRR is given and the cash flows must be determined. The necessary annual savings can be computed as follows:TVMF x Cash flows =PV (Investment today)3.60x =$50,000 = (.57 x $10,000)3.60x =$44,300x =$12,306If the annual savings equals $12,306, the present value of the cash inflows will exactly equal the cash outflows.

CMA EXAMINATION QUESTIONS Page 105 of 155

Page 106: P02 - Capital Budgeting

MANAGEMENT ADVISORY SERVICES CAPITAL BUDGETING

87 . Answer (B) is correct. If the 8% return exactly equals the present value of the future flows (i.e., the NPV is zero), then simply determine the present value of the future inflows. Thus, ($2,500)(PVIFA at 8% for 10 periods) + ($5,000)(PVIF at 8% for 10 periods = ($2,500)(6.710) + ($5,000)(.463) = $19,090.

Answer (A) is incorrect because $16,775 failed to include the present value of the $5,000 terminal benefit. Answer (C) is incorrect because they do not use present value analysis. Answer (D) is incorrect because they do not use present value analysis.

Note: Tabular solution differs from calculator solution due to interest factor rounding.93 . Answer (B) is correct. The correct answer is 16.33%. Investment cost ÷ $10,000 = 4.0 years Investment cost = $40,000 The present value of a 7-year annuity is $40,000, which equates to a rate of

return of about 16.33%. Answer (A) is incorrect because the return is 16.33%. Answer (C) is incorrect because the return is 16.33%. Answer (D) is incorrect because the return is 16.33%.

94 . Answer (A) is correct. The factor to use is 2.5, which is found at a little under 10% on the 3-year line of an annuity table. Answer (B) is incorrect because the factor of 2.5 is found at around 10%. Answer (C) is incorrect because the factor of 2.5 is found at around 10%. Answer (D) is incorrect because the factor of 2.5 is found at around 10%.

95 . Answer (D) is correct. $50,000 = $7,791 (PV at i for 10 periods)$50,000 ÷ $7,791 = 6.418

Using a PV table, 6.418 is PV at 9% for 10 periods. Answer (A) is incorrect because the rate of return is 9%. Answer (B) is incorrect because the rate of return is 9%. Answer (C) is incorrect because the rate of return is 9%.

96 . Answer (D) is correct. The total cash flows are only $70,000 (5 x $14,000). Thus, whatever the discount rate, the NPV will be less than $20,000 ($70,000 - $50,000). The return in the first year is $14,000, or 28% of the initial investment. Since the same $14,000 flows in each year, the IRR is going to be greater than 10% (actually, it is almost 14%). Answer (A) is incorrect because it is impossible for the NPV to be greater than $20,000, regardless of the discount rate used. Answer (B) is incorrect because it is impossible for the NPV to be greater than $20,000, regardless of the discount rate used. Answer (C) is incorrect because the IRR is greater; almost 14%.

97 . $13,500/$3,148 = 4.219PVAF of 4.288, n = 7, corresponds to 14%

98 . $100,000/$27,739 = 3.605PVAF of 3.605, n = 5, corresponds to 12%

CMA EXAMINATION QUESTIONS Page 106 of 155

Page 107: P02 - Capital Budgeting

MANAGEMENT ADVISORY SERVICES CAPITAL BUDGETING

67 . ($10,000 x 0.847) + ($12,000 x 0.718) + ($15,000 x 0.609) + ($20,000 x 0.516) + ($10,000 x 0.437) 99

?. IRR Answer: c Diff: E

Tabular solution: Solve first for numerical FVIFA and obtain corresponding interest rate from tableFV = $3,310 = $100(FVIFAIRR,20)FVIFAIRR,20 = 33.1 33.066IRR 5%

Financial calculator solution:Inputs: CF0 = 0; CF1 = -100; Nj = 19; CF2 = 3,210. Output: IRR = 5.0%.Alternate method annuity calculationInputs: N = 20; PMT = -100; FV = 3,310. Output: I = 5.0%

100 . $38,000/$11,607 = 3.27, which is the pv factor for n = 5, i = 16%

101 . $339,013.20 = $60,000FF = 5.65022Chart criteria for 10 years is 5.65022 = 12%

102 . $52,650 = $25,000FF = 2.106Chart criteria for 3 years is 2.106 = 20%

103 . $921,250= $250,000 FF = 3.685Chart criteria for six years is 3.685 = 16%

CMA EXAMINATION QUESTIONS Page 107 of 155

Page 108: P02 - Capital Budgeting

MANAGEMENT ADVISORY SERVICES CAPITAL BUDGETING

104

?. IRR Answer: c Diff: MTabular solution:Project A: Calculate PVIF and look in table matching period = 1 with the calculated factor value

$10,000 = $11,800(PVIFIRR,1) 0.8475 = (PVIFIRR,1) IRRA = 18%.

Project C: Calculate the PVIFA and look in table matching period = 3 with the calculated factor value$12,000 = $5,696(PVIFAIRR,3)2.10674 = (PVIFAIRR,3) IRRC 20%.

Financial calculator solution:Project A: Inputs: N = 1; PV = -10,000; FV = 11,800.

Output: I = 18% = IRRA.

Project C: Inputs: N = 3; PV = -12,000; PMT = 5,696.Output: I = 19.99% 20% = IRRC.

105

?. IRR Answer: d Diff: M

Tabular solution:Solve for numerical PVIF and PVIFA then obtain corresponding interest rate from tableMachine A: $2,000 = $3,877(PVIFIRRA,4) 0.51586 = PVIFAIRRA,4

IRRA 18%.CMA EXAMINATION QUESTIONS Page 108 of 155

Page 109: P02 - Capital Budgeting

MANAGEMENT ADVISORY SERVICES CAPITAL BUDGETING

Machine B: $2,000 = $832(PVIFAIRRB,4) 2.40385 = PVIFAIRRB,4

IRRB 24%.

Financial calculator solution:Machine A: Inputs: CF0 = -2,000; CF1 = 0; Nj = 3; CF2 = 3877. Output: IRR = 17.996% 18%.

Machine B: Inputs: CF0 = -2,000; CF1 = 832; Nj = 4. Output: IRR = 24.01% 24%.

- $30,000 = $10,911

68 . Investment ($50,000)Present value of cash inflows:Year 1 ($20,000 x 0.943) 18,860

Year 2 ($24,000 x 0.890) 21,360Year 3 ($38,000 x 0.840) 31,920Year 4 ($28,000 x 0.792) 22,176Net present value $ 44,316

69.Year 0 = ($50,000 - $950,000) = $(900,000)Year 1 = $350,000 x 0.877 = 306,950Year 2 =

$350,000 x 0.769 = 269,150Year 3 = $350,000 x 0.675 = 236,250Year 4 = $350,000 x 0.592 = 207,200$ 119,550

70.Yr. 0 ($60,000 - $200,000 - $60,000) x 1.000 = $(200,000)Yr. 1 $50,000 x 0.909 = 45,450Yr. 2

$150,000 x 0.826 = 123,900Yr. 3 $150,000 x 0.751 = 112,650 $ 82,00071.

Yr. 0 ($60,000 - $200,000 - $60,000) x 1.000 =$(200,000)Yr. 1 $ 50,000 x 0.806 = 40,300Yr. 2 $150,000 x 0.650 = 97,500Yr. 3 $150,000 x 0.524 = 78,600$ 16,400

106

?. IRR Answer: c Diff: E

Tabular solution:$200,000 = $44,503(PVIFAIRR,10)PVIFAIRR,10 = 4.49408

IRR 18%.

Financial calculator solution:Inputs: CF0 = -200,000; CF1 = 44,503; Nj = 10. Output: IRR = 18%.

107 . 4.10 x $1,220 = $5,002 7% IRR

CMA EXAMINATION QUESTIONS Page 109 of 155

Page 110: P02 - Capital Budgeting

MANAGEMENT ADVISORY SERVICES CAPITAL BUDGETING

108. Multiple IRRs Answer: c Diff: T

Numerical solution:This problem can be solved numerically but requires an iterative process of trial and error using the possible solutions provided in the problem.

Investigate first claim: Try k = IRR = 13% and k = 12.5%NPVk = 13% = -10,000 + 100,000/1.13 - 100,000/(1.13)2 = 180.91.NPVk = 12.5% = -10,000 + 100,000/1.125 - 100,000/(1.125)2 = -123.46.The first claim appears to be correct. The IRR of the project appears to be between 12.5% and 13.0%.

Investigate second claim: Try k = 800% and k = 780%NPVk = 800% = -10,000 + 100,000/9 - 100,000/(1 + 8)2

= -10,000 + 11,111.11 - 1,234.57 = -123.46.

NPVk = 780% = -10,000 + 100,000/8.8 - 100,000/(1 + 7.8)2

= -10,000 + 11,363.64 - 1,291.32 = 72.32.

The second claim also appears to be correct. The IRR of the project flows also appears to be above 780% but below 800%.

Below is a table of various discount rates and the corresponding NPVs.

Discount rate (%) NPV 12.0 ($ 433.67) 12.5 (123.46) 12.7 (1.02) IRR1 12.7% 13.0 180.91 25.0 6,000.00 400.0 6,000.00 800.0 (123.46)

CMA EXAMINATION QUESTIONS Page 110 of 155

Page 111: P02 - Capital Budgeting

MANAGEMENT ADVISORY SERVICES CAPITAL BUDGETING

787.0 2.94 IRR2 787% 780.0 72.32

By randomly selecting various costs of capital and calculating the project’s NPV at these rates, we find that there are two IRRs, one at about 787 percent and the other at about 12.7 percent, since the NPVs are approximately equal to zero at these values of k. Thus, there are multiple IRRs.

109 . Answer (D) is correct. The MIRR is the interest rate at which the present value of the cash outflows discounted at the cost of capital equals the present value of the terminal value. The terminal value is the future value of the cash inflows assuming they are reinvested at the cost of capital. The present value of the outflows is $10,000 because no future outflows occur. The terminal value is $14,396 [(1.166 x $6,000) + (1.08 x $5,000) + (1.00 x $2,000)]. Accordingly, the present value of $1 interest factor for the rate at which the present value of the outflows equals the present value of the terminal value is .695 ($10,000 ÷ $14,396). This factor is closest to that for 12%. Answer (A) is incorrect because the MIRR must be greater than 12%. The present value of $1 interest factor for this project that equates the present values of the costs and the terminal value is lower than the factor for 12%. The lower the present value factor, the higher the interest rate. Answer (B) is incorrect because the MIRR must be greater than 12%. The present value of $1 interest factor for this project that equates the present values of the costs and the terminal value is lower than the factor for 12%. The lower the present value factor, the higher the interest rate. Answer (C) is incorrect because the MIRR must be greater than 12%. The present value of $1 interest factor for this project that equates the present values of the costs and the terminal value is lower than the factor for 12%. The lower the present value factor, the higher the interest rate.

NPV = -$6,500 + + + +

= $765.91.

?.

YearCash FlowDiscounted Cash FlowCumulative Cash Flow0-$100,000-$100,000.00-$100,000.00140,00036,363.64-

63,636.36290,00074,380.1710,743.81330,00022,539.4433,283.25460,00040,980.8174,264.0

6Discounted Payback = 1 + = 1.86 years.59

?. The sum of the PVs of the t = 1, t = 2, and t = 3 cash flows at t = 0 for Project A is $99,474.08. Thus, the discounted payback period of Project A exceeds 3 years and Project A is not acceptable. The PVs of the t = 1, t = 2, and t = 3 cash flows at t = 0 for Project B are $45,454.55, $16,528.93, and $22,539.44, respectively. These PVs sum to $84,522.92, which is greater than the cost of the project, indicating that the discounted payback period is less than 3 years. Thus, Project B will be undertaken.

60 . X = $300,000 x PV Ann 4 (12%) = $300,000 x 3.037X= $911,100

Answer (B) is incorrect because $68,000 deducts salvage value when calculating depreciation expense, which is not required by the tax law. Answer (C) is incorrect because $64,200 assumes depreciation is deducted for tax purposes over 10 years rather than 5 years. Answer (D) is incorrect because $79,000 is taxable income.

49 . Answer (C) is correct. The payback period is the number of years required to complete the return of the original investment. This measure is computed by dividing the net investment required by the average expected net cash inflow to be generated. The first step is to determine the annual cash flow. The $80,000 cost reduction will be offset by the tax expense on the savings. The full $80,000, however, will not be taxable because depreciation can be deducted before computing income taxes. Allocating the $250,000 cost evenly over 5 years produces an annual depreciation expense of $50,000. Thus, taxable income will be $30,000 ($80,000 - $50,000). At a 40% tax rate, the tax on $30,000 is $12,000. The net annual cash

CMA EXAMINATION QUESTIONS Page 111 of 155

Page 112: P02 - Capital Budgeting

MANAGEMENT ADVISORY SERVICES CAPITAL BUDGETING

113

?. Crossover rate and missing cash flow Answer: e Diff: M NStep 1: Determine the NPV of Project A at the crossover rate:

NPVA = -$4 + $2/1.09 + $3/(1.09)2 + $5/(1.09)3

= -$4 + $1.83486 + $2.52504 + $3.86092 = $4.22082 million.

Step 2: Determine the PV of cash inflows for Project B at the crossover rate:NPVB = CF0 + $1.7/1.09 + $3.2/(1.09)2 + $5.8/(1.09)3

= CF0 + $1.55963 + $2.69338 + $4.47866 = CF0 + $8.73167 million.

Step 3: Determine the cash outflow at t = 0 for Project B:At the crossover rate, NPVA = NPVB; NPVA - NPVB = 0.NPVA = $4.22082 million; NPVB = CF0 + $8.73167 million.

$4.22082 - CF0 - $8.73167 = 0 -CF0 = $8.73167 - $4.22082 CF0 = -$4.51085 million.

Financial calculator solution:Step 1: Determine the NPV of Project A at the crossover rate:

Using a financial calculator, enter the following inputs:CF0 = -4, CF1 = 2, CF2 = 3, CF3 = 5, I = 9, and then solve for NPVA = $4.22082 million.

Step 2: Determine the PV of cash inflows for Project B at the crossover rate:Using a financial calculator, enter the following inputs:CF0 = 0, CF1 = 1.7, CF2 = 3.2, CF3 = 5.8, I = 9, and then solve for NPV = $8.73167 million.

(Remember, this isn’t the NPVB because at the crossover rate NPVB = NPVA = $4.22082 million.)

Step 3: Determine the cash outflow at t = 0 for Project B:CMA EXAMINATION QUESTIONS Page 112 of 155

Page 113: P02 - Capital Budgeting

MANAGEMENT ADVISORY SERVICES CAPITAL BUDGETING

115

?. NPV profiles Answer: b Diff: TNet Present Value

[$]

11.5

50,560

29,950

Project B’s NPV profile

Crossover rate = 11.5%

Project A’s NPV profile

Cost of Capital (%)

IRRA = 18%

IRRB = 15%

Financial calculator solution:Solve for IRRA

Inputs: CF0 = -50,000; CF1 = 15,990; Nj = 5. Output: IRR = 18.0%Solve for IRRB

CMA EXAMINATION QUESTIONS Page 113 of 155

Page 114: P02 - Capital Budgeting

MANAGEMENT ADVISORY SERVICES CAPITAL BUDGETING

Inputs: CF0 = -50,000; CF1 = 0; Nj = 4; CF2 = 100,560. Output: IRR = 15.0%118 . Answer (B) is correct. The first thing to note is that risky cash outflows are discounted at a lower discount rate, so in this case we discount the riskier Project B's cash flows at 11% - 2% = 9%. Project

A's cash flows are discounted at 11%. We would find the PV of the costs as follows:Project A Project BCF0 = -10,000,000 CF0 = -5,000,000CF1-10 = -1,000,000 CF1-10 = -2,000,000I = 11.0%I = 9.0%Solve for NPV = -$15,889,232 Solve for NPV = -$17,835,315Project A has the lower PV of costs. Project B is evaluated with a lower cost of capital, 9%, reflecting greater risk of the cash outflow only project. This is somewhat tricky, because CMA candidates typically think about raising the discount rate because of risk, but that is when revenues or net cash inflows are subject to risk. The discount rate is lowered to reflect risk when only costs are subject to risk. Answer (A) is incorrect because -$5.9 million results from failing to consider the initial $10 million outlay. Answer (C) is incorrect because -$16.8 million is based on the wrong discount. Answer (D) is incorrect because -$17.8 million is the NPV for Project B, which is not as good as Project A.

119 . Answer (B) is correct. The size of Mulva's capital budget will be determined by the number of projects it can profitably undertake, i.e., those projects for which the IRR > applicable WACC. First, find the costs of each type of financing: cost of retained earnings = ks = ($2.50 ÷ $40) + 0.05 = 11.25% and cost of debt = kd = 10%. To calculate the cost of new equity, ke, we solve for ke =[$2.50 ÷ ($40 - $2)] + 0.05 = 0.1158 = 11.58%. Given the firm's target capital structure and its retained earnings balance of $900,000, the firm can raise $1,500,000 with debt and retained earnings before it must use outside equity. Therefore, the WACC for 0 - $1,500,000 of financing = 0.4(0.10)(1 - 0.4) + 0.6(0.1125) = 0.0915 or 9.15%. Above $1,500,000, the firm must issue some new equity, so the WACC = 0.4(0.10)(1 - 0.4) + 0.6(0.1158) = 0.0935 or 9.35%. Projects A, B, and C will definitely be undertaken because the IRR > WACC. Next, determine whether Project D will be profitable. Since in taking A, B, and C, we will need financing of $1,200,000, the $550,000 needed for Project D would involve financing $300,000 with debt and retained earnings and $250,000 with debt and new equity. Thus, the WACC for Project D is ($300,000 ÷ $550,000) x 0.0915 + ($250,000 ÷ $550,000) x 0.0935 = 9.24%, which is less than Project D's IRR. Thus, Projects A, B, C, and D should be accepted, and the firm's capital budget is $1,750,000. Answer (A) is incorrect because $1,200,000 does not include Project D. Answer (C) is incorrect because $2,400,000 includes Project E, which would not be a profitable investment since its IRR is less than the weighted average cost of capital. Answer (D) is incorrect because $800,000 excludes Projects C and D, which are both profitable.

120 . Answer (B) is correct. The IRR is the discount rate at which the net present value (discounted net cash inflows - investment) of a project is zero. Hence, an investment should be profitable if the IRR exceeds the company's cost of capital. Projects B, D, and E, with a combined cost of $1,050,000, have the highest IRRs. Each is in excess of the company's maximum 11% cost of capital (10% + .5% + .5%). Because their combined cost exceeds the level ($1,000,000) at which the cost of capital rises to 11%, Projects A (10.5%) and C (10.8%) must be rejected using the IRR criterion. Answer (A) is incorrect because the IRRs for B, D, and E exceed the cost of capital. Answer (C) is incorrect because project C should be rejected. Answer (D) is incorrect because projects A and C should be rejected.

121 . Answer (A) is correct. If the projects are mutually exclusive, only one can be accepted. The decision rule is to accept the project with the highest NPV (Project A). Answer (B) is incorrect because Project B has a negative NPV and would not be accepted. Answer (C) is incorrect because, by definition, only one project can be accepted. Answer (D) is incorrect because, by definition, only one project can be accepted.

122 . E, because it has the highest NPV.

123 . Y, because it has the highest NPV.CMA EXAMINATION QUESTIONS Page 114 of 155

Page 115: P02 - Capital Budgeting

MANAGEMENT ADVISORY SERVICES CAPITAL BUDGETING

124 .

At a cost of capital of 15%, both projects have negative NPVs and, thus, both would be rejected.

Tabular solution (in thousands):NPVX = -100 + 50(PVIF15%,1) + 40(PVIF15%,2) + 30(PVIF15%,3) + 10(PVIF15%,4) = -100 + 50(0.8696) + 40(0.7561) + 30(0.6575) + 10(0.5718) = -0.833 = -$833.

NPVZ = -100 + 10(PVIF15%,1) + 30(PVIF15%,2) + 40(PVIF15%,3) + 60(PVIF15%,4) = -100 + 10(0.8696) + 30(0.7561) + 40(0.6575) + 60(0.5718)

CMA EXAMINATION QUESTIONS Page 115 of 155

Page 116: P02 - Capital Budgeting

MANAGEMENT ADVISORY SERVICES CAPITAL BUDGETING

126 . Answer (C) is correct. Only two of the projects can be selected because three would require more than $450,000 of capital. Project S can immediately be dismissed because it has a negative net present value (NPV). Using the NPV and the profitability index methods, the best investments appear to be Q and R. The internal rate of return (IRR) method indicates that P is preferable to R. However, it assumes reinvestment of funds during years 4 and 5 at the IRR (18.7%). Given that reinvestment will be at a rate of at most 12%, the IRR decision criterion appears to be unsound in this situation. Answer (A) is incorrect because the amount of capital available limits the company to two projects. Answer (B) is incorrect because the amount of capital available limits the company to two projects. Answer (D) is incorrect because the profitability index and NPV are higher for R than P.

127 . Answer (B) is correct. Because unused funds cannot be invested at a rate greater than 12%, the company should select the investment with the highest net present value. Project Q is preferable to R because its return on the incremental $45,000 invested ($235,000 cost of Q - $190,000 cost of R) is greater than 12%. Answer (A) is incorrect because Project P has a life of only 3 years, and the high IRR would be earned only for that period and could not be reinvested at that rate in years 4 and 5. Also, P's NPV is lower than that of Q. Answer (C) is incorrect because, although P's IRR of 18.7% for 3 years exceeds Q's (17.6% for 5 years), the funds from P cannot be invested in years 4 and 5 at greater than 12%. Answer (D) is incorrect because the payback period is a poor means of ranking projects. It ignores both reinvestment rates and the time value of money.

128

?. Mutually exclusive projects Answer: b Diff: M

Tabular solution: Solve for numerical PVIFA and PVIF, then obtain corresponding interest rate from table.Project A: $100,000 = 39,500(PVIFAIRRA,3) 2.53165 = PVIFAIRRA,3

IRRA 9%

Project B: $100,000 = 133,000(PVIFIRRB,3) 0.75188 = PVIFIRRB,3

IRRB 10%The firm’s cost of capital is not given in the problem; use IRR decision rule. Since IRRB > IRRA; Project B is preferred.

CMA EXAMINATION QUESTIONS Page 116 of 155

Page 117: P02 - Capital Budgeting

MANAGEMENT ADVISORY SERVICES CAPITAL BUDGETING

129 . Step 1: Draw the time lines (in millions of dollars):

Step 2: Calculate the NPV for each system:NPVS = -$3 + $2.5/1.12 - $0.5/(1.12)2 + $2.5/(1.12)3 - $0.5/(1.12)4 + $2.5/(1.12)5 - $0.5/(1.12)6 + $2.5/(1.12)7 - $0.5/(1.12)8 + $2.5/(1.12)9 + $2.5/(1.12)10 NPVS = -$3 + $2.232 - $0.399 + $1.779 - $0.318 + $1.419 - $0.253 + $1.131 - $0.202 + $0.902 + $0.805NPVS = $4.096 $4.1 million.

NPVL = -$5 + $2/(1.12)1 + $2/(1.12)2 + $2/(1.12)3 + $2/ (1.12)4 - $2/(1.12)5 + $1.5/(1.12)6 + $1.5/(1.12)7 + $1.5/(1.12)8 + $1.5/(1.12)9 + $1.5/(1.12)10

NPVL = -$5 + $1.786 + $1.594 + 1.424 + $1.271 - $1.135 + $0.760 + $0.679 + $0.606 + $0.541 + $0.483NPVL = $3.009 $3.01 million.

130 . Answer (B) is correct. The profitability index equals the present value of the cash inflows divided by the investment outlay. The highest profitability index is Project B's at 2.400 ($600,000 ÷ $250,000). Answer (A) is incorrect because the profitability index for A is .89 ($980,000 ÷ $1,100,000). Answer (C) is incorrect because the profitability index for C is 1.307 ($1,830,000 ÷ $1,400,000). Answer (D) is incorrect because the profitability index for D is 1.215 ($790,000 ÷ $650,000).

131 . Answer (C) is correct. Project C has the highest net present value ($1,830,000 - $1,400,000 = $430,000), so it should be chosen. Answer (A) is incorrect because Project A has a negative net present value. Answer (B) is incorrect because Project B has a net present value of only $350,000. Answer (D) is incorrect because Project D has a net present value of only $140,000.

132 . Answer (C) is correct. A company using the net present value (NPV) method should undertake all projects with positive NPVs that are not mutually exclusive. Given that Projects 1, 2, and 4 have positive NPVs, those projects should be undertaken. Furthermore, a company using the internal rate of return (IRR) as a decision rule ordinarily chooses projects with a return greater than the cost of

CMA EXAMINATION QUESTIONS Page 117 of 155

Page 118: P02 - Capital Budgeting

MANAGEMENT ADVISORY SERVICES CAPITAL BUDGETING

capital. Given a 12% cost of capital, Projects 1, 2, and 4 should be chosen using an IRR 136 . Answer (D) is correct. Given that only $600,000 is available and that each project costs $200,000 or more, no more than two projects can be undertaken. Because Projects 3 and 4 have the greatest

NPVs, profitability indexes, and IRRs, they are the projects in which the company should invest. Answer (A) is incorrect because Project 1 has a negative NPV. Answer (B) is incorrect because this answer violates the $600,000 limitation. Answer (C) is incorrect because the combined NPV of Projects 2 and 3 is less than the combined NPV of Projects 3 and 4.

137 . Answer (D) is correct. Given that $300,000 is available and that each project costs $200,000 or more, only one project can be undertaken. Because Project 3 has a positive NPV and the highest profitability index, it is the best investment. The high profitability index means that the company will achieve the highest NPV per dollar of investment with Project 3. The profitability index facilitates comparison of different-sized investments. Answer (A) is incorrect because Project 1 has a negative NPV. Answer (B) is incorrect because choosing more than one project violates the $300,000 limitation. Answer (C) is incorrect because choosing more than one project violates the $300,000 limitation.

138 . Answer (B) is correct. The NPV is the excess of the present value of estimated cash inflows over the net cost of the investment. At a zero cost of capital, the NPV is simply the sum of a project’s undiscounted cash flows. Thus, the NPV of Project A is $50,000 ($40,000 + $40,000 - $30,000), and the PNV of Project B is $200,000 ($700,000 + $500,000 - $1,000,000).Answer (A) is incorrect because $30,000 and $1,000,000 are the initial investment outlays for A and B, respectively. Answer (C) is incorrect because $80,000 and $1,200,000 are the cash inflows for A and B, respectively. Answer (D) is incorrect because $110,000 for Project A and $2,200,000 for Project B are calculated by adding the initial outlays to the sum of the cash inflows.

139 . Answer (D) is correct. The internal rate of return ins the discount rate that sets a project’s NPV equal to zero. The internal rate of return may be determined as follows if IRR is the internal rate of return, t is the time period, and n is the number of time periods.

Substituting in the foregoing expression yields an annual internal rate of return of 100%.

140 . Answer (A) is correct. NPV assumes that cash inflows from the investment project can be reinvested at the cost of capital, whereas IRR assumes that cash flows from each project can be reinvested at the IRR for that particular project. This underlying assumption is considered to be a weakness of the IRR technique. The cost of capital is the appropriate reinvestment rate because it represents the opportunity cost for a project at a given level of risk. The problem with the IRR method is that it assumes a higher discount rate even though a project may not have a greater level of risk.

CMA EXAMINATION QUESTIONS Page 118 of 155

Page 119: P02 - Capital Budgeting

MANAGEMENT ADVISORY SERVICES CAPITAL BUDGETING

141 . Answer (B) is correct. For two mutually exclusive projects, the company should undertake the project with the highest positive net present value. Project B has a higher NPV than Project A. The NPVs of both projects are calculated using the firm's 8% cost of capital as follows:

= $41,331

= $76,816Answer (A) is incorrect because the NPV method ranks B over A. Answer (C) is incorrect because the projects are mutually exclusive. Answer (D) is incorrect because both projects have positive NPVs and one should be selected.

142 71. Answer (C) is correct. Independent projects are those whose cash flows are not affected by the acceptance or nonacceptance of other projects. The company must decide whether to accept or to reject each of the projects. Because both projects have a positive NPV calculated using the firm's 8% cost of capital, both should be accepted. Answer (A) is incorrect because each project has a positive NPV and both should be undertaken. Answer (B) is incorrect because each project has a positive NPV and both should be undertaken. Answer (D) is incorrect because each project has a positive NPV and both should be undertaken.

143 . Answer (C) is correct. The IRR is the discount rate at which the net present value is zero. The NPV is the present value of future cash flows minus the present value of the investment. Because of a possible difference in the scale of the projects and other factors, a higher NPV does not necessarily result in a higher IRR. Answer (A) is incorrect because the ranking of IRRs is indeterminate. Answer (B) is incorrect because the ranking of IRRs is indeterminate. Answer (D) is incorrect because the ranking for payback periods is indeterminate.

144 . Answer (B) is correct. Each of the three alternatives are weighted by their probabilities: $10,000 x .40 = $4,0004,000 x .40 =1,600 -2,000 x .20 = -400 $5,200Answer (A) is incorrect because $5,600 fails to deduct for the possibility of a negative return. Answer (C) is incorrect because $6,000 adds, rather than deducts, the negative return. Answer (D) is incorrect because the answer is $5,200.

145 . Answer (D) is correct. The machine costs $160,000 and will require $30,000 to install and test. In addition, the company will have to invest in $35,000 of working capital to support the production of the new machine. Thus, the total investment necessary is $225,000. Answer (A) is incorrect because $190,000 ignores the investment in working capital. Answer (B) is incorrect because $195,000 ignores the $30,000 of shipping, installation, and testing costs. Answer (C) is incorrect because $204,525 is the present value of $225,000 at 10% for 1 year.

CMA EXAMINATION QUESTIONS Page 119 of 155

Page 120: P02 - Capital Budgeting

MANAGEMENT ADVISORY SERVICES CAPITAL BUDGETING

148 . Answer (C) is correct. The $35,000 of working capital requires an immediate outlay for that amount, but it will be recovered in 5 years. Thus, the net discounted cash outflow is $13,265 [$35,000 initial investment - ($35,000 future inflow x .621 PV of $1 for 5 years at 10%)]. Answer (A) is incorrect because $(7,959) assumes the initial investment and its return are reduced by applying a 40% tax rate. Answer (B) is incorrect because $(10,080) assumes the initial investment was discounted for 1 year. Answer (D) is incorrect because $(35,000) fails to consider that the $35,000 will be recovered (essentially in the form of a salvage value) after the fifth year.

149 . $38,000/$11,607 = 3.27

150 . NPV = (3,605 x $11,607) - $38,000 = $3,843

151 . $400,000/$125,000 = 3.2 years

152.Investment($400,000)Present value of cash inflows($125,000 x 3.605) 450,625Net present value$ 50,625

153 . $30,000/$9,000 = 3.33

154 . NPV = (3.170 x $9,000) + (0.683 x $2,000) - $30,000 = $(104)

155 . Cash flows = ($20,000 x 0.6) + ($30,000/5 x 0.4) = $14,400$30,000/$14,400 = 2.08

156 . Cash flows = ($20,000 x 0.6) + ($30,000/5 x 0.4) = $14,400NPV = ($14,400 x 3.127) - $30,000 = $15,029

157 . 2 + $8,000/$16,000 = 2.50

158 . ($20,000 x 0.877) + ($22,000 x 0.769) + ($16,000 x 0.675) + ($30,000 x 0.592) + ($30,000 x 0.519) - $50,000 = $28,588

159 . Cash flow = ($15,000 x 0.6) + ($30,000/5 x 0.4) = $11,400$30,000/$11,400 = 2.63

160 . ($15,000 x 0.60) + ($30,000/5 x 0.40) = $11,400

CMA EXAMINATION QUESTIONS Page 120 of 155

Page 121: P02 - Capital Budgeting

MANAGEMENT ADVISORY SERVICES CAPITAL BUDGETING

163 . Answer (D) is correct. The payback method evaluates investments on the length of time until total dollars invested are recouped in the form of cash inflow or cash outflows avoided. It is calculated as Initial Investment Annual cash inflow of a project. The payback period of the equipment under consideration by Tam is: $100,000 $20,000 = 5 years.

164 . Answer (C) is correct. The internal rate of return (IRR) determines the rate of discount at which the present value of the future cash flows will exactly equal the investment outlay. It is computed by setting up the following equation.

Initial investment = TVMF x Cash flowsand solving for the time value of money factor (TVMF). The IRR can then be found by locating the TVMF for (n) periods in the present value of an ordinary annuity table and tracing to the top of that column to find the rate of return. The problem asks for the TVMF for the IRR of the equipment, which is calculated as follows:

$100,000 = TVMF x $20,0005.00 = TVMF

In estimating the IRR, the factors in the table of present values of an annuity should be taken from the columns closes to 5.00

165 . Answer (D) is correct. The accounting rate of return (ARR) computes an approximate rate of return which ignores the time value of money. It is calculated as Expected increase in annual net income Average investment in a project. Tam’s expected increase in annual income is as follows:Annual savings in after-tax cash costs$20,000Annual depreciation on equipment ($100,000 10 years)(10,000)Increase in annual net income$10,000A $100,000 investment is required to purchase the equipment. Thus, the ARR of the equipment under consideration by Tam is $10,000 $100,000 = 10%.

166 . Answer (A) is correct. The net present value is equal to the sum of discounted cash inflows minus any discounted cash outflows. Ignoring the effects of depreciation on cash outflows for tax payments, the net present value of this investment is $79,200 {[80,000 units x ($2.50 - $1.50)] x(.91 + .83 + .75) - $120,000}Answer (B) is incorrect because $120,000 is the purchase price of the machine, not the net present value of the investment. Answer (C) is incorrect because $129,000 is determined by using 100,000 machine hours instead of 80,000 units. Answer (D) is incorrect because $199,200 results when the $120,000 cash outflow is not subtracted.

167 . Answer (C) is correct. The accounting rate of return equals projected net profit divided by the average investment. The unit labor cost expected to be saved is $1.00 ($2.50 - $1.50), and 80,000 machine hours is the productive capacity. Given that one unit is produced per hour, $80,000 should be saved per year. Thus, the accounting rate of return is

Answer (A) is incorrect because 10% is the cost of capital, not the accounting rate of return on the average investment for the 3-year period. Answer (B) is incorrect because 33% results from dividing the annual labor cost savings minus depreciation by the cost of the machine instead of the average investment. Answer (D) is incorrect because 80% results from dividing 80,000 machine hours used by 100,000 machine hours available.

CMA EXAMINATION QUESTIONS Page 121 of 155

Page 122: P02 - Capital Budgeting

MANAGEMENT ADVISORY SERVICES CAPITAL BUDGETING

168 . Answer (A) is correct. The payback period (in years) is the annual net cash inflow divided into the initial cash investment. Thus, the payback period is 1.5 years ($120,000 ÷ $80,000). Answer (B) is incorrect because 1.20 years equals the cost of the machine divided by the annual production capacity ($120,000 ÷ 100,000). Answer (C) is incorrect because 2.00 years equals the cost of the machine divided by the average investment {$120,000 ÷ [($120,000 - 0) ÷ 2]}. Answer (D) is incorrect because 3.00 years equals the cost of the machine divided by the annual net cash inflow minus depreciation [$120,000 ÷ ($80,000 - $40,000)].

169 . Answer (D) is correct. The accounting rate of return (unadjusted rate of return or book value rate of return) equals accounting net income divided by the required average investment. The accounting rate of return ignores the time value of money. The average income over 5 years is $43,000 per year [($35,000 + $39,000 + $43,000 + $47,000 + $51,000) ÷ 5]. Hence, the accounting rate of return is 34.4% [$43,000 ÷ ($250,000 ÷ 2)]. Answer (A) is incorrect because 12.0% is the discount rate. Answer (B) is incorrect because 17.2% equals $43,000 divided by $250,000. Answer (C) is incorrect because 28.0% equals $35,000 divided by $125,000.

170 . Answer (A) is correct. The NPV is the sum of the present values of all cash inflows and outflows associated with the proposal. If the NPV is positive, the proposal should be accepted. The NPV is determined by discounting each expected cash flow using the appropriate 12% interest factor for the present value of $1. Thus, the NPV is $106,160 [(.89 x $120,000) + (.80 x $108,000) + (.71 x $96,000) + (.64 x $84,000) + (.57 x $72,000) - (1.00 x $250,000)]. Answer (B) is incorrect because $(97,970) is based on net income instead of cash flows. Answer (C) is incorrect because $356,160 excludes the purchase cost. Answer (D) is incorrect because $96,560 equals average after-tax cash inflow times the interest factor for the present value of a 5-year annuity, minus $250,000.

171 . Answer (B) is correct. The traditional payback period is the number of periods required for the undiscounted expected cash flows to equal the original investment. When periodic cash flows are not expected to be uniform, a cumulative calculation is necessary. The first year's cash inflow is $120,000. Adding another $108,000 in Year 2 brings the total payback after 2 years to $228,000. Accordingly, the traditional payback period is about 2.23 years {2 + [($250,000 - $228,000) ÷ $96,000 cash inflow in year 3]}. Answer (A) is incorrect because the 5-year total expected cash inflow is $480,000. Answer (C) is incorrect because the total expected cash flow for 1.65 years is $190,200. Answer (D) is incorrect because the total expected cash flow for 2.83 years is $307,680.

172 . Answer (C) is correct. The traditional payback period is the number of years required to complete the return of the original investment. It equals the net investment divided by the average expected periodic net cash flow. The periodic net cash inflow equals the $125,000 annual savings minus the additional taxes paid. This $125,000 increase in pretax income will be reduced by an increase in depreciation of $40,000 [($250,000 cost - $50,000 salvage) ÷ 5 years]. Hence, taxes will increase by $34,000 [$40% x ($125,000 - $40,000)], and the after-tax periodic net cash inflow will be $91,000 ($125,000 - $34,000). Accordingly, the payback period is 2.75 years ($250,000 ÷ $91,000). Answer (A) is incorrect because 2.00 years does not consider cash flows for taxes. Answer (B) is incorrect because 2.63 years omits residual value from the depreciation calculation. Answer (D) is incorrect because 2.94 years treats depreciation as a negative cash flow and ignores taxes.

173 . Answer (C) is correct. The accounting rate of return equals annual after-tax net profit divided by average investment. Annual after-tax net profit equals $51,000 [($125,000 cost savings - $40,000 depreciation as calculated in the preceding question) x (1.0 - 0.4)]. Consequently, the accounting rate of return equals 40.8% [$51,000 ÷ ($250,000 investment ÷ 2)].

CMA EXAMINATION QUESTIONS Page 122 of 155

Page 123: P02 - Capital Budgeting

MANAGEMENT ADVISORY SERVICES CAPITAL BUDGETING

174. Answer (A) is correct. The NPV of the stamping machine is the excess of the present values of the estimated cash inflows over the net cost of the investment. The annual after-tax net cash inflow from the cost savings is $91,000. In the fifth year, the cash inflows will also include the residual value. Year Cash Flow AmountsPV FactorPV of Cash Flow0 $(250,000)1.000$(250,000)1$91,000 .909 82,7192$91,000 .826 75,1663$91,000 .751 68,3414$91,000 .683 62,1535$91,000 + $50,000 .621 87,561Net present value $125,940Answer (B) is incorrect because $200,000 is the cost of the machine minus residual value. Answer (C) is incorrect because $250,000 is the cost of the machine. Answer (D) is incorrect because $375,940 is the NPV of the machine plus its purchase price.

175 . Answer (C) is correct. The payback period is the time required to recover the initial investment. The net cash inflows used to determine the payback period are not discounted. The initial cost was $105,000, and inflows during the first 2 years were $95,000 ($50,000 + $45,000). Thus, the first $10,000 ($105,000 - $95,000) of the third year's net cash inflows will complete the recovery of the initial investment. This amount is one-fourth of the third year's inflows. Hence, the payback period is 2.25 years. Answer (A) is incorrect because the payback period is 2.25 years. Answer (B) is incorrect because the payback period is 2.25 years. Answer (D) is incorrect because the payback period is 2.25 years.

176 . Answer (C) is correct. This problem requires the use of the net present value (NPV) method of investment analysis. The objective is to determine what average annual net cash inflow will equal the initial cost when discounted at a rate of 24%. Given that the investment has an expected life of 5 years, the appropriate time value of money factor is that for the present value of an ordinary annuity for 5 years at 24%. In this case, the annual net cash inflow is unknown, but the product of the factor (2.74) and the inflow is $105,000. Thus, dividing $105,000 by 2.74 results in an average annual net cash inflow of $38,321. In other words, if annual inflows are $38,321 per year, the present value is $105,000. This present value is equal to the initial cost, and the net present value is zero. At a net present value of zero, the investor is indifferent as to whether to undertake the investment. Answer (A) is incorrect because the average annual net cash inflow is $38,321. Answer (B) is incorrect because the average annual net cash inflow is $38,321. Answer (D) is incorrect because the average annual net cash inflow is $38,321.

177 . Answer (B) is correct. The accounting rate of return (or unadjusted rate of return) is computed by dividing the annual increase in accounting net income by the required investment. The average net income over the life of the investment is $19,000 [($15,000 + $17,000 + $19,000 + $21,000 + $23,000) ÷ 5 years]. Consequently, the accounting rate of return is 18.1% ($19,000 ÷ $105,000). Answer (A) is incorrect because the accounting rate of return is 18.1%. Answer (C) is incorrect because the accounting rate of return is 18.1%. Answer (D) is incorrect because the accounting rate of return is 18.1%.

178 . Answer (B) is correct. The net present value is computed by deducting the initial cost of the investment from the present value of the future net cash inflows. The present value of each of the future net cash inflows is determined by multiplying it by the appropriate factor for the present value of an amount as shown below. The net present value is $10,450 ( $115,450 - $105,000).

$50,000 x .81 =$40,50045,000 x .65 =29,25040,000 x .52 =20,80035,000 x. 42 =14,70030,000 x .34 = 10,200Totals$115,450Answer (A) is incorrect because the net present value is $10,450. Answer (C) is incorrect because the net present value is $10,450. Answer (D) is incorrect because the net present value is $10,450.

179 . Answer (B) is correct. Operating revenues $800,000Less depreciation$400,000Book income$400,000Cash flow $800,000Average book income $400,000Average book value of investment = ($4,000,000 + 0) ÷ 2 $2,000,000Book rate of return = $400,000 ÷ $2,000,000 = 20%Answer (A) is incorrect because 10% is based on initial cost rather than average book value. Answer (C) is incorrect because 28% is not

CMA EXAMINATION QUESTIONS Page 123 of 155

Page 124: P02 - Capital Budgeting

MANAGEMENT ADVISORY SERVICES CAPITAL BUDGETING

181 . Answer (A) is correct. NPV: present value of cash flows at 14% = $800,000(5.216) = $4,172,800. NPV = $4,172,800 - $4,000,000 = $172,800. Answer (B) is incorrect because at a 14% hurdle rate, the NPV is $172,800. Answer (C) is incorrect because at a 14% hurdle rate, the NPV is $172,800. Answer (D) is incorrect because at a 14% hurdle rate, the NPV is $172,800.

182 . [$8,000 – ($19,200/3)]/$19,200 = 8.33%

183 . NPV = ($8,000 x 2.487) - $19,200 = $696

184 . $19,200/$8,000 = 2.40, which is the pv factor for n = 3, i = 12%

185 . Net income = ($14,389 - $8,000) x 0.6 = $3,833.40$3,833.40/$40,000 = 9.58%

186 . Cash flow = ($14,389 x 0.6) + ($40,000/5 x 0.4) = $11,833.40$11,833.40 x 3.605 - $40,000 = $2,659

187 . Cash flow = ($14,389 x 0.6) + ($40,000/5 x 0.4) = $11,833.40$40,000/$11,833.40 = 3.380, which is the pv factor for n = 5 and i between 14 and 16 percent.

188 . ($14,389 x 0.60) + [($40,000/5) x 0.40] = $11,833

189 . NI = ($31,294 - $20,000) x 0.6 = $6,766.40$6,766.40/$80,000 = 8.47%

190 . Cash flow = ($31,294 x 0.6) + ($80,000/4 x 0.4) = $26,776.40$26,776.40 x 3.037 - $80,000 = $1,320

191 . Cash flow = ($31,294 x 0.6) + ($80,000/4 x 0.4) = $26,776.40$80,000/$26,776.40 = 2.988, which is the pv factor for n = 4 and I between 12 and 14 percent.

192 . ($31,294 X 0.60) + [($80,000/4) x 0.40] = $26,776CMA EXAMINATION QUESTIONS Page 124 of 155

Page 125: P02 - Capital Budgeting

MANAGEMENT ADVISORY SERVICES CAPITAL BUDGETING

193 . Answer (C) is correct. First, calculate the annual earnings and cash flows:Operating revenues$400,000Less depreciation 300,000Book income 100,000Cash flow 400,000The payback period in this case is equal to the investment divided by the annual operating cash flows that result from that investment. Thus, $1,500,000 ÷ $400,000 is 3.75 years. Answer (A) is incorrect because the initial investment is divided by the annual cash flows. Answer (B) is incorrect because 2.14 results from adding depreciation to revenues for use in the denominator. Answer (D) is incorrect because the denominator is annual cash flows, not depreciation.

194 . Answer (B) is correct. First, calculate the annual earnings and cash flows:Operating revenues$400,000Less depreciation 300,000Book income 100,000Cash flow 400,000The average book income is $100,000. The average book value of investment is ($1,500,000 + 0) ÷ 2, or $750,000. Thus, the book rate of return is equal to $100,000 ÷ $750,000 = 13.33%. Answer (A) is incorrect because 6.67% used the entire investment in the denominator instead of the average investment. Answer (C) is incorrect because 16.67% uses both an incorrect numerator and denominator. Answer (D) is incorrect because 26.67% uses revenue in the numerator instead of income.

195 . Answer (B) is correct. First, calculate the annual earnings and cash flows:Operating revenues $400,000Less depreciation300,000Book income100,000Cash flow 400,000The cash flows associated with the investment are then discounted accordingly: AmountDiscount FactorPresent ValueYear 0 initial investment -$1,500,0001.000-$1,500,000Years 1 through 5 cash flow$ 400,0003.605$1,442,000Net Present Value-$58,000Answer (A) is incorrect because there is a negative NPV. Answer (C) is incorrect because -$116,000 overstates the negative NPV. Answer (D) is incorrect because $1,442,000 is the present value of the future cash flows, not the NPV.

196 . Answer (A) is correct. First, calculate the annual earnings and cash flows:Operating revenues$400,000Less depreciation 300,000Book income 100,000Cash flow 400,000IRR is calculated by trial and error. Calculate the NPV at different discount rates.NPV at 10% = $400,000 (discount factor for 10%, 5 years) - $1,500,000 = $400,000(3.791) - $1,500,000 = $16,400NPV at 11% = $400,000 (3.696) - $1,500,000 = -$21,600.Thus, IRR lies between 10% and 11%. IRR = 10 + [16,400 ÷ (16,400 + 21,600)] = 10.43%. By interpolation, the actual IRR appears to be 10.43%. Answer (B) is incorrect because it uses something other than cash flows in the calculation. Answer (C) is incorrect because it uses something other than cash flows in the calculation. Answer (D) is incorrect because it uses something other than cash flows in the calculation.

197 . Answer (B) is correct. The accounting rate of return (the unadjusted rate of return or book value rate of return) equals the increase in accounting net income divided by either the initial or the average investment. It ignores the time value of money. The average income over 5 years is $86,000 [($70,000 + $78,000 + $86,000 + $94,000 + $102,000) ÷ 5]. Dividing the $86,000 average net income by the $250,000 average investment ($500,000 cost ÷ 2) produces an accounting rate of return of 34.4%. Answer (A) is incorrect because 84.9% equals the NPV divided by the average investment. Answer (C) is incorrect because 40.8% equals year 5 net income divided by the average investment. Answer (D) is incorrect because 12% equals the after-tax target rate of return.

CMA EXAMINATION QUESTIONS Page 125 of 155

Page 126: P02 - Capital Budgeting

MANAGEMENT ADVISORY SERVICES CAPITAL BUDGETING

199 . Answer (B) is correct. The payback period is the number of years required to complete the return of the original investment. The cash flows are not time adjusted. When the annual cash flows are not uniform, a cumulative computation is necessary. Thus, the total payback after 2 years is $456,000 ($240,000 + $216,000), and another $44,000 ($500,000 - $456,000) must be recovered in the third year. The third year fraction is found by assuming that cash flows occur evenly throughout the period. Dividing $44,000 by the $192,000 of third year inflows yields a ratio of .23. Hence, the payback period is 2.23 years. Answer (A) is incorrect because the payback period would exceed 5 years if the calculation were based on net income instead of cash flows. Answer (C) is incorrect because 1.65 years is based on the sum of cash flows and net income. Answer (D) is incorrect because it is based on discounted cash flows--not actual cash flows.

200 . Answer (D) is correct. The profitability index is the present value of the estimated net cash inflows over the investment's life divided by the present value of the net initial investment. If the profitability index is greater than 1.0, the investment project should be accepted. The present value of the cash inflows is $712,320. Thus, the profitability index is 1.425 ($712,320 ÷ $500,000 net initial investment). Answer (A) is incorrect because .61 is the present value of the net income amounts divided by the net initial investment. Answer (B) is incorrect because .42 is the NPV divided by the net initial investment. Answer (C) is incorrect because .86 is the sum of the undiscounted net income amounts divided by the net initial investment.

201 . Answer (B) is correct. The NPV at an after-tax target rate of return of 12% is $212,320. Given that the NPV is positive, the investment project should be accepted assuming no capital rationing. Furthermore, the IRR (the discount rate that reduces the NPV to $0) must be greater than the 12% hurdle rate that produced a positive NPV. The higher the discount rate, the lower the NPV. Answer (A) is incorrect because the IRR would be 12% if the NPV were $0. Answer (C) is incorrect because the IRR would be under 12% if the NPV were negative. Answer (D) is incorrect because whether the IRR is equal to, less than, or greater than the after-tax target rate of return can be determined from the amount of the NPV.

202 . Answer (C) is correct. The payback period is the number of years required to complete the return of the original investment. The principal problems with the payback method are that it does not consider the time value of money and the inflows after the payback period. The inflow for the first year is $120,000, the second year is $60,000, and the third year is $40,000, a total of $220,000. Given an initial investment of $200,000, the payback period must be between 2 and 3 years. If the cash inflows occur evenly throughout the year, $20,000 ($200,000 - $120,000 - $60,000) of cash inflows are needed in year 3, which is 50% of that year's total. Thus, the answer is 2.5 years. Answer (A) is incorrect because 1.67 years assumes that the inflows of the first year continue at the same rate in the second year. Answer (B) is incorrect because $296,400 will be recovered after 4.91 years. Answer (D) is incorrect because less than $180,000 will be paid back after 1.96 years.

203 . Answer (A) is correct. The NPV is defined as the excess of the present value of the net cash inflows over the net cost of the investment. Discounting the future cash inflows by the present value factors results in an $18,800 NPV ( $218,800 – $200,000).Answer (B) is incorrect because $218,800 is the present value of the future net cash inflows. Answer (C) is incorrect because $100,000 is the excess of the undiscounted cash flows over the investment. Answer (D) is incorrect because $91,743 is based on dividing the $300,000 total inflows by the total of all present value factors, which produces a nonsense answer.

204 . Answer (B) is correct. The net present value is the excess of the present values of the estimated net cash inflows over the net cost of the investment ($160,000). The future cash inflows consist of $85,000 per year savings minus income taxes. Given salvage value of $10,000 the depreciation base is $150,000 ($160,000 cost - $10,000). The first and third years’ deprecation deduction is $45,000 (30% x $150,000) leaving $40,000 of taxable income. Thus, first-year and third-year tax expense is $16,000 (40% x $40,000). Accordingly, the net cash inflow in both first and third years is $69,000

CMA EXAMINATION QUESTIONS Page 126 of 155

Page 127: P02 - Capital Budgeting

MANAGEMENT ADVISORY SERVICES CAPITAL BUDGETING

205 . Answer (C) is correct. The payback period is the number of years required for the cumulative undiscounted net cash inflows to equal the original investment. The future net cash inflows consist of $69,000 in years one and three, $75,000 in year two, and $10,000 upon resale. After 2 years, the cumulative undiscounted net cash inflow equals $144,000. Thus, $16,000 ($160,000 - $144,000) is to be recovered in the third year, and payback should be complete in approximately 2.23 years [2 years + ($16,000 ÷ $69,000 net cash inflow in third year)]. Answer (A) is incorrect because 1.88 assumes an $85,000 annual net cash inflow. Answer (B) is incorrect because 3.00 is the estimated useful life of the machine. Answer (D) is incorrect because 1.62 results from adding income tax expense to the cost savings each year.

206 . Answer (C) is correct. The NPV is calculated by discounting the after-tax cash flows by the cost of capital. The cash flows of this company constitute an annuity of $100,000 per year plus additional flows of $60,000 in year 1 and $40,000 in year 2. Thus, the present value of these flows is $418,600 [($100,000 x 3.36 PV of an annuity of $1 for 5 periods at 15%) + ($60,000 x .87 PV of $1 for 1 period at 15%) + ($40,000 x .76 PV of $1 for 2 periods at 15%)]. The NPV is therefore $18,600 ($418,600 - $400,000). Answer (A) is incorrect because a negative NPV of $64,000 assumes net cash flows of $100,000 per year. Answer (B) is incorrect because a negative NPV of $14,000 results from discounting the net earnings. Answer (D) is incorrect because $200,000 is the undiscounted excess of the cash flows over the investment.

207 . Answer (B) is correct. The payback method calculates the number of years required to complete the return of the original investment. The initial cost is $400,000, so Willis will recoup its investment in 3 years ($160,000 in year 1 + $140,000 in year 2 + $100,000 in year 3). Answer (A) is incorrect because only $230,000 would have been recovered after 1.5 years. Answer (C) is incorrect because $400,000 will have been recovered by the end of 3 years. Answer (D) is incorrect because 4.00 years is based on net earnings rather than cash flows.

208 . Cash flows = ($25,000 x 0.6) + ($50,000/4 x 0.4) = $20,000$50,000/$20,000 = $2.50

209 . Cash flow = ($25,000 x 0.6) + ($50,000/4 x 0.4) = $20,000$20,000 x 3.037 - $50,000 = $10,740

210 . Cash flow = ($25,000 x 0.60) + [($50,000/4) x 0.40] = $20,000

211 . Answer (C) is correct. The NPV is calculated by discounting the after-tax cash flows from an investment by the cost of capital: Year 1 $320,000 x .87 = $278,400 Year 2280,000 x .76 =212,800 Year 3200,000 x .66 =132,000 Year 4200,000 x .57 =114,000 Year 5200,000 x .50 = 100,000Present value of inflows$837,200Subtracting the $800,000 initial cost from the $837,200 present value of the future inflows produces an NPV of $37,200. Answer (A) is incorrect because $(128,000) is based on a discounting of the net earnings rather than on cash flows. Answer (B) is incorrect because $200,000 equals total undiscounted net earnings minus the initial investment. Answer (D) is incorrect because $400,000 equals the undiscounted after-tax cash flows minus the initial cost.

CMA EXAMINATION QUESTIONS Page 127 of 155

Page 128: P02 - Capital Budgeting

MANAGEMENT ADVISORY SERVICES CAPITAL BUDGETING

213 . Answer (B) is correct. The payback method is simply the time required to recover the investment. It does not consider the time value of money or returns after the payback period. When cash flows are not uniform, a cumulative calculation is necessary. Thus, 3.0 years is the payback period ($320,000 in the first year, $280,000 in the second year, and $200,000 in the third year). Answer (A) is incorrect because only a little over half of the initial $800,000 investment would have been recovered by the end of 1.5 years. Answer (C) is incorrect because $800,000 will have been recovered by the end of 3 years. Answer (D) is incorrect because 4.0 years is based on net earnings.

214 . Answer (B) is correct. The NPV method discounts the expected cash flows from a project using the required rate of return. A project is acceptable if its NPV is positive. The future cash inflows consist of $170,000 of saved expenses per year minus income taxes after deducting depreciation. In the first year, the after-tax cash inflow is $170,000 minus taxes of $32,000 {[$170,000 - (30% x $300,000) depreciation] x 40%}, or $138,000. In the second year, the after-tax cash inflow is $170,000 minus taxes of $20,000 {[$170,000 - (40% x $300,000) depreciation] x 40%}, or $150,000. In the third year, the after-tax cash inflow (excluding salvage value) is again $138,000. Also in the third year, the after-tax cash inflow from the salvage value is $12,000 [$20,000 x (1.0 - .40)]. Accordingly, the total for the third year is $150,000 ($138,000 + $12,000). The sum of these cash flows discounted using the factors for the present value of $1 at a rate of 16% is $326,556. $138,000 x .862 = $118,956 $150,000 x .743 =111,450 $150,000 x .641 = 96,150 Discounted cash inflows $326,556Thus, the NPV is $26,556 ($326,556 - $300,000 initial outflow). Answer (A) is incorrect because $31,684 does not consider the income taxes on the salvage value. The asset was fully depreciated, so any sales receipts would be taxable income. Answer (C) is incorrect because $94,640 does not consider income taxes. Answer (D) is incorrect because $18,864 does not consider the salvage value.

215 . Answer (A) is correct. The profitability index is the present value of the future net cash inflows divided by the present value of the net initial investment. The present value of the future net cash inflows is $326,556. Hence, the profitability index is 1.089 ($326,556 ÷ $300,000). Answer (B) is incorrect because 1.106 does not consider the income taxes on the salvage value. The asset was fully depreciated, so any sales receipts would be taxable income. Answer (C) is incorrect because 1.315 ignores all cash flows for income taxes. Answer (D) is incorrect because 1.063 does not consider the salvage value.

216 . Answer (C) is correct. The payback period is the time required to recover the original investment. The annual net after-tax cash inflows for years 1 through 3 are $138,000, $150,000, and $150,000, respectively. After 2 years, $288,000 ($138,000 + $150,000) will have been recovered. Consequently, the first $12,000 received in year 3 will recoup the initial investment. Because $12,000 represents .08 of the third year's net after-tax cash inflows ($12,000 ÷ $150,000), the payback period is 2.08 years. Answer (A) is incorrect because 2.84 years does not consider depreciation. Answer (B) is incorrect because 1.76 years does not consider taxes. Answer (D) is incorrect because 3.00 years is the time required to depreciate the asset fully, not the payback period.

217 . Answer (D) is correct. The payback period is the time required to recover the original investment. The annual net after-tax cash inflows for years 1 through 3 are $138,000, $150,000, and $138,000 (excluding salvage value), respectively. After 2 years, $288,000 ($138,000 + $150,000) will have been recovered. Consequently, the first $12,000 received in year 3 will recoup the initial investment. Because $12,000 represents .09 (rounded) of the third year's net after-tax cash inflows ($12,000 ÷ $138,000), the payback period is 2.09 years. Answer (A) is incorrect because 2.84 years includes salvage value and does not consider depreciation. Answer (B) is incorrect because 1.76 years includes salvage value and does not consider taxes. Answer (C) is incorrect because 2.08 years includes salvage value.

218 . Payback period = $4,500,000/$750,000 = 6.0 yearsCMA EXAMINATION QUESTIONS Page 128 of 155

Page 129: P02 - Capital Budgeting

MANAGEMENT ADVISORY SERVICES CAPITAL BUDGETING

221 . REQUIRED: The cost of Project A given the IRR.DISCUSSION: (B) the IRR is the discount rate that sets the net present value of the project equal to zero, to the present value of the costs equals the present value of the costs equals the present value of the cash inflows. At an IRR of 15%, the present value of the inflows (and therefore the present value of the costs) for Project A equals the constant annual inflow times the present value of an ordinary annuity of $1 for three periods at 15%. Thus, the cost of Project A is $22,832 ($10,000 x 2.2832).Answer (A) is incorrect because $8,696 uses the present value interest factor for one period at 15%. Answer (C) is incorrect because $24,869 is obtained using a 10% discount rate. Answer (D) is incorrect because $27,232 is based on a 5% discount rate.

222 . REQUIRED: The NPV of Project B given the discount rate.DISCUSSION: (A) The NPV is the present value of the cash inflows minus the cost of the project. The inflows for this project are not constant, so the individual cash flows may be discounted using the interest factors for the present value of $1 due at the end of a period discounted at 10%. The sum of these present values is $24,079 [($5,000 x .9091) + ($10,000 x .8264) + ($15,000 x .7513)]. Hence, the NPV is $4,079 ($24,079 – $20,000 cost).Answer (B) is incorrect because $6,789 uses a 5% discount rate. Answer (C) is incorrect because $9,869 is the NPV of Project A at a 10% discount rate. Answer (D) is incorrect because $39,204 uses present value interest factors for annuities.

223 . REQUIRED: The payback period for Project C.DISCUSSION (C) The payback period is the time necessary to recoup the investment. After 2 years, the cumulative cash inflows for Project C are exactly equal to the initial investment outlay of $25,000.Answer (A) is incorrect because the payback period would be zero only if a project had no cost or provided immediate cash inflows at least equal to the investment outlay. Answer (B) is incorrect because after 1 year, the cumulative cash inflows for Project C are only $15,000. the initial investment outlay is $25,000. Answer (D) is incorrect because Project C pays back its initial investment outlay in only 2 years.

224 . REQUIRED: The year 2 cash inflow for Project D.DISCUSSION: (C) the discounted payback period is the time required for discounted cash flows to recover the cost of the investment. Thus, the cost of Project D must equal the present value of the end-of-year-1 cash inflow discounted at 5% plus the present value of the end-of-year-2 cash inflow. The year 2 cash inflow (x) is calculated as follows:

$30,000 = ($20,000 x .9524) + (X x .9070)X = (430,000 - $19,048) .9070X = $12,075

Answer (A) is incorrect because $5,890 uses present value interest factors for annuities. Answer (B) is incorrect because $10,000 is based on the regular payback period. Answer (D) is incorrect because $14,301 is obtained using a 10% discount rate.

225 . Answer (C) is correct. The NPV of Project A equals the present value of the cash inflows minus the investment's cost. The present value of the cash inflows equals the present value factor for an ordinary annuity (five periods at the 10% cost of capital) times the periodic amount. Given that the NPV is $12,405, the present value of the cash inflows must be $112,405 ($100,000 cost + $12,405). Thus, the end-of-period amount must be $29, 652 ($112,405 ÷ 3.7908).

CMA EXAMINATION QUESTIONS Page 129 of 155

Page 130: P02 - Capital Budgeting

MANAGEMENT ADVISORY SERVICES CAPITAL BUDGETING

227 . Answer (B) is correct. The IRR (12%) is the discount rate that sets the NPV equal to zero. Thus, the NPV of Project C will equal zero when the investment cost equals the present value of the cash inflows. Given that the investment cost is $60,000, the sum of the present values of the cash inflows must also equal $60,000. The present value of the cash inflow at the end of year 1 is $17,858 ($20,000 x .8929 PVIF for one period at 12%). The present value of the cash inflow at the end of year 2 is $27,902 ($35,000 x .7972 PVIF for two periods at 12%). Accordingly, the present value of the cash inflow at the end of year 3 must be $14,240 ($60,000 - $17,858 - $27,902), and the undiscounted cash inflow must be $20,006 ($14,240 ÷ .7118 PVIF for three periods at 12%). Answer (A) is incorrect because $17,162 results if a 10% IRR is used. Answer (C) is incorrect because $24,126 results if the project cash inflows are treated as a level annuity and a 10% discount rate is used. Answer (D) is incorrect because $24,981 results if the project cash inflows are treated as a level annuity and a 12% discount rate is used.

228 . Answer (B) is correct. After 2 years, the cumulative cash inflows for Project B equal $80,000. Consequently, the investment outlay of $50,000 has been fully recovered. Answer (A) is incorrect because, after 1 year, Project B has had a $50,000 outflow but only $40,000 of inflows, so the investment outlay has not been fully recovered. Answer (C) is incorrect because the investment outlay for Project C will not be recovered until the third year. Answer (D) is incorrect because the investment outlay for Project C will not be recovered until the third year.

229 . Answer (C) is correct. The payback period is the time required to recover the initial cash outlay. The cost of A is $700,000, and the cumulative cash inflow at the end of the second year is $700,000 ($400,000 + $300,000). Answer (A) is incorrect because a payback period cannot be 0 years unless the project is cost free. Answer (B) is incorrect because only $400,000 has been recouped after one year. Answer (D) is incorrect because the initial cash outlay is expected to be recovered after two years.

230 . Answer (A) is correct. The net present value of Project A is calculated by subtracting the initial cash investment from the present value of the expected cash inflows. It equals $362,340 [($400,000 x .9091) + ($300,000 x .8264) + ($600,000 x .7513) - $700,000]. Answer (B) is incorrect because $1,062,340 results from not subtracting the initial investment. Answer (C) is incorrect because $1,676,670 is calculated using present value factors for annuities. Answer (D) is incorrect because $2,376,670 is calculated using present value factors for annuities without subtracting the initial investment of $700,000.

231 . Answer (C) is correct. The approximate internal rate of return is defined as the discount rate at which the net present value is zero. Using a discount rate of 24%, the net present value of Project B is -$4,675 [(1.9813 x $250,000) - $500,000]. Answer (A) is incorrect because, at a discount rate of 16%, the net present value of Project B is $61,475. Answer (B) is incorrect because, at a discount rate of 20%, the net present value of Project B is $26,625. Answer (D) is incorrect because, at a discount rate of 28%, the net present value of Project B is -$32,900.

232 . Answer (A) is correct. When investment projects are independent, the cash flows are unrelated. The company may choose any or all of them, subject to capital availability. Projects with positive net present values will be chosen. Project A has a net present value of $362,340 [($400,000 x .9091) + ($300,000 x .8264) + ($600,000 x .7513) - $700,000]. The net present value of Project B is $121,825 [(2.4873 x $250,000) - $500,000]. Answer (B) is incorrect because both projects have positive net present values and are therefore undertaken by the company. Answer (C) is incorrect because both projects have positive net present values and are therefore undertaken by the company. Answer (D) is incorrect because both projects have positive net present values and are therefore undertaken by the company.

CMA EXAMINATION QUESTIONS Page 130 of 155

Page 131: P02 - Capital Budgeting

MANAGEMENT ADVISORY SERVICES CAPITAL BUDGETING

Answer (A) is incorrect because $19,979 results if the investment cost is omitted from the calculation and the PVIF is used. Answer (B) is incorrect because $21,863 results if the investment cost is omitted from the calculation, a 12% discount rate is used, and the PVIF is used. Answer (D) is incorrect because $31,182 results if a 12% discount rate is used.

226 . Answer (C) is correct. The NPV of Project B equals the present value of the cash inflows minus the investment cost. The present value of the cash inflows equals the PVIFA for two periods at the 12% discount rate times the periodic amount. The present value of the cash inflows must be $67,604 ($40,000 x 1.6901). Thus, the NPV must be $17,604 ($67,604 - $50,000 investment cost).

234 . Answer (D) is correct. The internal rate of return is the discount rate at which the NPV is zero. Consequently, the cash outflow equals the present value of the inflow at the internal rate of return. The present value of $1 factor for project B's internal rate of return is therefore .4020 ($4,000,000 cash outflow ÷ $9,950,000 cash inflow). This factor is closest to the present value of $1 for 5 periods at 20%. Answer (A) is incorrect because 15% results in a positive NPV for project B. Answer (B) is incorrect because 16% is the approximate internal rate of return for project A. Answer (C) is incorrect because 18% is the company's cost of capital.

235 . REQUIRED: The net present value of each project.DISCUSSION: (B) The NPV is the excess of the present value of estimated cash inflows over the net cost of the investment. At a zero cost of capital the NPV is simply the sum of a project’s undiscounted cash flows. Thus, the NPV of Project A is $50,000 ($40,000 + $40,000 – $30,000), and the NPV of Project B is $200,000 ($700,000 + $500,000 – $1,000,000).Answer (A) is incorrect because $30,000 and $1,000,000 are the initial investment outlays for A and B, respectively. Answer (C) is incorrect because $80,000 and $1,200,000 are the cash inflows for A and B, respectively. Answer (D) is incorrect because $110,000 for Project A and $2,200,000 for Project B are calculated by adding the initial outlays to the sum of the cash inflows.

236 . REQUIRED: The internal rate of return for Project A.DISCUSSION: (D) The internal rate of return is the discount rate that sets a project’s NPV equal to zero. The internal rate of return may be determined as follows if IRR is the internal rate of return, t is the rime period, and n is the number of time periods:

Substituting in the foregoing expression yields an internal rate of return of 100%.

IRR = 100%Answer (A) is incorrect because applying a discount rate of 10% results in a NPV of $69,421. Answer (B) is incorrect because applying a discount rate of 15% results in a NPV of $65,028. Answer (C) is incorrect because applying a discount rate of 25% results in a NPV of $57,600.

CMA EXAMINATION QUESTIONS Page 131 of 155

Page 132: P02 - Capital Budgeting

MANAGEMENT ADVISORY SERVICES CAPITAL BUDGETING

Answer (A) is incorrect because ($18,112) results if the PVIF for 12% and two periods is used. Answer (B) is incorrect because ($16,944) results if the PVIF and a 10% discount rate are

used. Answer (D) is incorrect because $19,420 results if a 10% discount rate is used.

237 . REQUIRED: The difference between the net present value (NPV) and internal rate of return (IRR)DISCUSSION: (A) NPV assumes that cash inflows from the investment project can be reinvested at the cost of capital, whereas IRR assumes that cash flows from each project can be reinvested at the IRR for that particular project. This underlying assumption is considered to be a weakness of the IRR technique. The cost of capital is the appropriate reinvestment rate because it represents the opportunity cost for a project at a given level of risk. The problem with the IRR method is that it assumes a higher discount rate even though a project may not have a greater level of risk.Answer (B) is incorrect because NPV and IRR make consistent accept/reject decisions for independent projects. When NPV is positive, IRR exceeds the cost of capital and the project is acceptable. Answer (C) is incorrect because the NPV method can be used to rank mutually exclusive projects, whereas IRR cannot. The reinvestment rate assumption causes IRR to make faulty project rankings under some circumstances. Answer (D) is incorrect because IRR is expressed as a percentage and NPV in dollar terms.

238 . Calculate the after-tax component cost of debt as 10%(1 - 0.3) = 7%. If the company has earnings of $100,000 and pays out 50% or $50,000 in dividends, then it will retain earnings of $50,000. The retained earnings breakpoint is $50,000/0.4 = $125,000. Since it will require financing in excess of $125,000 to undertake any of the alternatives, we can conclude the firm must issue new equity. Therefore, the pertinent component cost of equity is the cost of new equity. Calculate the expected dividend per share (note this is D1) as $50,000/10,000 = $5. Thus, the cost of new equity is $5/[($35(1 - 0.12)] + 6% = 22.23%. Jackson’s WACC is 7%(0.6) + 22.23%(0.4) = 13.09%. Only the return on Project A exceeds the WACC, so only Project A will be undertaken.

239 . The size of Gibson’s capital budget will be determined by the number of projects it can profitably undertake, that is, those projects for which IRR > applicable WACC. First, find the costs of each type of financing: cost of retained earnings = ks = $4/$42 + 0.06 = 15.52% and cost of debt = kd = 11%. To calculate the cost of new equity, ke we solve for ke = $4/($42-$2) + 0.06 = 0.16 = 16%. Given the firm’s target capital structure and its retained earnings balance of $600,000, the firm can raise $1,000,000 with debt and retained earnings before it must use outside equity. Therefore, the WACC for 0 - $1,000,000 of financing = 0.4(0.11)(1 - 0.4) + 0.6(0.1552) = 11.95%. Above $1,000,000, the firm must issue some new equity, so the WACC = 0.4(0.11)(1 - 0.4) + 0.6(0.16) = 12.24%. Obviously, Projects A and B will be undertaken. You must then determine whether Project C will be profitable. Since in taking A and B we will need financing of $800,000, the $400,000 needed for Project C would involve financing $200,000 with debt and retained earnings and $200,000 with debt and new equity. Thus, the WACC for Project C is ($200,000/$400,000) 0.1195 + ($200,000/$400,000) 0.1224 = 12.095% which is greater than Project C’s IRR. Clearly, only Projects A and B should be accepted, and the firm’s capital budget is $800,000.

240 . Calculate the retained earnings break point (BPRE) as $300,000/0.6 = $500,000. Calculate ks as D1/P0 + g = $2(1.06)/$30 + 6% = 13.07%. Calculate ke as D1/(P0 - F) + g = $2(1.06)/($30 - $5) + 6% = 14.48%. Find WACC below BPRE as: WACC = 0.6(13.07%)+ 0.4(9%)(1 - 0.35) = 10.18%. Thus, up to $500,000 can be financed at 10.18%. Find WACC above BPRE as: WACC = 0.6(14.48%) + 0.4(9%)(1 - 0.35) = 11.03%. Thus, financing in excess of $500,000 costs 11.03%. Projects 2, 3, and 4 all have IRRs exceeding either WACC and should be accepted. These projects require $450,000 in financing. Project 1 is the next most profitable project. Given its cost of $100,000, half or $50,000 can be financed at 10.18% and the other half must be financed at 11.03%. The relevant cost of capital for Project 1 is then 0.5(10.18%) + 0.5(11.03%) = 10.61%. Since Project 1’s IRR is less than the cost of capital, it should not be accepted. The firm’s optimal capital budget is $450,000.

CMA EXAMINATION QUESTIONS Page 132 of 155

Page 133: P02 - Capital Budgeting

MANAGEMENT ADVISORY SERVICES CAPITAL BUDGETING

241 . Step 1: Calculate the retained earnings breakpoint:BPRE = Retained earnings/wc = ($500,000 0.6)/0.6 = $500,000.

Step 2: Calculate the WACCs: (There will be two: one with retained earnings and one with new equity.)WACC1 = [0.4 8% (1 - 0.4)] + [0.6 12%] = 9.12%.WACC2 = [0.4 8% (1 - 0.4)] + [0.6 13%] = 9.72%.

Step 3: Determine the optimal capital budget:Now, work through the projects, starting with the highest-return project first, to determine the firm’s optimal capital budget. Initially, the WACC is 9.12 percent for the first $500,000 of projects, providing they return more than 9.12 percent. On the basis of this, we will take Projects A, B, and C, for a total budget of $400,000. Project D will be funded half by WACC 1 and half by WACC2; however, since Project D returns 9.85 percent, we should still accept it because this is greater than WACC 2. Project E returns 9.25 percent, but it will be funded entirely out of WACC 2 funds at 9.72 percent, so we would not accept Project E. Therefore, Projects A, B, C, and D should be accepted and the total capital budget is $600,000.

242 . REQUIRED: The characteristics of projects on an IOS.DISCUSSION: (D) An IOS schedule is drawn for a set of independent projects. The decision to be made is whether to accept or reject each project without regard to other investment opportunities. Thus, the cash flows of one independent project are not influenced by those of another. Independence should distinguished from mutual exclusivity. Projects are mutually exclusive if acceptance of one requires rejection of the other.Answer (A) is incorrect because IOS schedules do not require that all projects have the same investment cost. The steps of the schedule can be of varying lengths. Answer (B) is incorrect because IOS schedules cannot be drawn for mutually exclusive projects. Answer (C) is incorrect because IOS schedules do not require that all projects have the same NPV. The NPV of each project depends on the investment cost and on the present value of the expected cash flows. Both costs and cash flows can vary for projects on an IOS.

243 . REQUIRED: The project(s) in which the company should invest and its optimal capital budget.DISCUSSION: (B) The intersection of the IOS and MCC schedules determines the cost of capital and the optimal capital budget. The company should begin with the project having the highest return and continue accepting projects as long as the IRR exceeds the MCC. The highest rank project is A, with a $50 million cost and a 14% IRR. The MCC is only 6% over this range of financing. The next highest ranked project is B, with a $75 million cost and a 12% IRR. When $125 million has been invested, the marginal cost of the next dollar of capital is 10%, so Project B is also acceptable, bringing the optimal capital budget to $125 million. Project C is not acceptable because it has an 8% return. The MCC is 10% for the first $50 million invested in this project and 12% for the remaining $75 million.Answer (A) is incorrect because both A and B should be undertaken. Answer (C) is incorrect because A is acceptable, but C is not. Answer (D) is incorrect because C is not acceptable. It offers an IRR less than the marginal cost of financing this project.

244 . REQUIRED: The marginal cost of capital and the appropriate discount rate.DISCUSSION: (C) The appropriate discount rate (the cost of capital used in capital budgeting) theoretically is determined at the intersection of the IOS and MCC schedules. This intersection is at an MCC of 10% and an optimal capital budget of $125 million. However, if the optimal capital budget is assumed to be $150 million, the company is still in the second interval of the MCC schedule. The

CMA EXAMINATION QUESTIONS Page 133 of 155

Page 134: P02 - Capital Budgeting

MANAGEMENT ADVISORY SERVICES CAPITAL BUDGETING

marginal cost of financing in this part of the schedule is 10%.Answer (A) is incorrect because 6% applies only to the first $75 million of new financing. Given that the optimal capital budget exceeds $75 million, 6% cannot be the discount rate.

Answer (B) is incorrect because 8% is the IRR of Project C. Answer (D) is incorrect because, at an investment level of $150 million, the MCC is 10%.

247 . REQUIRED: The IRR to the nearest percent for replacing the old machine.DISCUSSION: (D) The IRR is the discount rate at which the present value of the cash flows equals the original investment. Thus, the NPV of the project is zero at the IRR. The IRR is also the maximum borrowing cost the firm could afford to pay for a specific project. The IRR is similar to the yield rate/effective rate quoted in the business medial. The formula for the IRR involving an annuity equates the annual cash flow, times an unknown annuity factor with the initial net investment: $940,000 = $300,000 x Factor. The solution of the equation gives a factor of 3.1333, which is found in the 18% column on the five-period line.Answers (A), (B), and (C) are incorrect because dividing the net investment by the annual savings yields a factor of 3.1333, which corresponds to an 18% IRR.

248 . REQUIRED: The payback period for the new machine.DISCUSSION: (C) The payback method determines how long it takes for the investment dollars to be recovered by the annual net cash inflows. The time is computed by dividing the net investment by the average periodic net cash inflow. The initial net cash outlay divided by the annual cash savings equals 3.13 years ($940,000 ÷ $300,000).Answer (A) is incorrect because the initial cash outlay divided by the annual cash savings equals 3.13, not 1.14. Answer (B) is incorrect because the initial cash outlay divided by the annual cash savings equals 3.13, not 2.78. Answer (D) is incorrect because the initial cash outlay is $940,000 ($1,000,000 outlay - $60,000 inflow).

249 . REQUIRED: The present value of the depreciation tax shield for the second year of the machine’s life.DISCUSSION: (B) The applicable depreciation for 1997 is $380,000. At a tax rate of 40%, the savings is $152,000 ($380,000 x 40%). Its present value is $121,600 ($152,000 x 0.80 PC of $1 for two periods at 12%).Answer (A) is incorrect because the present value of the depreciation tax shield equals the present value of the product of 1990 depreciation multiplied by the tax rate of 40%. Answer (C) is incorrect because $109,440 is the present value of the tax saving discounted at 18%, not the required 12%. Answer (D) is incorrect because the present value of the depreciation tax shield is the PV of $1 for 2 periods at 12% times the savings of $152,000 ($380,000 x 40%).

250 . REQUIRED: The present value of the after-tax cash flow associated with the salvage of the old machine.DISCUSSION: (A) The old machine will be sold for $60,000, and the entire selling price represents a taxable gain because the book value is zero. At a 40% tax rate, the tax is $24,000. However, the tax will not be paid until the end of the year. Discounting the tax payments results in a present value of $21,360 ($24,000 x 0.89). This amount is subtracted from the $60,000 selling price (not discounted because received immediately) to yield an after-tax NPV of $38,640.Answer (B) is incorrect because the $24,000 tax payment to be paid at year-end should be discounted using the present value factor of 0.89. Answer (C) is incorrect because the $60,000 payment received for the machinery is received immediately upon sale, and therefore should not be discounted. Answer (D) is incorrect because only the discounted amount of income tax should be subtracted from the $60,000 received for the machinery.

CMA EXAMINATION QUESTIONS Page 134 of 155

Page 135: P02 - Capital Budgeting

MANAGEMENT ADVISORY SERVICES CAPITAL BUDGETING

251 . REQUIRED: The present value of the annual after-tax cash savings that arise from the new machine without consideration of the depreciation tax shield.DISCUSSION: (D) The annual savings of $300,000 must be reduced by the 40% tax, so the net effect of purchasing the new machine is an annual cash savings of $180,000 [(1 – 0.4) x $300,000]. The present value of the after-tax savings is therefore $649,800 ($180,000 x 3.61 PV of an ordinary annuity for five periods at 12%).Answer (A) is incorrect because $563,400 is the present value of the annual after-tax cash savings discounted at 18%, not the firm’s required 12%. Answer (B) is incorrect because the annual savings must be reduced by the 40% tax. This is achieved by multiplying annual savings by 1 – 0.4, not 1 – 0.6. Answer (C) is incorrect because the present value is the annual after-tax savings discounted at the PV of an ordinary annuity for five periods at 12%.

252 . REQUIRED: The present value of the additional after-tax cash flow resulting from a future sale of the new machine.DISCUSSION: (B) At the time of sale, the new machine will be fully depreciated, and any sale proceeds will be fully taxable as a gain. Hence, the $80,000 taxable gain will result in $32,000 of tax ($80,000 x 0.40), and the after-tax cash flow will be $48,000. Its present value will be $27,360 ($48,000 x 0.57 PV of $1 for five periods at 12%).Answer (A) is incorrect because the after-tax cash flow of $48,000, not the $32,0000 of tax, should be discounted at the PV factor of $1 for 5 periods at 12%. Answer (C) is incorrect because the $32,000 of tax should be deducted from the sale amount before discounting the after-tax cash flow. Answer (D) is incorrect because $48,000 is the after-tax cash inflow, which needs to be discounted to its PV.

253 . (3,750+3,750+11,250+11,250)/4)/(100,000/2)

254 . Accrual accounting income = $103,000 - (($250,000 - $10,000)/5)= $103,000 - $48,000= $ 55,000

AARR with initial investment = $55,000/($250,000 + $25,000)= $55,000/$275,000= 0.20

255 . a. Cash flow from sale: $116,000 ($120,000 - 40% tax on the $10,000 tax gain)b. Increase in annual cash outflows: $20,800 ($20,000 pretax cost increase + $800 increase in income taxes; the $20,000 increase in cash costs is more than offset by losing a $22,000 depreciation

deduction)

256 . Investment required ÷ Net annual cash inflow =Factor of the internal rate of return

$83,150 ÷ Net annual cash inflow = 3.326 $83,150 ÷ 3.326 = Net annual cash inflowCMA EXAMINATION QUESTIONS Page 135 of 155

Page 136: P02 - Capital Budgeting

MANAGEMENT ADVISORY SERVICES CAPITAL BUDGETING

= $25,000258 . a. Cash flow from sale: $90,000 ($70,000 + 40% tax savings on the $50,000 tax loss)

b. Increase in annual cash outflows: $27,600 ($30,000 pretax cost increase - $2,400 decrease in income taxes; the $30,000 increase in cash costs is partially offset by losing a $24,000 depreciation deduction)

259 . a. Net investment: $124,000 [$300,000 - $160,000 - 40% x ($200,000 - 160,000)]b. Increase in income taxes: $5,000 [40% x ($37,500 pretax flow - $75,000 depreciation + $50,000 lost depreciation)]c. Increase in cash flows: $32,500 ($37,500 - $5,000 increase in income taxes)

260 . a. Net investment: $74,000 [$150,000 - $60,000 - 40%($100,000 - 60,000)]b. Increase in income taxes: $16,000 [40% x ($50,000 pretax flow - $30,000 depreciation + $20,000 lost depreciation)]c. Increase in cash flows: $34,000 ($50,000 - $16,000 increase in income taxes)

261.Predicted

Cash FlowsYear(s)PV FactorPV ofCash FlowsInitial Investment$(160,000)01.000$(160,000)Salvage of old20,00001.00020,000Annual operations40,0001-32.32292,880Annual operations20,0004-6(3.889-2.322)31,340Save by not rebuilding40,00010.87735,080Salvage of new24,00060.45610,944 Net present value$ 30,244262.DescriptionYearsAmount16% FactorPresent ValueVan & equipment0($125,000)1.000($125,000)Working capital0($ 60,000)1.000($ 60,000)Building rent1-6($ 35,000)3.685($128,975)Net annual cash inflow1-

6$ 80,0003.685$294,800Salvage value, equipment6$ 5,0000.410$ 2,050Release of working capital6$ 60,0000.410$ 24,600Net present value$ 7,475263.

YearExplanationAmount12% FactorPresent Value0Investment to update assets$ (500,000)1.000$ (500,000)1-9Annual cash inflows 1,805,0005.3289,617,0409Selling price for the division .600,0000.361 216,600Net present value$9,333,640The sales price of $9,000,000 is less than the present value of the cash flows resulting from retaining the division. General thus should not accept the offer.

264 .DescriptionYearsAmount10% FactorPresent ValueEquipment0($300,000)1.000($300,000)Working capital0($ 50,000)1.000($ 50,000)Net annual cash inflow 1-7$ 70,0004.868$340,760Salvage value,

equipment 7$ 5,0000.513$ 2,565Release of working capital 7$ 50,0000.513$ 25,650 Net present value$ 18,975265.

DescriptionYearsAmount16% FactorPresent Value Equipment0($100,000)1.000($100,000)Working capital0($ 40,000)1.000($ 40,000)Building rent1-6($ 24,000)3.685($ 88,440)Net annual cash inflow1-6$ 60,0003.685$221,100Salvage value, equipment6$ 10,0000.410$ 4,100Release of working capital6$ 40,0000.410$ 16,400Net present value$ 13,160

CMA EXAMINATION QUESTIONS Page 136 of 155

Page 137: P02 - Capital Budgeting

MANAGEMENT ADVISORY SERVICES CAPITAL BUDGETING

266. PresentYears Amount 14% Factor Value

Working capital investment Now $(100,000) 1.000 $(100,000)Annual cash inflows ...... 1-5 17,000 3.433 58,361Working capital released 5 100,000 0.519 51,900Net present value ........ $ 10,261

Yes, the distributorship should be accepted since the project has a positive net present value.

267. PresentItem Years Amount 14% Factor ValueInvestment now ($16,000) 1.000 ($16,000)Annual cash inflows ............... 1-12 3,600 5.660 20,376Working capital required .............. now (4,500) 1.000 (4,500)Working capital released .............. 12 4,500 0.208 936Salvage value equipment ............. 12 2,000 0.208 416Net present value $ 1,228

268. PresentProject A: Amount 12% Factor ValueCash investment now.............. ($15,000) 1.000 ($15,000)Cash inflow at the end of 5 years $21,000 0.567 $11,907Cash inflow at the end of 8 years $21,000 0.404 $ 8,484Net present value................ $ 5,391

Project B:Cash investment now.............. ($11,000) 1.000 ($11,000)Annual cash outflow for 5 years.. ($ 3,000) 3.605 ($10,815)Additional cash inflow at the end of 5 years............... $21,000 0.567 $11,907Net present value................ ($ 9,908)

CMA EXAMINATION QUESTIONS Page 137 of 155

Page 138: P02 - Capital Budgeting

MANAGEMENT ADVISORY SERVICES CAPITAL BUDGETING

269 . a. Investment required ÷ Net annual cash inflow = Payback period $60,000 - $5,000) ÷ ($18,000 + $12,000) = 1.83 years (rounded)

b. Incremental net income ÷ Investment = Simple rate of return $18,000 ÷ $55,000 = 32.7% (rounded)

270 . a. Payback period: 3.0 years (30,000 + 60,000 + 90,000)

b. Book rate of return: 20%Average return: $54,000 ($270,000 total / 5 years)Depreciation: 36,000 ($180,000 / 5 years)Average income $18,000Average investment: $180,000 / 2 = $90,000Book rate of return = $18,000 / $90,000 = 20%

c. NPV: $6,930CashFactorPV130,000.89326,790260,000.79747,820390,000.71264,080460,000.63638,160530,000.567 17,010193,860Investment180,000NPV13,860

271 . a. Payback period: 3.15 years (75,000 + 90,000 + 115,000 + .15 x 130,000)

b. Book rate of return: 33.3%Average return: $100,000 ($600,000 total / 6 years)Depreciation: 50,000 ($30,000 / 6 years)Average income $50,000Average investment: $300,000 / 2 = $150,000Book rate of return = $50,000 / 150,000 = 33.3%

c. NPV: $130,530CashFactorPV175,000.90968,175290,000.82674,3403115,000.75186,3654130,000.68388,7905100,000.62162,10090,000.56450,760430,530Investment300,000PV130,530

272 . (1) $100,000/$16,000 = 6.25 years payback period(2) [($160,000 - $90,000)/10 yrs.] / [($100,000 + $10,000)/2] = 12.73% accounting rate of return on average investment(3) Net present value = PV of ten annual cash inflows of $16,000 + PV of inflow from salvage in tenth year – Original investment$2,180 = ($16,000 x 6.145 + ($10,000 x 0.386) – ($100,000)

CMA EXAMINATION QUESTIONS Page 138 of 155

Page 139: P02 - Capital Budgeting

MANAGEMENT ADVISORY SERVICES CAPITAL BUDGETING

274120. (a)Cash flow YearAmountDiscount factorPresent valueInvestment0$(100,000) 1.00 $(100,000) Working cap. 0$(200,000) 1.00 (200,000)Cash inflow1100,000 .8621 86,210Cash

inflow2150,000 .7432 111,480Cash inflow3200,000 .6407 128,140Cash inflow4200,000 .5523 110,460Working cap. 4200,000 .5523 110,460Net present value$246,750(b) After the first two years, $250,000 of the original $300,000 investment would be recouped. It would take one-quarter of the third year ($50,000/$200,000) to recoup the last $50,000. Thus, the payback period is 2.25 years.

275 . a. Annual cash inflow $ 60,000Present value factor for 10 years x 5.650 Initial investment $339,000

b. Payback period = $339,000/$60,000 = 5.65 years

c. Initial investment $62,900PV of salvage value ($10,000 x 0.275) (2,750) Net PV of annual net cash inflow $60,150

Annual cash inflow = $60,150/3.020 = $19,917.22

d. Payback = $62,900/$19,917.22 = 3.158

e. Annual net cash inflow = $226,000/5.650 = $40,000

f. PV factor for 10 years = $226,000/$40,000 = 5.650

Look up value 5.650 in PV of annuity table under 10 years and the internal rate of return is 12%.

276 . a. Annual cash flow: 60% x [125,000 x ($200 - $130)] – (60% x $6,000,000) + (40% x $7,200,000/4) = $2,370,000

NPV: [($2,370,000 x 3.037) - $7,200,000 - 500,000 + ($500,000 x .636)] = ($184,310)CMA EXAMINATION QUESTIONS Page 139 of 155

Page 140: P02 - Capital Budgeting

MANAGEMENT ADVISORY SERVICES CAPITAL BUDGETING

277 . a.Predicted

Cash FlowsYear(s)PV FactorPV of Cash FlowsInitial investment$(20,960)01.000$(20,960)Annual operations5,000104.83324,165 Net present value$ 3,205b. Present value factor of an annuity of $1.00 = $20,960/$5,000 = 4.192.

From annuity table, the 4.192 factor is closest to the 10-year row at the 20% column. Therefore, the IRR is 20%.

278 . a.Predicted

Cash FlowsYear(s)PV FactorPV ofCash FlowsInitial investment$(95,000)01.000$(95,000)Annual operations, net18,0001 - 84.63983,502Salvage value, work cap14,50080.3515,090 Net present value$ (6,408)

b. Trial and error is necessary. You know it is below 14% because the answer to Part A was negative and, therefore, less than the discount rate. Therefore, let's try 12%.Predicted

Cash FlowsYear(s)PV FactorPV OfCash FlowsInitial investment$(95,000)01.000$(95,000)Annual operations, net18,0001 - 84.968 89,424Salvage value, work cap14,50080.404 5,858 Net present value$ 282

The (almost) zero net present value indicates an internal rate of return of approximately 12%.

279 . a. NPV: $78,000 [(5.650 x $120,000) - $600,000]b. Payback period: 5 years ($600,000/$120,000)c. IRR: 15% (5.0 is about halfway between 5.216 and 4.833)

280 . a. NPV: $36,300 [(3.605 x $60,000) - $180,000]b. Payback period: 3 years ($180,000/$60,000)c. IRR: between 18 % and 20% (3.0 is between 3.127 and 2.991)

281 . a.Predicted

Cash FlowsYear(s)PV FactorPV ofCash FlowsInvestment$(36,586)01.000$(36,586)Working capital needed(4,000)01.000(4,000)Annual operations10,0001-53.43334,330Working capital returned4,00050.5192,076Salvage value4,00050.5192,076Net present value$(2,104)

b. Trial and error is required. Because net present value is negative in part a, the internal rate of return is less than 14%. Start by trying 12%.Predicted

CMA EXAMINATION QUESTIONS Page 140 of 155

Page 141: P02 - Capital Budgeting

MANAGEMENT ADVISORY SERVICES CAPITAL BUDGETING

283 . a. Since depreciation is the only noncash item on the income statement, the net annual cash flow can be computed by adding back depreciation to net income.Net income ….......... $200,000Depreciation …........ 100,000Net annual cash flow … $300,000

PresentYears Amount 12% Factor Value

Initial investment Now $(1,000,000) 1.000 $(1,000,000)Net annual cash flows 1-10 300,000 5.650 1,695,000 Net present value $ 695,000

b. The formula for computing the factor of the internal rate of return (IRR) is:Investment required ÷ Net annual cash inflow = Factor of the IRR $1,000,000 ÷ $300,000 = 3.333

26% factor …..... 3.465 3.465 True factor ….... 3.333 28% factor _____ 3.269

0.132 0.196

26% + 2%(0.132 ÷0.196) = 27.3%

c. The formula for the payback period is: Investment required ÷ Net annual cash inflow = Payback period $1,000,000 ÷ $300,000 = 3.33 years

d. The formula for the simple rate of return is: Net income ÷ Initial investment = Simple rate of return $200,000 ÷ $1,000,000 = 20.0%

CMA EXAMINATION QUESTIONS Page 141 of 155

Page 142: P02 - Capital Budgeting

MANAGEMENT ADVISORY SERVICES CAPITAL BUDGETING

285 . a. Year Amount 10% Factor Present ValueInvestment required now ($25,000) 1.000 ($25,000)Annual cost savings 1-15 3,500 7.606 26,621Salvage value ..... 15 1,000 0.239 239Net present value $ 1,860

b. Year Amount 10% Factor Present ValueAnnual cash inflows 1-8 $ 5,000 5.335 $26,675Since the present value of the cash inflows is $26,675, the company should be willing to pay up to this amount to acquire the machine.

c. Investment required ÷ Net annual cash flow = Factor of the internal rate of return $30,000 ÷ %6,000 = 5.000 14% factor ............ 5.216 5.216 True factor ........... 5.000 16% factor ............ 4.833 0.216 0.383

14% + 2%(0.216 ÷ 0.383) = 15.1%

The machine should be purchased, since the internal rate of return is greater than the required rate of return.

286 . a. The present value of the $12,000 annuity is found by multiplying $12,000 by the annuity discount factor associated with 6 percent interest for four years: $12,000 3.4651 = $41,581.20.From the information on the profitability index, it is known that the present value of the cash inflows is 1.03953 times the initial investment. Thus, the initial investment is $41,581.20/1.03953 = $40,000.

b. By dividing $40,000 by the annual cash inflow of $12,000, it is determined that the discount factor associated with the IRR is 3.3333. This discount factor is associated with an interest rate that lies between 7 and 8 percent. Using interpolation, the IRR is computed to be approximately 7.72 percent.

287 . a. Annual cash flows: $27,000 [$35,000 - 40% x ($35,000 - $15,000)]b. Payback period: 5.56 years ($150,000/$27,000)

CMA EXAMINATION QUESTIONS Page 142 of 155

Page 143: P02 - Capital Budgeting

MANAGEMENT ADVISORY SERVICES CAPITAL BUDGETING

289 . a. Income before taxes, 250,000 x ($60 - $40) - $3,000,000 - $5,000,000/4 $ 750,000Income tax (300,000)Net income $ 450,000Plus depreciation 1,250,000Net cash flow $1,700,000

b. Payback period: 2.94 years ($5,000,000/$1,700,000)

c. NPV: $389,000 [($1,700,000 x 3.170) - $5,000,000]

290 . a. Annual cash flows: $46,400 [$56,000 - 40% x ($56,000 - 32,000)]b. Payback period: 3.45 years ($160,000/$46,400)c. NPV: $7,272 [($46,400 x 3.605) - $160,000]

291 . a. Income before taxes, [200,000 x ($40 - $27) - $1,500,000 - $3,500,000/4] $ 225,000Income tax (40%) ( 90,000)Plus depreciation 875,000Net cash flow $1,010,000

b. Payback period: 3.47 years ($3,500,000/$1,010,000)

c. NPV: negative $298,300 [($1,010,000 x 3.170) - $3,500,000]

292 . a. Increase in annual net cash flow: $148,000 [$200,000 - 40% x ($200,000 - $70,000)]b. Profitability index: 1.19 [($148,000 x 5.65)/$700,000]

293 . a. Increase in annual net cash flow: $22,000 [$30,000 - (40% x ($30,000 - $10,000)]b. Profitability index: 1.24 [($22,000 x 5.65)/$100,000]c. Effect on profitability index: Increase (PI would increase because the tax shield of depreciation would occur earlier and so be more valuable when considering the time value of money.)

294 . a. Annual net cash flows: $57,600 [$80,000 pretax - 40% x ($80,000 - $24,000 depreciation)]b. NPV: $17,640 [($57,600 x 5.650) - $240,000 - $100,000 + ($100,000 x .322)]

CMA EXAMINATION QUESTIONS Page 143 of 155

Page 144: P02 - Capital Budgeting

MANAGEMENT ADVISORY SERVICES CAPITAL BUDGETING

295 . a. Annual net cash flows: $550,000 [$750,000 - 40% x ($750,000 - $250,000 depreciation)]b. NPV: $434,400 [($550,000 x 4.968) - $2,000,000 - $500,000 + ($500,000 x .404)]c. PI: 1.17 {[($550,000 x 4.968) + ($500,000 x .404)]/($2,000,000 + $500,000)}

296 . a. Annual net cash flows: $2,325,000[$2,875,000 pretax - 40% x ($2,875,000 - $1,500,000 depreciation)]

pretax income = 275,000 x ($60 - $35) - $4,000,000 = $2,875,000

b. NPV: $775,050 [($2,325,000 x 2.914) - $6,000,000]

c. Allowable loss of X-10 sales, approximately 36,941 units [($775,050/2.914)/60%]/12

d. Allowable error in per-unit VC, $1.61{[($775,050/2.914)/60%]/275,000 units}

297 a. Annual net cash flows: $26,200 [$33,000 pretax - 40% x ($33,000 - $16,000 depreciation)]b. NPV: Negative $11,970 [($26,200 x 5.650) - $160,000]c. IRR: between 10% and 12% [factor of 6.107 (160,000/26,200) is between 6.145 and 5.650]

298126.a.Revenue$100,000 - cash expenses (60,000 )Annual inflow$ 40,000 b.PV inflow $40,000 4.4941 =$179,764 PV outflow $160,000 1.0 =(160,000) NPV =$ 19,764 c.IRR factor = $160,000/$40,000 = 4.0 which is approximately 23%d.Payback = $160,000/$40,000 = 4 yrs.e.$179,764/$160,000 = 1.123525f.Car wash exceeds minimum on SRR and IRR, but not payback.

299 . Present value of keeping current system:Predicted

Cash FlowsYear(s)PV FactorPV ofCash FlowsOverhaul$(40,000)01.000$ (40,000)Annual operations(70,000)1-52.991(209,370)Salvage value5,00050.4022,010 Net present value$(247,360)

Present value of new system:CMA EXAMINATION QUESTIONS Page 144 of 155

Page 145: P02 - Capital Budgeting

MANAGEMENT ADVISORY SERVICES CAPITAL BUDGETING

300 . a. Payback MethodProposal AProposal BProposal CYear 1$80,000$45,000$90,000Year 2 10,000 45,000 .$90,000$90,000$90,000Payback 2 years2 years1 year

Net Present Value

Proposal A:PredictedCash FlowsYear(s)PV FactorPV of

Cash FlowsInvestment$(90,000)01.000$(90,000)Annual operations:Year 1 80,00010.877 70,160Year 2 10,00020.769 7,690Year 3 45,00030.675 30,375 Net present value$ 18,225

Proposal B:PredictedCash FlowsYear(s)PV FactorPV of

Cash FlowsInvestment$(90,000)01.000$(90,000)Annual operations:Year 145,00010.87739,465Year 245,00020.76934,605Year 345,00030.67530,375Net present value$ 14,445

Proposal C:PredictedCash FlowsYear(s)PV FactorPV of

Cash FlowsInvestment$(90,000)01.000$(90,000)Annual operations:Year 190,00010.87778,930 Net present value$ 11,070Accrual Accounting Rate of Return:Proposal A: [80,000 + 10,000 + 45,000)/3 - (90,000/3)]/90,000 = 0.167

Proposal B: (45,000-30,000)/90,000 = 0.167

Proposal C: (90,000- 90,000)/90,000 = 0.0

b. Summary:MethodProposal AProposal BProposal CPayback method ranks2.52.51.0Net present value1.02.03.0AARR1.51.53.0Even though Proposal C is Number 1 for payback, it comes in last with the other two methods. Because the net present value method takes into account the time value of money and the other proposals are less comprehensive, Proposal A would be the best alternative.

301125.a.Old loss $(2,000)New receipts $20,000 5% =$ 1,000 Depr. $12,000/10 yrs. = (1,200 )New (Loss)$ (200)b.Change in annual cash inflow is $3,000Payback = $12,000/$3,000 = 4 yrs.c.1.PV of inflow $3,000 4.1925 =$12,577.50 PV of outflow $12,000 1.0 =(12,000.00)NPV$ 577.50 2.IRR is approximately 23%d.Change in inflow = $2,700 PV inflow $2,700 4.1925 =$11,319.75 PV outflow $12,000 1.0 =(12,000.00) NPV$ (680.25 )e.$12,000/4.1925 = $2,862.25Receipts = ($2,862.25 - $2,000)/.05 = $17,245

CMA EXAMINATION QUESTIONS Page 145 of 155

Page 146: P02 - Capital Budgeting

MANAGEMENT ADVISORY SERVICES CAPITAL BUDGETING

Cash FlowsYear(s)PV FactorPV ofCash FlowsInvestment$(36,586)01.000$(36,586)Working capital needed (4,000)01.000 (4,000)Annual operations 10,0001-53.605 36,050Working capital returned 4,00050.567 2,268Salvage value 4,00050.567 2,268Net present value$-0-

303118. No matter what happens, the tax rate for the next two years is 30 percent. Using the differences in depreciation amounts, one can determine the difference in present values between the two methods at the end of year 2 when the tax rate is expected to change.

Present value calculations for years 1 and 2:Year 1 ($20,000) .30 .9091 =$ (5,455)Year 2 ($25,000) .30 .8265 =$ (6,199 )Total present value difference at end$(11,654)So, after the first two years, Method 1 has generated $11,654 more present

value than Method 2. This simply means that at the point of indifference, Method 2 would be required to generate $11,654 more present value than Method 1 in the last three years. For the last three years of the project's life, the difference in depreciation amounts is $15,000. This $15,000 amount can be used in the following equation to solve for the tax rate that yields a present value of $11,654:

$11,654 = $15,000 tax rate (.7513 + .6830 + .6209)$11,654 = $30,828 tax rateTax rate = $11,654/$30,828Tax rate = 37.8%

Thus, an increase in the tax rate to about 37.8 percent would cause management to be indifferent between the two depreciation methods.

304. Dividing $200,000/$60,000, gives the annuity discount factor (3.3333) for 11 percent associated with the minimal required time for this project to be successful. According to the tables in Appendix A, the project will have a positive net present value if the cash flows last through year 5.

305 . (1)

YearUnadjusted Estimate of Cash InflowsInflation AdjustmentInflation Adjusted Estimate of Cash Inflows1$50,0001.100$55,000240,0001.21048,400330,0001.33139,930$120,000$143,330(2)

YearUnadjusted Cash FlowsAdjusted Cash FlowsPV of $1@ 16%PV of Unadjusted Cash FlowsPV of Adjusted Cash Flows0$(100,000)$(100,000)1.000$(100,000)$(100,000)150,00055,000 .862 43,100 47,410240,00048,400 .743 29,720 35,961330,00039,930 .641 19,230 25,595Net present value of investment$ (7,950)$ 8,966

306.

YearEstimated Net Pretax Cash Inflows4% Annual Price-level AdjustmentPrice-level Adjusted Net Cash Inflows1$10,000(1 + .04)1 = 1.040$10,400215,000(1 + .04)2 = 1.08216,230315,000(1 + .04)3 = 1.12516,875415,000(1 + .04)4 = 1.17017,550510,000(1 + .04)5 = 1.21712,17065,000(1 + .04)6 = 1.2656,325

Total price-level adjusted net pretax cash inflows from operations $79,550Plus cash inflow from salvage $2,500

CMA EXAMINATION QUESTIONS Page 146 of 155

Page 147: P02 - Capital Budgeting

MANAGEMENT ADVISORY SERVICES CAPITAL BUDGETING

With a zero net present value, the internal rate of return is 12%.

c. Payback period = ($36,586 + $4,000)/$10,000 = 4.06 years.b. PI: 0.976 [($2,370,000 x 3.037 + 500,000 x .636)/($7,200,000 + 500,000)]

273123.307.

(1)(2)(1) x (2) = (3)YearPeriodic

Cash Inflows4% Price-level AdjustmentInflation Adjusted Estimate of Cash Inflows1$25,000(1 + .04)1 = 1.040$26,000227,000(1 + .04)2 = 1.08229,214329,000(1 + .04)3 = 1.12532,625423,000(1 + .04)4 = 1.17026,910520,000(1 + .04)5 = 1.21724,340615,000(1 + .04)6 = 1.26518,975$ 139,000$ 158,064

(1)(2)(1) x (2) = (3)YearDepreciable Basis of Property5-Year Property Recovery Percentage

Tax Depreciation1$100,0000.200$20,0002100,0000.32032,0003100,0000.19219,2004100,0000.11511,5005100,0000.11511,5006100,0000.0585,800$ 100,000(1)(2)(1) - (2) = (3)(3) x (4) (5)(1) - (5) (6)(4)YearAdjusted Estimate of Net Cash Inflows

Tax DepreciationTaxable Income (Loss)Income

TaxNet After-tax Cash InflowsFederal and State Income Tax Rate1$26,000$20,000$ 6,000$ 2,400$23,60040%229,21432,000(2,786)

(1,114)30,32840%332,62519,20013,4255,37027,25540%426,91011,50015,4106,16420,74640%524,34011,50012,8405,13619,20440%618,9755,80013,1755,27013,70540%Total estimated net after-tax cash inflows from project $134,838Less initial cash outflow for machinery 100,000Excess of after-tax cash inflows from project over initial cash outflow $ 34,838

308.

Funds—SourceProportion of FundsAfter-tax CostWeighted CostBonds.25.071.018Preferred stock.25.152.0375Common stock and retained earnings.50.143.0701.00.1255or 12.55%Computations:

1.12 - (.12 x .4)2$15/$100 = .153$105,000/25,000 = $4.20; $4.20/$30 = .14

CMA EXAMINATION QUESTIONS Page 147 of 155

Page 148: P02 - Capital Budgeting

MANAGEMENT ADVISORY SERVICES CAPITAL BUDGETING

a.Cost savings per year$5,000 Maintenance per year (700 )Net cash flows per year$4,300

CashDiscount factorPresent value$30,0001.0000$(30,000.00) 4,3005.5348 23,799.64 Net present value of investment$ (6,200.36)

b.Payback equals $30,000/$4,300 = 6.976 years

Project C:Cash investment now ($21,000) 1.000 ($21,000)Annual cash inflow for 4 years $11,000 3.037 $33,407Cash outflow at the end of 3 years ($ 5,000) 0.712 ($ 3,560)Additional cash inflow at the end of 4 years $15,000 0.636 $ 9,540Net present value................ $18,387

245 . REQUIRED: The reason for the increases in the MCC.DISCUSSION: (D) The MCC is a weighted average of the costs of the different financing sources. If the cost of any source of financing increases, the MCC curve will rise. The MCC curve is upward sloping because the lowest cost financing sources are assumed to be used first. Thus, as cumulative debt increases, the cost of debt also increases.Answers (A) and (B) are incorrect because financing costs do not depend on rates of return on investment. Answer (C) is incorrect because, as additional funds are raised, an increase in the cost of a source of financing, not a decrease, will result in an increase in the MCC.

219 . 5.650 x $750,000 - $4,500,000 = $(262,500)

220 . $4,500,000/$750,000 = 6.000, which is the pv factor for n = 10, and I greater than 10 percent, but less than 12 percent.

212 . Answer (C) is correct. The profitability index is the present value of the future net cash flows divided by the present value of the net initial investment. The present value of the future net cash flows is $837,200. Dividing the $800,000 initial cost of the investment into the $837,200 present value of the future net cash inflows produces a profitability index of

approximately 1.05. Any profitability index greater than one is considered acceptable. Answer (A) is incorrect because 0.05 results from using the NPV in the numerator. Answer (B) is incorrect because 0.96 reverses the numerator and denominator of the calculation. Answer (D) is incorrect because 1.25 uses undiscounted earnings in the numerator instead of the present value of future net cash flows.

($85,000 – $16,000). The $60,000 (40% x $150,000)depreciation deduction in the second year results in $25,000 ($85,000 - $60,000) of taxable income. Second year tax expense is therefore $10,000 ($40,000 x 25%), and the second year net cash inflow is $75,000 ($85,000 - $10,000). The net cash inflow from the sale of the machine is $10,000. No tax is paid on this amount because the remaining book value is also $10,000. The present value of these net cash inflows is determined using the appropriate PV of $1 factors for a hurdle rate of 16%. Hence, the net present value is $5,842 ($165,842 PV - $160,000 cost).$69,000 x .862=$59,478$75,000 x .743=55,725$69,000 x .641=44,229$10,000 x .641= 6,410Present value of net cash inflows$165,842Answer (A) is incorrect because $3,278 assumes that taxes must be paid on the salvage value. Answer (C) is incorrect because ($568) omits salvage value from the calculation. Answer (D) is incorrect because $30,910 equals the present value of a 3-year annuity of $85,000 discounted at 16%, minus $160,000.

198 . Answer (B) is correct. The NPV method discounts the expected cash flows from a project using the required rate of return. A project is acceptable if its NPV is positive. Based on the interest factors for the present value of $1 at 12% and the annual after-tax cash flows, the NPV of the project over its 5-year life is $240,000 x .89 = $ 213,600216,000 x .80 =172,800192,000 x .71 =136,320168,000 x .64 =107,520144,000 x .57 = 82,080Total present value$ 712,320Purchase cost (500,000) NPV$ 212,320Answer (A) is incorrect because $304,060 is the present value of the net income amounts. Answer (C) is incorrect because $(70,000) is the excess of the net initial investment over the sum of the undiscounted net income amounts. Answer (D) is incorrect because $712,320 is the present value of the cash inflows.

CMA EXAMINATION QUESTIONS Page 148 of 155

Page 149: P02 - Capital Budgeting

MANAGEMENT ADVISORY SERVICES CAPITAL BUDGETING

based on average book value, nor did they treat expenses properly. Answer (D) is incorrect because 35% is not based on average book value, nor did they treat expenses properly.

180 . Answer (C) is correct. The discounted payback period is the number of years needed to get the PV of the cash flows to equal the initial investment. At a 14% discount rate, this occurs at 9.2 years. Answer (A) is incorrect because it takes 9.2 years to reach the discounted payback period. Answer (B) is incorrect because it takes 9.2 years to reach the discounted payback period. Answer (D) is incorrect because it takes 9.2 years to reach the discounted payback period.

Answer (A) is incorrect because 20.4% is the accounting rate of return using the original investment. Answer (B) is incorrect because 34.0% results from adding the residual value to the original investment. Answer (D) is incorrect because 51.0% assumes an average investment of $100,000.

161 . $9,000/(.5 x $20,000) = 90%

162 . NPV = ($30,000 - $16,000) x 2.914 - $20,000 = $20,796

146 . Answer (A) is correct. Gunning uses straight-line depreciation. Thus, the annual charge is $38,000 [($160,000 + $30,000) ÷ 5 years], and the tax savings is $15,200 (40% x $38,000). That benefit will be received in 1 year, so the present value is $13,817 ($15,200 tax savings x .909 present value of $1 for 1 year at 10%). Answer (B) is incorrect because $16,762 is greater than the undiscounted tax savings. Answer (C) is incorrect because $20,725 assumes a 60% tax rate (the complement of the actual 40% rate). Answer (D) is incorrect because $22,800 assumes a 60% tax rate and no discounting.

147 . Answer (D) is correct. The new machine will increase sales by 20,000 units a year. The increase in the pretax total contribution margin will be $200,000 per year [20,000 units x ($40

SP - $30 VC)], and the annual increase in the after-tax contribution margin will be $120,000 [(1.0 - .4) x $200,000]. The present value of the after-tax increase in the contribution margin over the 5-year useful life of the machine is $454,920 ($120,000 x 3.791 PV of an ordinary annuity for 5 years at 10%). Answer (A) is incorrect because $242,624 deducts fixed costs from the pretax contribution margin and applies a 60% tax rate. Answer (B) is incorrect because $303,280 deducts fixed costs from the after-tax contribution margin before discounting. Answer (C) is incorrect because $363,936 deducts fixed costs from the contribution margin before calculating taxes and the present value.

Answer (B) is incorrect because NPV and IRR make inconsistent accept/reject decisions for independent projects. When NPV is positive, IRR exceeds the cost of capital and the project is acceptable. Answer (V) is incorrect because the NPV method can be used to rank mutually exclusive projects, whereas IRR cannot. The reinvestment rate assumptions causes IRR to make faulty project ranking under some circumstances. Answer (D) is incorrect because IRR is expressed as a percentage and NPV in dollar terms.

criterion if they are not mutually exclusive. Use of the profitability index yields a similar decision because a project with an index greater than 100% should be undertaken. Answer (A) is incorrect because Project 3 has a negative NPV. Answer (B) is incorrect because Project 3 has a negative NPV. Answer (D) is incorrect because Project 4 has a positive NPV and should be undertaken.

133 . Answer (D) is correct. With only $12,000,000 available, and each project costing $4,000,000 or more, no more than two projects can be undertaken. Accordingly, Projects should be selected because they have the greatest NPVs and profitability indexes. Answer (A) is incorrect because Project 3 has a negative NPV. Answer (B) is incorrect because choosing three projects violates the $12,000,000 limitation. Answer (C) is incorrect because the combined NPV of Projects 1 and 4 is less than the combined NPV of Projects 1 and 2.

CMA EXAMINATION QUESTIONS Page 149 of 155

Page 150: P02 - Capital Budgeting

MANAGEMENT ADVISORY SERVICES CAPITAL BUDGETING

134 . Answer (C) is correct. With only $6,000,000 available, and each project costing $4,000,000 or more, no more than one project can be undertaken. Project 1 should be chosen because it has a positive NPV and the highest profitability index. The high profitability index means that the company will achieve the highest NPV per dollar of investment with Project 1. The profitability index facilitates comparison of different-sized investments. Answer (D) is incorrect because, despite having the highest NPV, Project 2 has a lower profitability index than Project 1. Consequently, Project 1 offers the greater return per dollar of investment. Answer (A) is incorrect because Project 3 has a negative NPV and should not be selected regardless of the capital available. Answer (B) is incorrect because selecting two projects violates the $6,000,000 limitation on funds.

135 . Answer (C) is correct. A company using the NPV method should undertake all projects with a positive NPV, unless some of those projects are mutually exclusive. Given that Projects 2, 3, and 4 have positive NPVs, they should be undertaken. Project 1 has a negative NPV. Answer (A) is incorrect because Project 1 has a negative NPV and should not be undertaken. Answer (B) is incorrect because Project 1 has a negative NPV and should not be undertaken. Answer (D) is incorrect because Project 1 has a negative NPV and should not be undertaken.

Financial calculator solution:Project A: Inputs: CF0 = -100,000; CF1 = 39,500; Nj = 3. Output: IRRA = 8.992% 9.0%.

Project B: Inputs: CF0 = -100,000; CF1 = 0; Nj = 2; CF2 = 133,000. Output: IRRB = 9.972% 10.0%.

= -8.013 = -$8,013.

Financial calculator solution (in thousands):Project X: Inputs: CF0 = -100; CF1 = 50; CF2 = 40; CF3 = 30; CF4 = 10; I = 15. Output: NPVX = -0.833 = -$833.

Project Z: Inputs: CF0 = -100; CF1 = 10; CF2 = 30; CF3 = 40; CF4 = 60; I = 15. Output: NPVZ = -8.014 = -$8,014.

125.

Tabular solution:NPVA = $15,625(PVIFA10%,5) - $50,000 = $15,625(3.7908) - $50,000 = $59,231.25 - $50,000 = $9,231.25.

NPVB = $99,500(PVIF10%,5) - $50,000 = $99,500(0.6209) - $50,000 = $61,779.55 - $50,000 = $11,779.55.

NPVB > NPVA; $11,779.55 > $9,231.25; Choose Project B.

Financial calculator solution:Project A: Inputs: N = 5; I = 10; PMT = 15,625. Output: PV = -59,231.04. NPVA = $59,231.04 - $50,000 = $9,231.04.

Project B: Inputs: N = 5; I = 10; FV = 99,500. Output: PV = -61,781.67. NPVB = $61,781.67 - $50,000 = $11,781.67.

CMA EXAMINATION QUESTIONS Page 150 of 155

Page 151: P02 - Capital Budgeting

MANAGEMENT ADVISORY SERVICES CAPITAL BUDGETING

Alternate method by cash flowsProject A: Inputs: CF0 = -50,000; CF1 = 15,625; Nj = 5; I = 10. Output: NPV = $9,231.04.

Project B: Inputs: CF0 = -50,000; CF1 = 0; Nj = 4; CF2 = 99,500; I = 10. Output: NPV = $11,781.67.Solve for crossover rate using the differential project CFs, CFA-B

Inputs: CF0 = 0; CF1 = 15,990; Nj = 4; CF2 = -84,570.Output: IRR = 11.49%. The crossover rate is 11.49%.Tabular solution:Solve for numerical PVIFA and PVIF and obtain corresponding interest rates from table.Project A: 50,000= 15,990(PVIFAIRRA,5) 3.12695 = PVIFAIRRA,5

IRRA 18%Project B: 50,000= 100,560(PVIFIRRB,5) 0.49722 = PVIFIRRB,5

IRRB 15%Solving for the crossover rate of 11.49% requires interpolation, which is not covered in the text. However, by using trial and error and an NPV profile drawing, the student can select the correct multiple choice answer, 11.5%. Drawing an NPV profile drawing using the calculated IRRs, and the NPVs at k = 0%, shows that there is a crossover rate. Of the responses listed in the problem, 16.5% and 20.0% are clearly too high, since the IRRB is 15%. At k = 6.5% the NPVs are not equal, thus 11.5% must be the correct response.

116 . Answer (A) is correct. The NPV of both machines must be calculated and compared to determine which will yield a better return of cash flows. Machine A is calculated as one lump sum payable in 4 years minus the initial investment cost.

The NPV of Machine B is calculated as the present value of an ordinary annuity of $13,000 for 4 years, minus the initial investment cost.

By comparing the NPV of both machines, Cliff would choose Machine A because NPVA > NPVB by $1,044. Answer (B) is incorrect because Machine A is a better choice. Answer (C) is incorrect because the difference is $1,044. Answer (D) is incorrect because Machine A has the higher NPV.

117 . Answer (D) is correct. The NPV of the electric bus is $27,801, which is greater than that of two gas-powered busses bought 4 years apart. The NPV for the $90,000 electric bus involves multiplying the $28,000 annual cash flows times the present value factor of 4.2072, which equals $117,801. Deducting the $90,000 initial cost results in an NPV of $27,801. The NPV for the two gas-powered busses is $8,205, calculated as follows:$22,000 x 4.2072$92,558.40- First bus55,000.00- Second bus (55,000 x .5337)29,353.50 NPV $ 8,204.90Answer (A) is incorrect because it is based on only the first four years for the

CMA EXAMINATION QUESTIONS Page 151 of 155

Page 152: P02 - Capital Budgeting

MANAGEMENT ADVISORY SERVICES CAPITAL BUDGETING

gas-powered bus. Answer (B) is incorrect because it did not discount the purchase of the second gas-powered bus. Answer (C) is incorrect because it is simply a doubling of the NPV for the first four years of the gas-powered bus.

At the crossover rate, NPVA = NPVB; NPVA - NPVB = 0.NPVA = $4.22082 million; NPVB = CF0 + $8.73167 million.$4.22082 - CF0 - $8.73167 = 0 -CF0 = $8.73167 - $4.22082 CF0 = -$4.51085 million.

114 . Answer (C) is correct. The cost of capital at which the two projects will produce the same NPV can be found by calculating the IRR of the difference in cash flows between the two projects. Proposal A requires an additional investment of $53,000 and generates extra cash flows of $15,000 for 6 years. The IRR for this set of cash flows exceeds 16% but is less than 18%. This further means that, for any cost of capital below this range, Proposal A will have a higher NPV, and, for any cost of capital above this range, Proposal B will have a higher NPV. Answer (A) is incorrect because Proposal A would be superior. Answer (B) is incorrect because Proposal A would be superior. Answer (D) is incorrect because Proposal B would be superior. Proposal A would have a negative NPV with regard to the incremental cash flows.

inflow is therefore $68,000 ($80,000 - $12,000), and the payback period is 3.68 years ($250,000 investment ÷ $68,000). Answer (A) is incorrect because the payback period is 3.68 years. Answer (B) is incorrect because the payback period is 3.68 years. Answer (D) is incorrect because the payback period is 3.68 years.

110

?. Modified IRR Answer: d Diff: M

Tabular solution:TV = $73,306(FVIFA12%,8) = $73,306(12.300) = $901,663.80.

$275,000 =

0.30499 =

(1 + MIRR)8 = (FVIFIRR,8) = 3.27869.

Look in table: Periods = 8, I = 16%. MIRR = 16%.

Alternate method3.278691/8 = 1 + MIRR MIRR = 16%.

Financial calculator solution:TV Inputs: N = 8; I = 12; PMT = 73,306. Output: FV = -$901,641.31.MIRR Inputs: N = 8; PV = -275,000; FV = 901,641.31. Output: I = 16.0%.

Alternate methodInputs: CF0 = 0; CF1 = 73,306; Nj = 8; I = 12. Output: NFV = $901,641.31.Inputs: CF0 = -275,000; CF1 = 0; Nj = 7; CF2 = 901,641.31.Output: IRR = 16.0% = MIRR.

111. MIRR and CAPM Answer: d Diff: MCMA EXAMINATION QUESTIONS Page 152 of 155

Page 153: P02 - Capital Budgeting

MANAGEMENT ADVISORY SERVICES CAPITAL BUDGETING

Step 1: Calculate the historical beta.Regression method: Financial calculator: Different calculators have different list entry procedures and key stroke sequences.Enter Y-list: Inputs: Item(1) = 9 INPUT; Item(2) = 15 INPUT; Item(3) = 36 INPUT.Enter X-list: Inputs: Item(1) = 6 INPUT; Item(2) = 10 INPUT; Item(3) = 24 INPUT; use linear model.Output: m or slope = 1.50.Graphical/numerical method:Slope = Rise/Run = (36 - 9)/(24 - 6) = 27/18 = 1.5. Beta = 1.5.

Step 2: Calculate cost of equity using CAPM and beta and given inputs:ke = kRF + Beta(MRP) = 7.0% + 1.5(6%) = 16.0%.

Step 3: Calculate MIRR: Tabular/numerical solution:Calculate terminal value (TV):TV = $1,000(FVIFA16%,3) = $1,000(3.5056) = $3,505.60.Use PV (cost) and TV to solve for MIRR: $2,028 = TV/(1 + MIRR)3

$2,028 = $3,505.60/(1 + MIRR)3

(1 + MIRR)3 = 1.7286

1 + MIRR = 1.20 MIRR = 20%.

Financial calculator solution:Terminal value: using cash flowsInputs: CF0 = 0; CF1 = 1000; Nj = 3; I = 16. Output: NFV = $3,505.60.MIRR: using TVMInputs: N = 3; PV = -2,028; PMT = 0; FV = 3,505.60.Output: I = 20.01 = MIRR 20%.

112 ROR = [(1 x 1.09)5 x 93%]/9% = 1.54 x 93% / 9% = 15.91% = 16%

233 . Answer (C) is correct. The cash inflow occurs 5 years after the cash outflow, and the NPV method uses the firm's cost of capital of 18%. The present value of $1 due at the end of 5 years discounted at 18% is .4371. Thus, the NPV of project A is $(265,460) [(.4371 x $7,400,000 cash inflow) - $3,500,000 cash outflow]. Answer (A) is incorrect because $316,920 discounts the cash inflow over a 4-year period. Answer (B) is incorrect because $23,140 assumes a 16% discount rate. Answer (D) is incorrect because $(316,920) discounts the cash inflow over a 4-year period and also subtracts the present value of the cash inflow from the cash outflow.

246 . REQUIRED: The NPV of the new machine.DISCUSSION: (C) The #300,000 of annual savings discounted at 12% has a present value of $1,083,000 ($300,000 x 3.61 PV of an ordinary annuity for five periods at 12%). The net cost of the new machine is the $1,000,000 purchase price minus the $60,000 cash inflow from the sale of the old machine, or $940,000. The resulting NPV is $143,000 ($1,083,000 present value of future savings - $940,000 cash outlay).Answer (A) is incorrect because the $300,000 savings is an annual savings, which should be discounted at the present value of an annuity of $1 for 5 periods at 12% or 3.61 ($300,000 x 3.61 = $1,083,000). The $300,000 annual savings should not be discounted using the present value of $1 for 5 periods at 12%, or 0.57 ($300,000 x 0.57 = $171,000). Furthermore, the

CMA EXAMINATION QUESTIONS Page 153 of 155

Page 154: P02 - Capital Budgeting

MANAGEMENT ADVISORY SERVICES CAPITAL BUDGETING

properly discounted amount ($1,083,000) should be reduced by the net cash outlay ($1,000,000 - $60,000) for the new equipment. Answer (B) is incorrect because the NPV is equal to the present value of future cash savings minus net initial cash outlays, Answer (D) is incorrect because the cash inflows generated from the sale of old machinery should be added to the difference of the present value of cash inflows minus the purchase price of new machinery.

257.

YearEstimated Demand in Units

Unit Sales PriceUnit Variable CostUnit Contribution MarginNet Pretax Cash Inflows From

Sales110,000$15$7$8$80,000210,000157880,000310,000157880,000410,000157880,000510,000157880,000

Total net pretax cash inflows from sales $400,000Initial cash outflow (cost of asset) $250,000Less pretax estimated salvage value (50,000) 200,000Excess of net pretax cash inflows over cost $ 200,000

282 . (1) $100,000/$29,129 = 3.4 years payback period

(2) ($29,129 x 3.605) - $100,000 = $5,010 net present value

(3) $100,000/$29,129 = 3.433 discount factorPresent value of $1 received annually for 5 years at 14% interest has a factor equal to 3.433. The internal rate of return on the purchase is 14%.

(4) Yes. The net present value is positive and the internal rate of return (14%) is more than the discount rate (12%).

284 . (1) $45,000 initial outlay/$12,000 annual outflow = 3.75 payback years

(2) Cash inflow ($12,000 x 3.791) $45,492Investment 45,000Net present value $ 492

(3) $45,000 x .621 = $27,945

288 . a. Annual net cash flows: $14,800 [$18,000 pretax - 40% x ($18,000 - $10,000 depreciation)]

b. NPV: Negative $16,380 [($14,800 x 5.650) - $100,000]

c. PI: 1.052 {[($57,600 x 5.650) + ($100,000 x .322)]/($240,000 + $100,000)}

PredictedCash FlowsYear(s)PV FactorPV of

Cash FlowsInvestment$(100,000)01.000$(100,000)Salvage value, old20,00001.00020,000Annual operations(40,000)1-52.991(119,640)Salvage value20,00050.4028,040 Net present value$(191,600)

Buying the new equipment is the most desirable by $55,760 ($247,360 - $191,600).

302 .. a. Year Cash flow Discount factorPresent value1 $150,000 1.0000 $(150,000.00) 1 32,000 .9009 28,828.802 57,000 .8116 46,261.203 5,000 .7312 3,656.004 28,000 .6587 8,443.605 16,000 .5935 9,496.006 3,000 .5346 1,603.807 15,000 .4817 7,225.507 70,000 .4817 33,719.00 Net present value $ (766.10)

CMA EXAMINATION QUESTIONS Page 154 of 155

Page 155: P02 - Capital Budgeting

MANAGEMENT ADVISORY SERVICES CAPITAL BUDGETING

b. Profitability index equals present value of cash flows divided by investment: $149,233.90/$150,000 = .995

c. Payback period is 6.11 years, computed as follows:

Year Cash Flow Cumulative Cash Flow1 $32,000 $ 32,000 2 57,000 89,0003 5,000 94,0004 28,000 122,0005 16,000 138,0006 3,000 141,0007 85,000 226,000

$150,000 - $141,000 = $9,000/$85,000 = .11

d. The project is quantitatively unacceptable because it has a negative NPV, a less-than-one PI, and a payback period of over six years. However, the NPV and PI are extremely close to being acceptable. Because the new machine will provide XYZ zero-defect production, the investment may be desirable if additional qualitative factors are considered such as improved competitive position, customer satisfaction, goodwill generated, improved product quality and reliability, and a desire to be in the forefront of manufacturing capability. XYZ may want to attempt to quantify these benefits and reevaluate the machine's acceptability as an investment.

Price-level adjustment 1.265 3,163

Total price-level adjusted net pretax cash inflows over initial cash outflow $82,713Less initial cash outflow 50,000Excess of net pretax cash inflows over initial cash outflow $ 32,713

79 . Answer (D) is correct. The $100 increase is deductible for tax purposes. Therefore, the net effect is $65 after tax. At a 12% discount rate, the present value of $65 is $58.04. Thus, the net present value will decline by $58.04. Answer (A) is incorrect because it ignores income taxes and present values. Answer (B) is incorrect because it ignores the tax shield. Answer (C) is incorrect because it ignores present values.

CMA EXAMINATION QUESTIONS Page 155 of 155